You are on page 1of 100

Printed and Published By

Nakkheeran Gopal
On Behalf Of
Volume : 16 Issue : 06 ©lWY¬ - 2020
Nakkheeran Publications,
105, Jani Jahan Khan Road,
Royapettah, Chennai-14, Printed at
Nakkheeran Printers and Binders
Dsú[
No, 105, Jani Jahan Khan Road,
Royapettah, Chennai-14. úR£V UdLsùRôûL T§úYÓ 04
Editor : Nakkheeran Gopal
קRôL TWÜm LúWô]ô ûYWv úSôn 08
Sd¸Wu Tl°úL`u^ýdLôL
Sd¸Wu ©¬iPov & ûTiPov
105, _ô² _ôuLôu NôûX, Uj§V @W£u JWôiÓ §hPeLs 10
BWôVlúThûP, ùNuû]#14#p
@f£hÓ ùY°«ÓTYo Utßm
WôÔY §]j§u YWXôtß NôRû] 16
D¬ûUVô[o : Sd¸WuúLôTôp

ùL[WY A£¬Vo: Rªr Y[of£j Õû\ ®ÚÕLs 18


Dr. @¡Xu BWômSôRu
@Pp _p §hPm 20
A£¬Vo:
Fv.ùNpYWôw
SPl× ¨LrÜLs 22
ùTôßlTô£¬Vo:
úL.úLNYu SPl× ¨LrÜLs ®]ô#®ûPLs (¥NmTo # 2019) 36

@hûP Y¥YûUl×:
R. NWYQlùTÚUôs SPl× ¨LrÜLs ®]ô#®ûPLs (_]Y¬ # 2020) 40

BRr Y¥YûUl×: SPl× ¨LrÜLs PV¬d ϱl×Ls (_]Y¬ # 2020) 44


Fv. ûUdúLp vPô-u
Ød¡V SPl× ¨LrÜLs PV¬d ϱl×Ls
Sd¸Wu Tl°úL`uv 52
ùRôûXúT£: 044#4399 3032 (_]Y¬ 2019 ØRp ¥NmTo 2019 YûW)

ùRôûX SLp: 28485454
C#ùU«p:
nakkheeran2003@yahoo.com TNPSC £û\jÕû\ T¦LÞdLô] úRoÜ # 2019 81
www.nakkheeran.in (@Np ®]ôjRôs)
úR£V UdLsùRôûL
T§úYÓ

úR£V UdLsùRôûL T§úYÓ ùTVo


(N ational P opulation Register- NPR) ÅhÓj RûXYÚP]ô] D\Ü
FuTÕ Sôh¥u YZdLUô] RkûR«u ùTVo
Ï¥«ÚlTô[oL°u T§Ü AÏm. @mUô®u ùTVo
B§p Ï¥Ù¬ûUf NhPm, 1955
Uû]®«u ùTVo (§ÚUQUô]ôp)
Utßm Ï¥Ù¬ûU (T§Ü ùNnRp)
Tô-]m
A¡VYt±u ¸r Dsðo (¡WôUm /
ÕûQ SLWm), ÕûQ UôYhPeLs, ©\kR úR§
UôYhPm, Uô¨X Utßm úR£V §ÚUQ ¨ûX
@[®p úNL¬dLlThP RLYpLs ©\kR BPm
Ds[]. (Ï¥UdLs Utßm úR£V úR£Vm (@±®jRT¥)
@ûPVô[ @hûPL°u ùY°ÂÓ) YZdLUô] Y£l©Pj§u RtúTôûRV
®§Ls, 2003. ØLY¬
IÚ YZdLUô] Ï¥«ÚlTô[o RtúTôûRV ØLY¬«p Re¡«ÚdÏm
NPR Bu úSôdLeLÞdLôL, LPkR LôXm
Aß UôReLs @pXÕ @RtÏ ¨WkRW Ï¥«Úl× ØLY¬
úUtThP LôXUôL IÚ Dsðo ùRô¯p
Tϧ«p Y£jR IÚ STo @pXÕ
Lp®j Rϧ
@ÓjR Aß UôReLÞdÏ @kR
NPR Bu úTôÕ, IÚ
​​ T§X°lTYo
Tϧ«p Y£dL ®Úm×m IÚ STo F]
FkR AYQjûRÙm RVô¬dLj
YûWVßdLlTÓ¡\Õ.
úRûY«pûX. DsÕû\ @ûUfNo
Bk§Vô®u IqùYôÚ Ï¥ULû]
@ªj `ô @°jR @±dûL«uT¥,
Ùm T§ÜùNnÕ úR£V @ûPVô[
NPR RLYp ÑV Nôu\°dLlTÓm.
@hûPûV YZeL NhPm LhPôVUôL
@RôYÕ, T§X°jRYo YZe¡V FkR
ØVp¡\Õ.
RLYÛm BÚdÏm N¬Vô]Õ Fuß
Fu.©.Ao ×Õl©dÏm ùNVpØû\ LÚRlTÓ¡\Õ. AYQeLs @pXÕ
Bk§Vô®u T§Yô[o ù_]Wp Utßm TúVôùUh¬d úRûY«pûX.
Øu]ôs @§LôWléoY UdLs
NPR dLô] RLYpLû[ úNL¬dÏm
ùRôûL LQdùLÓl× AûQV¬u
ùNVpØû\ GlWp 2020#Bp
¸r úUtùLôs[lTÓm.
ùRôPe¡ ùNlPmTo UôRj§tÏs
Sôh¥Ûs[ IqùYôÚ YZdLUô] ¨û\YûPÙm. @Nôm R®W, Bk§Vô
Ï¥«ÚlTô[¬u ®¬Yô] @ûPVô[ ØÝYÕm NPR SPjRlTÓm, Gù]²p
RWÜjR[jûR DÚYôdÏYúR BRu Uô¨Xm GtL]úY úR£V Ï¥UdL°u
úSôdLm AÏm. RWÜjR[j§p BÕ T§Ü êXm ùNußs[Õ.
úTôu\ ×s°®YW ®YWeLs
NPR U t ß m UdLsùRôûL
BÚdÏm:
LQdùLÓl× ùNVpØû\ IúW

 A-Series in
          ©lWY¬ 2020
úSWj§p ùRôPeÏm úTôÕ, ​​BWiÓ @Yt±u UdLsùRôûL ϱjÕ
RWÜjR[eLÞm Iu\pX. ®¬Yô] LQdùLÓl× SûPùTßm.
Bk§V UdLsùRôûL«u ùYqúYß Ï¥¿o, F¬Nd§, ¿olTôN]m, ®YNôV
ÏQô§NVeLs ϱjR TpúYß Øû\, IÚ ÅÓ êXUôLÜm RLYpLs
×s°®YW RLYpL°u ªLlùT¬V úNL¬dLlTÓ¡u\].
Itû\ ARôWUôL TjRôiÓ UdLsùRôûL LQdùLÓl×, 2021
LQdùLÓl× Ds[Õ. BWiÓ LhPeL[ôL ùNnVlTÓm.
NPR UdLsùRôûL RLYpLû[ UhÓúU ØRp LhPj§p, ÅÓ Th¥V-Óm
ùLôi¥ÚdÏmúTôÕ, U ​​ dLsùRôûL, @pXÕ ÅhÓ LQdùLÓl©u
ùTôÚ[ôRôW ùNVpTôÓ, Lp®V±Ü T¦Ls 2020 GlWp ØRp ùNlPmTo
Utßm Lp® Utßm ©\Ytû\j R®W YûW SPjRlTÓm.
ÅhÓYN§ Utßm ÅhÓ YN§Ls 130 AiÓLÞdÏm úUXô] YWXôtû\d
úTôu\ RLYpLÞdÏ UdLsùRôûL ùLôiP BkR LQdùLÓl×
LQdùLÓl×dÏ áÓRp ®YWeLs IqùYôÚ 10 YÚPeLÞdÏm IÚ
úRûYlTÓ¡u\]. Øû\ ×s°®YWeL°u DiûUVô]
LPkR TjRôi¥p Sôh¥u ùNpYjûR ùY°lTÓjÕ¡\Õ,
Øuú]t\jûR UßAnÜ ùNnYRtÏm, BÕ 1872 ØRp ØRp UdLsùRôûL
@WNôeLj§u RtúTôûRV LQdùLÓl©-ÚkÕ ùRôPeÏ¡\Õ.
§hPeLû[ LiLô¦lTRtÏm, úR£V UdLsùRôûL T§Ü FuTÕ
F§oLôXj§tLô] §hPeLÞdÏm Bk§Vô®p YôÝm UdLs, Ï¥UdLs
UdLsùRôûL LQdùLÓl× @pXÕ BpûXVô FuTRtLô]
@¥lTûPVôÏm. RWÜjR[UôÏm, A]ôp Ï¥UdL°u
UdLsùRôûL LQdùLÓl× @§Lô¬ úR£V UߺWûUlTô[o FuTÕ
UdLsùRôûL LQdùLÓl©u úTôÕ Bk§V Ï¥UdL°u RWÜjR[UôÏm.
¡WôUYô£L°PªÚkÕ ®YWeLû[ Fu.Ao.£ (National Register of Citizens
úNL¬d¡\ôo. -NRC) ù N V p Ø û \ T § X ° j R Y o
UdLsùRôûL LQdùLÓl× L°PªÚkÕ Ï¥Ù¬ûUdLô]
UdLsùRôûL, ùTôÚ[ôRôW AR ô W j ûR ú L ô Ú ¡ \ Õ. @ pX Õ
ùNVpTôÓ, Lp®V±Ü Utßm Lp®, ¿iPLôXUôL RÓl×dLôYp. A]ôp
ÅhÓYN§ Utßm ÅhÓ YN§Ls, NPR Bp, FkR AYQjûRÙm YZeL
SLWUVUôdLp, LÚÜßRp Utßm úYi¥V @Y£VªpûX.
B\l×, Nô§ Utßm §hPªPl NPR#Bp £X ùTVoLs RY\®Pl
ThP TZeÏ¥«]o, ùUô¯, URm, Th¥ÚdLXôm, BÕ @YoL°u
BPmùTVoÜ, BVXôûU Tt±V Ï¥Ù¬ûU WjÕ ùNnVlTPôÕ,
®¬Yô] Utßm DiûUVô] Gù]²p BÕ NRC Bu ùNVpØû\
RLYpLû[ YZeÏ¡\Õ. @pX. NRC IÚ ®j§VôNUô] ùNVp.
®YNô«Ls Utßm ®YNôVj NPR LôWQUôL VôÚm Ï¥Ù¬ûUûV
ùRô¯Xô[oLs, @YoL°u Tô-]m, BZdL UôhPôoLs.
ÅPt\ ùRô¯pÕû\«p Ds[ NPR Bu úTôÕ úNL¬dLlThP
ùRô¯Xô[oL°u ùRô¯pNôo RWÜLû[ NRC Bu @¥lTûPVôL
YûLlTôÓ, YojRLm, Y¦Lm, ùRô¯p TVuTÓjR FkR §hPØm BpûX
@pXÕ úNûY Utßm ùRô¯Xô[o F] DsÕû\ @ûUfNL @§Lô¬Ls
úTôu\ûY LQd¡PlTÓ¡\Õ. ùR¬®jÕs[]o.
Tô-]m Utßm Lp®V±Ü ®¡Rm, @kR Tϧ«p ùRôPeLlThP
TX SLWeLs, ÏÓmTeLs Utßm FkRùYôÚ §hPeLÞdÏm

©lWY¬ 2020 A-Series in


  
úSW¥ TV]ô°L[ôL BÚdÏm ùNX®p úUtùLôs[Üm, úR£V
DiûUVô] Ï¥«ÚlTô[oL°u UdLsùRôûL T§úYhûP ì.3941.35
×s°®YWeLû[ @ûPVô[m LôQ úLô¥ ùNX®p úUmTÓjRÜm Il×Rp
NPR RWÜ DRÜ¡\Õ Fuß @§Lô¬Ls @°dLlThPÕ.
áß¡u\]o. Bk§V UdLsùRôûL LQdùLÓl×
FÓjÕdLôhPôL, Ï_Wôj§p IÚ Sôh¥u IhÓùUôjR UdLs
ùRô¯pÕû\ SLWj§p ¨WkRW ùRôûLûVÙm, LQdùLÓlTÕPu,
Ï¥«ÚlTô[oL°p ùTÚmTôúXôo @Nôm R®W ©\ Uô¨XeL°p Y£dÏm
Ï_Wôj§ úTÑm SToL[ôL UdL°u Fi¦dûLûV T§Ü ùNnV
BÚdLXôm, A]ôp RtúTôûRV úR£V UdLsùRôûL T§úYÓ DRÜm.
Ï¥«ÚlTô[oL°p ùTÚmTôúXôo Bk§V UdLsùRôûL LQdùLÓl×
Sôh¥u TpúYß TϧL°- FuTÕ DX¡úXúV ªLlùT¬V
ÚkÕm Bk§ úTÑm UdLû[d ¨oYôL Utßm ×s°«Vp çVô]
ù L ô i ¥ Ú d L X ô m . NPR R W Ü SPY¥dûLVôÏm. @ÓjR 10 AiÓ
@WNôeL Y¥YûUl× Utßm LÞdLô] UdLsùRôûL LQdùLÓl×
AÙxUôu úTôu\ §hPeLû[ 2021#Bp BWiÓ LhPeL[ôL
Uôt±VûUdL DRÜm. RtúTôûRV úUtùLôs[lTÓm.
×s°®YWeL°uT¥ TôWj, _kRu ÅÓL°u Th¥Vp Utßm ÅÓLs
úVôw]ô @pXÕ Ts°L°p LQdùLÓl× # 2020 GlWp ØRp
Lt©dÏm EPLm, BR]ôp ùNlPmTo YûW, Utßm UdLsùRôûL
§hPeLs ªLÜm TVàs[RôL LQdùLÓl× 2021 ©lWY¬ 9 ØRp 28
BÚdÏm. YûW.
Uj§V @ûUfNWûYd áhPj§p, 2021 # úR£V Ød¡VjÕYm YônkR BkR
Am AiÓ Bk§V UdLsùRôûL UôùTÚm T¦«p, 30 XhNm L[l
LQdùLÓlûT ì. 8754.23 úLô¥ T¦Vô[oLs CÓTÓjRlTÓYôoLs,

 A-Series in
          ©lWY¬ 2020
2021 UdLs ùRôûL LQdùLÓl×d úTôÕ ùTôÕUdLs AuûXu
Lô] קV ØuØVt£Ls YÚUôß: êXm RôeL[ôLúY ØuYkÕ
×s° ®YW úNL¬l×dÏ ØRuØû\ LQdùLÓl×dLô] ®YWeLû[
VôL ùNpúTôu ùNV-Lû[l ùR¬®jRp.
TVuTÓjÕRp. UdLsùRôûL LQdùLÓl× Utßm
UdLsùRôûL LQdùLÓl× úR£V UdLs ùRôûL T§úYÓ NôokR
LiLô¦l× Utßm úUXôiûUd LQdùLÓl©p CÓTÓúYôÚdLô]
Lô] RWÜ, UdLsùRôûL LÜWY E§Vm @YoL[Õ Ye¡d
LQdùLÓl×l T¦«p CÓTÓm LQd¡p úSW¥VôL ùNÛjRlTÓm.
@û]jÕ @§Lô¬Ls/@ÛYXo L[lT¦«p CÓTÓm 30 XhNm
LÞdÏ úRûYVô] ®YWeLû[ T¦Vô[oLÞdÏ RWUô] T«t£
TpúYß ùUô¯L°p YZeÏYRtÏ YZeLlTÓYÕPu, úR£V / UôYhP
GtTôÓ. @[®Xô] T«t£ ¨ßY]eL°u
UdLsùRôûL LQdùLÓl× T¦«u úNûYLÞm TVuTÓjRlTÓm.

BkR Fi¦dûL 2011#Bp 28 XhNUôL @û]jÕ ×s° ®YWeLÞm


BÚkRÕ. ùY°«PlTÓm.
×s° ®YW úNL¬l×dÏ ùNpúTôu @ ¥ U h P ¨o Yô L l © ¬ Ü L [ô ]
ùNV-Lû[ TVuTÓjÕYÕPu ¡WôUm / YôoÓ @[®Xô] ¸rUhP
LiLô¦l×l T¦dLôL ûUV ¨oYôL @ûUl×Ls YûW ×s°
RLÜ Iuû\ GtTÓjÕYRu ®YWeLs T¡okÕ ùLôs[lTÓm.
êXm, úUmThP RWj§Xô] SôPôÞUu\, NhPúTWûYj
UdLsùRôûL ®YWeLû[ ®ûW®p ùRôϧLû[ UßYûWVû\ ùNnYRt
ùY°«P Y¯YÏdÏm. LôL, UßYûWVû\ AûQVj§tÏ
×s° ®YWl TWYp úUmThPRôL YhPôW @[®Xô] LQdùLÓl×
BÚlTúRôÓ, IÚ ùTôjRôû] ®YWeLs YZeLlTÓm.
BVd¡]ôp, IÚ ùLôsûLûV @W£u ùLôsûLLû[ YÏdLÜm,
DÚYôdLj úRûYVô] @û]jÕ ©\ ¨oYôL @pXÕ AnÜL°u
®YWeLÞm ¡ûPdLf ùNnRp. ×s° ®YWeLû[ §WhPÜm UdLs
UdLsùRôûL LQdùLÓlûT ùRôûL LQdùLÓl× Nd§YônkR
IÚ úNûYVôL úUtùLôsYRôp, LÚ®VôL BÚdÏm. UdLsùRôûL
@ûUfNLeLÞdÏ úRûYlTÓm ×s° LQdùLÓlûT IÚ úNûYVôL
®YWeLû[ ÑjRUôLÜm Fk§WeL[ôp úUtùLôsYRu êXm, @ûUfNLeLs
T¥dLdá¥VûYVôLÜm, Uô¨X @WÑLs @pXÕ ©\RWl©]
SPY¥dûLdÏ Gt\Yôßm YZeL ÚdÏ úRûYVô] ×s° ®YWeLs,
Ø¥Ùm. BVk§WeL[ôp T¥dLjRdL
UdLsùRôûL LQdùLÓl× FuTÕ YûL«Ûm, SPY¥dûLdÏ Gt\
ùYßm ×s° ®YW úNL¬l× YûL«Ûm YZeLlTÓm.
UhÓUpX. BRu Ø¥ÜLs ùTôÕ
UdLs TVuTÓjRjRdL YûL«p ØRp Bk§V UdLsùRôûL LQdùLÓl×
ùY°«PlTÓm. 1872#Am AiÓ FÓdLlThPÕ. SôÓ ØÝYÕm
@ûUfNLeLs, Õû\Ls, Uô¨X IúW úSWj§p SPkR ØRp LQdùLÓl×
@WÑLs, AWônf£ @ûUl×Ls 1881#Am Ai¥p FÓdLlThPÕ. @ÕØRp
DhTP NmTkRlThP @û]jÕj 10 AiÓLÞdÏ IÚ Øû\ LQdùLÓl×
FÓdLlTÓ¡\Õ.
RWl©]Úm TVuTÓjÕYRtúLtT

©lWY¬ 2020 A-Series in


  
קRôL TWÜm LúWô]ô ûYWv úSôn
IÚ ×§Wô] ûYWv (Corona viruses-Cov)# úSônj ùRôt\ôÏm.
@±® V- p Ø u × @ ± V lTP ô R BkR ûYWv ùRôt\ôp Tô§dLlThP
ûYWv # º]ô®p Üaôu SL¬p YoL°u BÚUp, êfÑd Lôtß,
¾®W ÖûWÂWp úSôûV (Severe N°, WjRm êXUôL ©\ÚdÏm @kR
Acute Respiratory Syndrome) DÚYôd¡ Tô§l× TWY Yônl×s[Õ.
YÚ¡\Õ. F]úY, LúWô]ô ûYWv Tô§l×dÏs
BkR úSôVôp 150#dÏm úUtThPYoLs [ô]YoLû[ R²ûUlTÓj§
Tô§dLlThÓs[]o. @§p TX úTo £¡fûNV°dL úYiÓm. IÚúYû[
¨ûXûU BlúTôÕ LYûXd¡PUôL @kúSôndÏ D¬V £¡fûN
Ds[Õ. úUtùLôs[ôRThNj§p ¾®W ÑYôNl
úSôVô°LÞdÏ ¨úUô²Vô LônfNûX ©Wf£û]Ls GtThÓ D«¬Zl×
GtTÓjRd á¥V, קVùRôÚ ûYWv úS¬PdáÓm. LPkR £X AiÓLÞdÏ
BlúTôÕ TWYj ùRôPe¡Ùs[Õ. Øu× º]ô®pRôu LúWô]ô ûYWv
BÕ ªLÜm LYûXûV GtTÓjRd Tô§l× @§L @[®p BÚkRÕ.
á¥VRôL Ds[Õ. DXùLeÏm @lúTôÕ @kR ûYWv RôdLjÕdÏ
ÑLôRôWj Õû\ @§Lô¬Ls ªLÜm A[ô¡ ètßdLQdLôú]ôo @eÏ
®¯l×Pu BÚd¡\ôoLs. D«¬ZkR]o. BûRVÓjÕ, ¾®W
BÕ Buß YkÕ®hÓ Sôû[ UÚjÕY £¡fûNLs Utßm
úTôn®Óm YûLûVf úNokR ûYWv LiLô¦l× SPY¥dûLLs
RôdÏRXô @pXÕ ùT¬V ATjûR êXUôL @kR YûL ûYWv ùRôtß
GtTÓjÕm úSô«u @±Ï±Vô Fuß Øt±ÛUôL LhÓlTÓjRlThPÕ.
úLs® FÝkÕs[Õ. @Ru LôWQUôL UdL°ûPúV
BÕ Fu] ûYWv? ©¥j§ÚkR @fNm Ntß ®X¡«ÚkRÕ.
LúWô]ô ûYWv FuTÕ Tôíh¥ 2002#Bp º]ô®p BÚkÕ TWYj
®XeÏLs Utßm T\ûYL°p BÚkÕ ùRôPe¡V, Nôov F]lTÓm ¾®W
U²RoLÞdÏl TWÜm IÚ YûLVô] ÑYôNd úLô[ô\ôp 8,098 úTo

 A-Series in
          ©lWY¬ 2020
Tô§dLlThP§p 774 úTo D«¬Zk LúWô]ô ûYWv ùRôtßdÏ
R]o. A[ôYÕ DiÓ. @eLô¥«p úLô¯,
LúWô]ô ûYWvLs úXNô] N°«p ùY[Yôp, ØVp, Tôm× úTôu\
ùRôPe¡ UWQjûR GtTÓjÕYÕ D«¬]eLÞm ûYj§Úd¡\ôoLs.
YûW«Xô] Tô§l×Lû[ @ûYÙm ûYWvLs DÚYôL
GtTÓjRXôm. YônlûT GtTÓjÕm.
""BkRl קV ûYWv, BkR BWiÓdÏm º]ô®p Gu TWÜ¡\Õ ?
BûPlThP YûLûVf úNokRRôLj UdLsùRôûL ùSÚdLj§u
úRôuß¡\Õ. קV LúWô]ô ûYWû^ LôWQUôLÜm, BkR ûYWv Ds[
TôodÏm úTôÕ, @±Ï±Ls FqY[Ü ©Wô¦LÞPu @§L ùRôPo©p
¾®WUôL BÚd¡u\] FuTûRd BÚlTRôÛm º]ô®p @§L Tô§l×
LôQ SôeLs ®Úmסú\ôm. BÕ GtTÓ¡\Õ Fuß úTWô£¬Vo
N° úTôu\ûR®P ¾®WUô] ÜpaÜv á±]ôo.
@±Ï±VôL BÚdLXôm, LYûX @ÓjR úSônd ¡Úª º]ô®úXô
GtTÓjÕYRôL BÚdLXôm, A]ôp @pXÕ @kRl Tϧ«úXô BÚkRôp
Nôov úTôX @§ ¾®WUô]RôL VôÚm AfNoVlTPl úTôY§pûX''
BpXôUp BÚdLXôm'' Fuß Fu\ôo.
F¥uTod TpLûXdLZL úTWô£¬Vo TWÜm ®Rm
Uôod ÜpaÜv á±Ùs[ôo.
IÚ U²R¬Pm BÚkÕ Buù]ôÚ
""קV ûYWvLs FlúTôÕm YÚdÏ TWYd á¥VRôL BkR
LiÓ©¥dLlThÓd ùLôi¥Úd ûYWv BpûX FuTÕ DߧVôLj
¡u\]. LY²dLlTPôUp BÚkR ùR¬¡\Õ.
IÚ ¡Úª ùRôÏl©p BÚkÕ @ûY
ÖûWÂWpLû[ ¡Úªj ùRôtß
U²RoLÞdÏl TWÜ¡u\]. LPkR
Rôd¡]ôp, BÚUp, N° úTôu\
LôXj§p ùRôtßúSôn TW®VûR
Yt\ôÛm, @kR úSôVô°dÏ @Ú¡p
Sôm TôojRôp, @Õ LúWô]ô
ùNu\ôÛm Tô§l× GtTÓYÕ
ûYW^ôL BÚkRôp, ®XeÏLs
LYûXdϬV ùT¬V ®`Vm.
LôlTLl Tϧ«p BÚkÕ Rôu
U²RoLÞdÏ BûP«p BÕ TWYd
Yk§ÚdÏm'' Fuß Sôh¥eaôm
á¥VRôL BÚkRôp, @YoLÞdÏ
TpLûXdLZLj§p SfÑ«¬Vp
£¡fûN @°dÏm úTôÕ ùSÚe¡f
YpÛSWôL BÚdÏm úTWô£¬Vo
ùNpÛm UÚjÕYoLÞdÏ Tô§l×
ù_ô]ôRu Tôp áß¡\ôo.
GtTh¥ÚdL úYiÓm.
Nôov ¡Úª ×àÏl éû]«Pm
BÚkRúTô§Ûm, U²RoLÞdÏ
BÚkÕ U²RoLÞdÏl TW®VÕ.
BûP«p BÕ TWÜUô FuTûR
2012#Bp DÚYô¡V ùUov úSôVôp
¾oUô²dL BkR @YLôNm úTôÕUô]
2,494 úTo Tô§dLlThP§p 858 úTo
RpX Fuß £X ¨×QoLs
D«¬ZkR]o. BÕ Itû\j §ªp
áß¡u\]o.
ùLôiP IhPLeL°Pm BÚkÕ
59 úSôVô°L°p @û]YÚdÏm 2019
TW®VÕ.
¥NmTo 12 ØRp 29#Am úR§dÏs
BkR úSôVô°LÞdÏ Üaô²p
@±Ï±Ls úRôu±Ùs[].
Ds[ ùRtÏ º]ô LPp DQÜ
ùUôjR ®tTû] @eLô¥ÙPu @Ru©\Ï ×§V úSôVô°Ls VôÚdÏm
ùRôPo× BÚlTÕ LiÓ©¥dLl BkRl Tô§l× GtTP®pûX.
ThÓs[Õ. LPÛdÏf ùNpTYoLs BkRl Tô§l× GtThP úSôVô°Ls
(ÕÚYl Tϧ §ªeLXm úTôu\) Fi¦dûL @§L¬dL®pûX.

©lWY¬ 2020 A-Series in


  
Uj§V @W£u JWôiÓ §hPeLs
Uj§V LXôfNôWm úR£V @±®Vp @ÚeLôh£VL
Uj§V LXôfNôW @ûUfNLm, NûT, áÏs LûX Utßm LXôfNôWl
Sôh¥u Y[ªdL LûX, LXôfNôW ©¬ÜPu IÚ LôXj§p IÚ ØVt£
TôWmT¬VjûRl TôÕLôlTRtÏ Fu\ AuûXu LiLôh£«p
DߧéiÓs[Õ. IjÕûZdL Ø¥Ü ùNnÕs[Õ.
ULôjUô Lôk§V¥L°u 150#YÕ Bk§V ÑRk§Wl úTôWôhP
©\kR AiÓ ùLôiPôhPm, §Vô¡L°u @LWô§ûV ©WRUo
ÏmTúU[ô®p LXôfNôWd ÏmT®Zô ×Õ§p-«p úXôd LpVôi Uôod
F]lTÓm 29 Sôs ®Zô, 10#YÕ Fu\ BPj§p SûPùTt\ ¨Lrf£«p,
úR£V LXôfNôWl ùTÚ®Zô, 2019 Uôof 7#Am úR§ ùY°«hPôo.
ANôj ¶kj F]lTÓm ÑRk§W ùS ô n P ô ® p × § R ô L @ ûU d L l
Bk§V @WÑ @ûUdLlThPRu ThÓs[ Ti¥h ¾uRVôs
76#YÕ AiÓ ¨û]Ü ¨Lrf£, DTôjVôVô LhPPd LûX
ULôWôx¥Wô®p UôWdLiúPvYWo ¨ßY]jûR ©WRUo SúWk§W úUô¥
úLô®-u LûXSVªdL LhPPdLûX 2019 Uôof 9#Am úR§ ùRôPe¡
Áh×lT¦, IúW TôWRm Du]R ûYjRôo.
TôWRm ùRôPoTô] TpúYß @UoSôj Vôj§ûW ùNpÛm
¨Lrf£Ls Ds°hPûY. Y¯«p, TphPôp FuàªPj§p
©WVôdWôw ÏmTúU[ô®uúTôÕ, 29 103.7 ùULôùahv @ûXY¬ûN
Sôs LXôfNôWd ÏmT ®Zô. «p TiTûX Yôù]ô- ¥Wôuv
©WRUo ùTt\ T¬ÑL°u GXm 2 ªhPûW LXôfNôW @ûUfNLj§u
ùRôϧL[ôL SûPùTt\Õ. GXj§p úYiÓúLô°uúT¬p, ØRpØû\
¡ûPjR ùRôûL ØÝYÕm SUôª VôL RLYp I-TWl× @ûUfNLm
LeLô §hPj§tÏ @°dLlThPÕ. @ûUjÕs[Õ.
ÏmTúU[ô ϱjÕ £\l× Wô_vRôu Uô¨Xj§p Ds[
RTôpRûXûV W«púY Utßm ù_nléo SLWjûR DXLl TôWmT¬V
ùRôûXjùRôPo×j Õû\ BPeL°u Th¥V-p úNodL,
BûQVûUfNo Uú]ôw Bk§Vô ùNn§ÚkR ®iQlTm,
£uaô, 2019 ©lWY¬ 2#Am úR§ Ùù]vúLô TôWmT¬V BPeLs
ùY°«hPôo. 5 ìTôn U§l×s[ Th¥V-p úNodLlThPÕ.
£\l× RTôp Dû\Ùm ùY°«Pl NÁTj§p Bk§V ùRôp-Vp
ThPÕ. AnÜjÕû\ ULôWôxhWô Uô¨Xm,
YôWQô£«p RNôvYúUj Õû\dÏ ×lúLôYu Fu\ BPj§p SPj§V
@ÚúL, LeûL S§dLûW«p @ûUk @LrYôn®p ®RoTô Tϧ«p DúXôL
Õs[ Uôu#Uaô-p Bk§V ùRôp- LôX Ï¥«Úl×Ls BÚkRRtLô]
Vp AnÜ ¨ßY]m @ûUjÕs[, RPVeLs ¡ûPjÕs[]. BkR
Uj§V @WÑ TôÕLôl×dÏ DhThP BPj§p @LrYônÜl T¦Ls
¨û]Üf £u]j§p, ùUn¨Lo 2018, ¥NmTo ØRp 2019 Uôof YûW
@àTY @ÚeLôh£VLjûR ©WRUo SûPùTt\].
ùRôPe¡ ûYjRôo. úR£V @±®Vp Lôk§«u Sôhϱl× (1943#44)
@ÚeLôh£VL NûT, BRû] AdvKúTôoÓ TpLûXdLZL
DÚYôd¡Ùs[Õ. @fÑ ¨ßY]j§u DR®ÙPu Bk§V
úR£V AYQd LôlTL ¨ßY]j

 A-Series in
          ©lWY¬ 2020
§]ôp ULôjUô Lôk§«u 150#YÕ Ui Fu\ ùTV¬p Lôh£dÏ
©\kR AiûPùVôh¥ ùY°«Pl ûYdLlThÓs[Õ.
ThPÕ. BRtLô] ¨Lrf£ ×Õ§p- AÙx @ûUfNLm
«p úSÚ ¨û]Ü @ÚeLôh£VLm AÙx @ûUfNLm ùRôPeLlThÓ
Utßm èXLj§p SûPùTt\Õ. HkRôiÓLs AÏm ¨ûX«p,
Uj§Vl©WúRNj§p 10#YÕ úR£V Uôtß UÚjÕY Øû\ûV
LXôfNôWl ùTÚ®ZôûY, ©WRUo ©WTXlTÓjÕY§Ûm, YÛYô]
2019 @dúPôTo UôRm ùRôPe¡ Utßm @û]jûRÙm Ds[Pd¡V
ûYjRôo. UÚjÕYd LhPûUlûT DÚYôdÏm
_ô-Vu YôXôTôd úR£V ¨û] YûL«Ûm @Õ ùNVpThÓ YÚ¡\Õ.
Üf£u] §ÚjRf NhPm 2019 2019 Am AiÓ ØÝYÕm AÙx
SôPôÞUu\j§p ¨û\úY±VÕ. UÚjÕY Øû\Lû[ Ød¡V
úSRô´ ÑTôxNk§W úTôv Utßm UÚjÕY Øû\VôdL ùRôPo
@YWÕ A^ôj ¶kj KTÜw ØVt£Ls úUtùLôs[lThÓ,
BVdLm ϱjR @û]jÕ AYQe B§p ϱl©PjRdL ùYt±Ùm
LÞm WL£Vl Th¥V--ÚkÕ FhPlThÓs[Õ.
¿dLlThÓ, Bk§V úR£V AYQd AÙx ÑLôRôW úNûYLs F°§p
LôlTLj§p ùTôÕUdLs TôoûYd ¡ûPdLf ùNnRp, AÙx AWônf£
LôL ûYdLlThÓs[]. BqYôß Lû[ EdÏ®jRp, AÙx Lp®,
ûYdLlThÓs[ 304 AYQeLs/ AÙx UÚkÕLs Utßm @Õ
úLôl×L°p 303 GtL]úY úSRô´ N ô o k R @ m Ne L s , ® ¯ l × Q o Ü
YûXR[Uô] www.netajipapers.gov.in GtTÓjÕRp, AÙx UÚjÕY
FuT§p T§úYt\m ùNnVl Øû\ûV DXLUVUôdLp Utßm
ThÓs[Õ. AÙx Õû\«p RLYp
_ô-Vu YôXôTôd UiûQd ùRô¯pÖhTjûR ×ÏjÕRp A¡V
ùLôiP LXNm úR£V @ÚeLôh£ 7 ùTÚm @mNeL°p ùYt±
VLj§p D«o¿jR §Vô¡L°u LôQlThÓs[Õ.

ù_nléo @WiUû]

©lWY¬ 2020 A-Series in


  
AÙx ÑLôRôW úNûYLû[ T§lTLjÕPu BûQkÕ, ×WKTNo
úUmTÓjR, úR£V AÙx BVdLm AÙxUôu: UÚjÕY RôYWeLs &
F]lTÓm Uj§V @WÑ DR®ÙPu UÚjÕY ÏQeLs ϱjR ÏZkûR
á¥V §hPm ùNVpTÓjRlTÓ¡\Õ. LÞdLô] Lôªd ×jRLm Iußm
AÙx Õû\«u Y[of£dÏ ùY°«PlThÓs[Õ.
Uô¨X / ë²Vu ©WúRN @WÑLs AÙoúYR Utßm TôWmT¬V
úUtùLôsÞm ØVt£LÞdÏ BkR UÚjÕYj§tLô] ©mvùPd
§hPm DßÕûQ VôL Ds[Õ. TpLûXdLZLm Iuû\ Bk§Vô®p
©WRU¬u _u AúWôd¡Vô GtTÓjRÜm, ©mvùPd SôÓLs
§hPj§u¸r, AÙx UÚjÕYd Il×d ùLôiÓs[].
LôlÀhÓj §hPeLû[ úNolTRtÏ, ùTôÕUdLs ReL[Õ Y£l©Pe
19 AÙoúYR, £jRô & Ù]ô² Utßm LÞdÏ @Ú¡p Ds[ úVôLô T«t£
14 úVôLô & BVtûL Øû\ £¡fûN ûUVjûR LiP±V GÕYôL, úVôLô
ùRôÏl×Ls úR£V ÑLôRôW ùXôúLhPo Fu\ ùNpúTôu
AûQVj§tÏ T¬kÕûWdLl ùNV-Ùm ùRôPeLlThÓs[Õ.
ThÓs[Õ. ùTÚkùRô¯pLs @ûUfNLm
BUVUûX ©Wôk§Vj§u TôWmT¬V Bk§Vl ùTôÚ[ôRôWj§u Ød¡V
UÚjÕY Øû\VôL LÚRlTÓm, Nd§L°p Iu\ôL AhúPôùUôûTp
úNôYô#¬dTô Øû\ûV ÁiÓm ùRô¯p Ds[Õ. RtúTôÕ DX¡p
@±ØLlTÓjÕm SPY¥dûL«u Ds[ ùTÚmTôXô] AhúPô
Ød¡V @mNUôL, úX Tϧ«p ùUôûTp ùRô¯p Õû\«]o
úNôYô#¬dTô úR£V ¨ßY]jûR Bk§Vô®p Ds[]o.
@ûUdL @àU§ @°dLlThÓs[Õ. BWiÓ NdLW, êuß NdLW Yi¥Ls,
¨¥ AúVôd Utßm Bk§Vô®p TV¦Vo úTÚkÕLs, BXÏ WL
Ø R Ä Ó (AGNI) A ¡ V Y t ± u Y¦L YôL]eLs, ¥WdÏLs,
IjÕûZl×Pu IÚe¡ûQkR ¥WôdPoLs, L]WL Y¦L YôL]eLs
ÑLôRôW AWônf£ (SIHR) §hPm úTôu\ @û]jÕ YûLVô]
Iuß DÚYôdLlThÓs[Õ. YôL]eLÞm Sôh¥p RVô¬dLl
Bj§hPj§tÏ BWiPôiÓL°p TÓ¡u\].
ì.490 úLô¥ ùNX®PlTPÜs[Õ. RtúTôÕ Bk§Vô Lôtß UôÑ
Y[of£ Utßm SÅ]UVUôdLp ùSÚdL¥ûV F§oùLôiÓs[Õ.
§hPjÕPu BûQkÕ AÙx Lp® DX¡p ªLÜm UôÑTôÓs[ 20
Øû\ EdÏ®dLlThPÕ. AÙoúYRô, SLWeL°p 14 Bk§Vô®p Ds[].
Ù]ô², £jRô Utßm úaôªúVôT§ BkR NYôpLû[ Nk§dL @W£u
UÚjÕY Lp® ¨ûXVeL°p úNokÕ TpúYß Õû\Ls Dj§Lû[
B[¨ûX ThPlT¥l×Lû[ T¥dL, YÏjÕs[]. ©Fv#4 -ÚkÕ
úR£V Rϧ Utßm ÖûZÜj úRoÜ ©Fv#6 dÏ úSW¥VôL ùNpÛRp,
(NEET) @±ØLlTÓjRlThPÕ. L]WL YojRL YôL]eL°p
ùNuû]«p Ds[ úR£V £jR F¬ùTôÚs £dL]j§tLô]
UÚjÕY ¨ßY]m (NIS), TiûPdLôX Y¯Ls úTôu\ SPY¥dûLLs
TôWmT¬V Øû\Vô] YoUd LûXûV FÓdLlTÓ¡u\].
D«ol©jÕ @Rû] úUmTÓjÕ KúTm Bk§Vô §hPj§u @àTY
YRtLô] SPY¥dûLLû[ @¥lTûP«p, BWiPôm LhP
úUtùLôiÓs[Õ. §hPm Uj§V @ûUfNWûY
ØmûTûVf úNokR @Uo £jW LRô Il×RÛPu 2019 Uôof 8 @uß

 A-Series in
          ©lWY¬ 2020
@±®dûL ùNnVlThPÕ. TôÕLôl×, TkúRôTvÕ, ÑLôRôWm,
úThP¬Vôp BVeÏm YojRL YôL]e NêLl TôÕLôl× A¡VYtû\
LÞdÏl Toªh ùTßY§p BXdÏ YÛlTÓjÕRp, úYûXYônl×Ls
@°lTÕ ùRôPoTô] @±®dûLûV @§L¬lTûR EdLlTÓj§
Uj§V NôûX úTôdÏYWjÕ, ùRô¯p¨ßY]eL°p ÑêLUô]
ùSÓgNôûXLs @ûUfNLm ùNVpTôhûPd ùLôiÓYÚRp
ùY°«hÓs[Õ. Fàm úSôdLjÕPu ùRô¯Xô[o
ªuNôW YôL]eLÞdLô] TôLeLs SXf NhPeL°p §ÚjReLs
DtTj§ûV DsSôh¥p @§L¬dL Utßm @UXôdLj§p ùY°lTûPj
ÑeLj ¾oûYûV ¨§VûUfNLm RuûUûVd ùLôiÓYW TX
Uôt±VûUjÕs[Õ. SPY¥dûLLû[ ùRô¯Xô[o SXu
¿oY[jÕû\ @ûUfNLm Utßm úYûXYônl× @ûUfNLm
úR£V LeûLdÏÝ®u ØRXôYÕ úUtùLôiÓs[Õ.
áhPm, ©WRUo SúWk§W úUô¥ ùRô¯Xô[o SXf NhP ºo§ÚjReLs:
RûXûU«p DjRWl©WúRN Uô¨Xm E§VeLs ϱjR ùRô¯Xô[o
Lôué¬p ¥NmTo 14#Am úR§Vuß NhPm: Ïû\kRThN E§Vf NhPm
SûPùTt\Õ. 1948, E§Vm YZeLp NhPm 1936,
LeûL Utßm @Ru ¡û[ S§Lû[l úTô]v YZeLp NhPm 1965,
×]WûUjRp UôÑ RÓl× A¡VYtû\d NUE§Vf NhPm 1976 A¡VûY
LiLô¦dÏm IhÓùUôjR ùTôßl× BûQdLlThÓ, E§VeLs
BkRdÏÝÜdÏ YZeLlThPÕ. NhPm 2019 ùLôiÓYWlThPÕ.
¿oY[ úUmTôhÓj §hPm: BÕ SôPôÞUu\j§u BÚ
Bk§Vô®p Ri½ol Tt\ôdÏû\ @ûYL°Ûm ¨û\úYt\lThÓ,
Ds[ YhPeLs Utßm UôYhPe Ï¥VWÑj RûXYWôp Il×Rp
L°p ¿o úNªl×dLô] ùNVpTôÓ @°dLlThPÕ.
L°u Øuú]t\jûR úYLlTÓjR ùRô¯tNeLf NhPm 1926, ùRô¯pÕû\
Uj§V Uô¨X @WÑLs LXkRônÜ úYûXYônl× (¨ûX AûQLs)
ùNnYúR ¿oY[ úUmTôhÓj NhPm 1946, ùRô¯p RLWôßLs
§hPUôÏm. NhPm 1947 A¡VûY BûQdLlThÓ,
¿o úNªl×dLô] ¿oY[ §hPm ùRôPdLm: ùRô¯Û\ÜLs §ÚjRf NhP UúNôRô
DÚYôdLlThPÕ.
Uj§V ¿oY[jÕû\ @ûUfNo
Lú_k§W £e ù`LôYj _þûX ùRô¯Xô[o BZlÀhÓf NhPm
Iu\ôm úR§ ¿oY[ úUmTôhÓj 1923, úTß LôX TVu NhPm 1961,
§hP ØLôm ùRôPeLlTh¥ÚlTûR T¦dùLôûP YZeÏRp NhPm
@±®jRôo. ¿o úNªl×j §hPm, ¿o 1972, Øû\NôWô ùRô¯Xô[oLs
TôÕLôl×dLô] ØLôUôÏm NêLlTôÕLôl×f NhPm 2008
Ds°hP 9 NhPeLs BûQdLl
S ôÓ ØÝYÕm ª L Üm Ri½o
ThÓ, NêLl TôÕLôl× NhP
Tt\ôdÏû\ Ds[ 256
UúNôRô DÚYôdLlThPÕ. BÕ
UôYhPeLÞdÏ Uj§V @W£u
2019 ¥NmTo 11 @uß UdL[ûY«p
1,100 @ÛYXoLs L[ AnÜdLôL
RôdLp ùNnVlThPÕ.
ùNu±ÚkRûRj ùRôPokÕ Uj§V
@ûUfNo úUtùLôiP BWiPôYÕ ùRô¯tNôûXLs NhPm 1948,
TVQUôÏm. úRôhPj ùRô¯Xô[o NhPm
ùRô¯Xô[o SXu Utßm úYûXYônl× 1951, ÑWeLeLs NhPm 1952,
Lh¥PeLs Utßm ©\ LhÓUô]j
ùRô¯Xô[o IqùYôÚY¬u

©lWY¬ 2020 A-Series in


  
ùRô¯Xô[oLs (úYûXYônlûT Nôu±Rr YZeLlThÓs[Õ.
Øû\lTÓjÕRp Utßm úNûY HAo£¥£«u NûUVp ©¬ÜLû[
¨ûXûULs) NhPm 1996 Ds°hP ££¥® êXm úSW¥ I°TWl×
13 ùRô¯Xô[o NhPeLs BûQdLl ùNnÙm YN§, 18 NûUVp ©¬ÜL°-
ThÓ, T¦«Pj§p TôÕLôl×, ÚkÕ (úU 2019) 40 NûUVp ©¬ÜL[ôL
ÑLôRôWm Utßm T¦¨ûXûULs @§L¬dLlThÓs[Õ.
ϱjR ùRô¯Xô[o NhP UúNôRô Bk§V W«púY 44 YkúR TôWj
DÚYôdLlThPÕ. BÕ 23.07.2019 W«pLû[ RVô¬dL Ds[Õ.
@uß UdL[ûY«p @±ØLm Bk§V W«púY @pXôUp,
ùNnVlThPÕ. HAo£¥£#Vôp BVdLlTÓm
xWm Ñ®Rô BûQVlTdLm êXm ØRXôYÕ úR_v W«p, §p-#Xdú]ô
Uô¨XeLû[ IÚe¡ûQdÏm T¦ BûPúV BVdLlTÓ¡\Õ.
SûPùTtß YÚ¡\Õ. 2#YÕ YkúRTôWj Fdv©Wv,
2024#Am Ai¥p Sôh¥u ùUôjR ×Õ§p- # LjWô BûPúV YZdLUô]
DsSôhÓ DtTj§«u 5 ¥¬p- úNûY«p CÓTÓjRlThÓs[Õ.
Vu PôXo ùTôÚ[ôRôWm Fu\ DX¡úXúV ªL @§L @[®p
BXdÏPu Bk§Vl ùTôÚ[ôRôWm T¦Vô[oLû[ úYûXdÏ @Uoj§V
DX¡úXúV Øu]¦ ùTôÚ[ôRôW ¨ßY]eL°p Iu\ôL Bk§V
SôÓL°p Iu\ôL DÚYôL Ds[Õ. W«púY DÚùYÓjÕs[Õ.
SôÓ ØÝYÕm UôYhP DRVm DR® úXôúLô ûTXhÓLs &
NUôLmv §hPªPlThÓ 731 ùPdɵVu TR®LÞdLô]
UôYhPeL°p @ûUdL Il×Rp 64,000 BPeLÞdÏ BÕYûW
@°dLlThPÕ. BpXôR YûL«p 47.45 XhNm
Ù.úL.£uaô RûXûU«u ¸r Bk§V ®iQlTRôWoLs ®iQl©j
¬Noq Ye¡«u ¨×Qo ÏÝ Ïß, Õs[]o.
£ß Utßm SÓjRW ùRô¯pLÞdÏ 37
¨ûX 1(ØkûRV ÏÚl#¥) T¦«p
T¬kÕûWLû[ ùNn§ÚkRÕ.
Lô-VôL Ds[ 63,000 BPeLÞdÏ
W«púY Õû\ ÑUôo 1.17 úLô¥ ®iQlTRôWoLs
BÕYûW BpXôR YûL«p êXR]f úRoÜ Fݧ]o.
ùNXÜ: 2019#20 Thù_h U§lÀh¥p 13,500 B[¨ûX ùTô±Vô[o
ì.1,60,176 úLô¥ TR®LÞdÏ ÑUôo 24.75 XhNm
2030 YûW ì.50 XhNm úLô¥ ØRÄÓ ®iQlTRôWoLs ®iQl
ùNnV §hPªPlThÓ, 2019 ¨§¨ûX ©jR]o.
@±dûL«p, W«púYj Õû\ûV
LlTp Õû\
Sôh¥u Y[of£dLô] Fk§WUôL
LlTp Õû\«u IhÓùUôjR
Uôt\ Y¯YûL LôQlThÓs[Õ.
Y[of£ûV ®ûWÜTÓjÕm
TV¦Ls W«pL°u NWôN¬ úYLm
úSôd¡p, 2019#Am Ai¥p,
60% YûW @§L¬l×
TpúYß Øuú]t\LWUô] ùLôsûL
WôwRô² W«p TVQm IúW BWÜ Lû[ ×Ïj§V Uj§V @WÑ, קV
ùLôiPRôL Uôt\lThÓs[Õ ØuØVt£Lû[Ùm úUtùLôiPÕ.
TÑûU ªuUVUôdLlThP W«p
TôûR YWXôtßf £\lתdL LlTp
UßÑZt£f NhPm 2019 ¨û\úYt\l
ÑtßfãZp úUXôiûU Øû\Lû[
ThPÕ.
LûP©¥jRRtLôL, SPlTôi¥p 85
W«p ¨ûXVeLÞdÏ HFvJ:14001 LPp £lTk§LÞdLô] TúVô#ùUh¬d

 A-Series in
          ©lWY¬ 2020
Advt.
@ûPVô[ AYQm LPp £lTk§ úUô¥ Sôh¥tÏ @olT¦jÕ
LÞdÏ, @YoL[Õ ØLlT§ÜPu ûYjRôo.
á¥V TúVôùUh¬d @ûPVô[ RªrSôÓ, úLW[ô Utßm Ak§Wl
AYQm YZe¡V DX¡u ØRp ©WúRNj§p ©WjúVL T«t£ ûUVe
SôPôL Bk§Vô Uô±Ùs[Õ. LÞm @ûUdLlThÓs[]. LPpNôo
SmSôhÓ LPp £lTk§Ls, úTôdÏYWjÕdLô] Tp§\u
TpúYß SôÓLû[ LPkÕ ùNpÛm úUmTôhÓ ûUVm Iuß _YaoXôp
úTôÕ, @YoLÞdÏ, TôÕLôlTô] úSÚ Õû\ØLj§p 2019#Am AiÓ
@ûPVô[ AYQUôL BkR קV ùRôPeLlThPÕ.
AYQm §LÝm.
ØRXôYÕ ©mvùPd Õû\ØLeLs
NôLoUôXô §hPj§u BWiPôm UôSôÓ, 2019#Bp Bk§Vô®p
LhPj§p, ¾uRVôs DTôjVôVô SûPùTt\Õ.
¡Wôªu LÜNp úVô_]ô §hPm,
Õû\ØL Utßm LPpNôo T¦LÞd NôLoUôXô §hPj§p LPúXôW LlTp
Lô] §\u úUmTôhÓ §hPUôL úTôdÏYWjÕdÏ ªÏkR Ød¡VjÕYm
Uôt\l ThÓ, Ï_Wôj, ULôWôx¥Wô, @°dLlTÓYRôp, LPúXôW LlTp
úLW[ô, Ak§Wô Utßm RªrSôÓ úTôdÏYWjûR úUmTÓjR, LlTp
A¡V Uô¨XeL°p ùRôPeLl Õû\ TpúYß ØuØVt£Lû[
ThÓs[Õ. úUtùLôiÓs[Õ.
_ôodLih Uô¨Xm Wôg£«p NoYúRN LPpNôo @ûUlTô]
Nô¡lLgf TpYûL úTôdÏYWjÕ HFmJ#®u ©#©¬Ü Dßl©]WôL
Øû]VjûR ©WRUo §Ú SúWk§W Bk§Vô ÁiÓm úRokùRÓdLl
ThÓs[Õ.

©lWY¬ 2020 A-Series in


  
WôÔY §]j§u YWXôtß NôRû]
1949#Am AiÓ _]Y¬ 15. ÑRk§Wj TXm ùLôiP SôPôLj §Lr¡\Õ
§tÏl ©u, Øu× TR®«-ÚkR Bk§Vô. @ÕUhÓUu±, DX¡úXúV
©¬h¥x @W£u No KlWôu£v Nd§YônkR TûPTXj§p SôuLôYÕ
×hùN¬ ùY°úV\, Bk§Vô®u ØRp BPjûR Y¡d¡\Õ. RûWlTûP,
ùXl¥]uh ù_]WXôL úL.Fm.L¬VlTô LPtTûP, ®Uô]lTûP F] Sôh¥u
TR®úVt\ Sôs. ©¬h¥`ô¬PªÚkÕ ØlTûPLÞdÏm RtúTôÕ RûXûUj
Bk§VôÜdÏ @§LôW Uôt\m R[T§VôL ¨VªdLlThÓs[ôo, ©©u
ùNnVlThPÕ, YWXôtßf £\l× ªdL WôYj. WôÔY @¦YÏl×Ls, WôÔY
RÚQm. @R]ôp, ReLs TûPTXjûRd NôLNeLs, @Ru ùRô¯pÖhTeLs,
Lôi©dÏm YûL«p SôùPeÏm NôRû]Ls F] BkR AiÓ, 72#YÕ
Ds[ WôÔY ÅWo Lû [ j § Wh ¥ , YÚP WôÔY §]jûR ®U¬ûNVôLd
IqùYôÚ YÚPØm _]Y¬ 15 @uß ùLôiPô¥VÕ Uj§V @WÑ. ùPp-«p,
WôÔY @¦YÏlûT SPjÕ¡\Õ ùTôÕ @§Lô¬ LUôi¥e (General officer
Bk§Vô. úUÛm, Sôh¥tLôLj §VôLm commanding) RûXûU«p Y¯SPjRl
ùNnR @û]jÕ WôÔY ÅWoLû[Ùm ThP @¦YÏlûT WôÔYj RûXYo
¨û]ÜáÚm ùTôÚhÓ BkR Sôs YQdLm ùNÛj§V©u AnÜùNnRôo.
@àN¬dLlTÓ¡\Õ. 1.4 ªp-Vu Ut\ BWiÓ úNûYj RûXYoLÞm
WôÔY ÅWoLû[dùLôiÓ, DX¡úXúV (service chiefs) IqùYôÚ AiÓmúTôX
BWiPôYÕ ªLlùT¬V WôÔY BkR AiÓm @¦YÏl©p

 A-Series in
          ©lWY¬ 2020
LXkÕùLôiÓ YQdLm ùNÛj§]o. Ï U ô ú Y ô u ù W ´ ù U u h (Kumaon
ØlTûPL°u RûXûUj R[T§VôL regiment) , ¡ùWú]¥VoLs (Grenadiers)
¥NmTo 31, 2019#Am úR§Vuß Utßm TôWôãh ùW´ùUuh(Parachute
ùTôßlúTt\ ù_]Wp ©©u WôYjÕm regiment) A¡V @¦LÞm @¦YÏl©p
WôÔY §] @¦YÏl©p LXkÕùLôiÓ, TeúLt\].
BkR AiÓ YQdLm ùNÛj§]ôo. BÕYûW, ØlTûPL°p RûXûUj
Ï¥VWÑjRûXYo WômSôj úLô®kj, R[T§L[ôL Ds[YoL°p IÚYo
©WRUo úUô¥, TôÕLôl×jÕû\ RûXYWôL BÚlTRtÏl T§XôL,
@ûUfNo WôwSôj £e, Øu]ôs ©WRUo TôÕLôl×j R[T§L°u RûXûUj
UuúUôLu £e ØRXôú]ôo BkR R[T§VôL IÚYo ¨VªdLlTÓYôo
®Zô®p LXkÕùLôiP]o. Fußm, @YúW ØlTûPj R[T§Ls
15 ÅWoLs, Õ¦fNÛdLô] ÏÝÜdÏm RûXYWôLf ùNVpTÓYôo
®ÚÕL[ôp @XeL¬dLlThPôoLs. Fußm @WÑ Ø¥ùYÓjRÕ. @kRl
18 ThPô-VuLÞdÏ (battalions) @XÏ TR®dÏ ¨VªdLlTÓm DVW§Lô¬,
úUtúLôsLs (citations) ¡ûPjR]. @ûY, SôuÏ vPôoLÞdLô] Rϧ YônkR
@¦YÏl©p Lôh£lTÓjRlThP]. ù_]Wp AL BÚlTôo.
úLlPu Pô²Vô ù`o¡p FuTYo úR£V TôÕLôl×j ùRôPo×ûPV
ØRp ùTi WôÔY ÅWWôL @û]jÕ @û]jÕ AÙReLs ùLôsØRp,
ÅWoLs ùLôiP WôÔY @¦YÏlûT T«t£ Dj§Ls, ØlTûPL°u áhÓ
Øu²uß Y¯SPj§fùNu\ôo. SPY¥dûL, úLk§W SPY¥dûLLs
ùNuû]«p Ds[ @§Lô¬, T«t£ Ds°hPûY ϱjÕm, @WûN SPjÕm
@LôPª«-ÚkÕ Uôof 2017#Bp Ah£Vô[oLÞdÏ ØlTûP úNûY
¨VªdLlThP ù`o¡p ªu]Ô Ï±jR AúXôNû]Lû[ YZeÏYÕ,
Utßm RLYp ùRôPo× ThPRô¬ Fuß Ød¡V Il×RpLû[l ùTtßjRÚYÕ
RLYpLs ùR¬®d¡u\]. úRNj§tÏ A¡V T¦Lû[ IÚe¡ûQdÏm WôÔY
úNûY ùNnÕ YÚm R]Õ ÏÓmTj§u AúXôNLo T¦ûVÙm קV RûXûUj
TôWmToVjûR Pô²Vô SôuLôYÕ R[T§ AtßYôo.
RûXØû\VôLf ÑUkÕ ùNp¡\ôo. TôÕLôl× @ûUfN¬u úSW¥
LPkR AiÓ, WôÔY §] úUtTôoûY«p ùNVpTÓm קV
@¦YÏl©p, ØRpØû\VôL 144 Ai RûXûUj R[T§, Bk§V TôÕLôl×jÕû\
WôÔY ÅWoLs ùLôiP ÏÝûY AÙR ùLôsØRp LÜu£p Utßm
Y¯SPj§V ØRp ùTi @§Lô¬VôL TôÕLôl×j §hPªPp ÏÝ®u
YWXôtû\ DÚYôd¡]ôo, úLlPu Dßl©]WôL BÚlTôo. BkRl TR®dLôL
Tôq]ô Lvç¬. BkR AiÓ, Pô²Vô ©©u WôYj ¨VªdLlThÓs[ôo.
ù`o¡p úYù\ôÚ YWXôtû\l ÑRk§W Bk§Vô®p ØRpØû\VôL,
TûPj§Úd¡\ôo. ØlTûPL°u RûXûUj R[T§«u
WôÔY §] @¦YÏl©p 18 Øu²ûX«p, BkR Ai¥u WôÔY
BVk§WUVUôdLlThP LôXôhTûP, §]m @àN¬dLlThÓs[Õ.
"Ràx' ÕlTôd¡ @ûUl× Utßm Bk§V RûWlTûP
WôÔY ®Uô]l TôÕLôl× A¡VYt±u Bk§V RûWlTûP 6 LhPû[VLeL°u
LôXôhTûP úTôo YôL]m ©.Fm.© #2 úL ¸r BVeÏ¡\Õ. IqùYôÚ LhPû[VLØm
(BMP # 2K) × Õ ù P p - W ô w T ô j § p ùXl¥]uh ùN]Wp RWj§Ûs[ LhPû[
Lôh£lTÓjRThP]. Lôolv AKl @ÛYXL¬u ¸r BVeÏ¡\Õ. FpXô LhPû[
£d]p (corps of signals) , ºd¡V ûXh VLeLs ×Õ ùPp-«p Ds[ RûXûUVLj§p
L ô X ô h T û P (Sikh light infantry), BûQdLlThÓs[].

©lWY¬ 2020 A-Series in


  
Rªr Y[of£j Õû\ ®ÚÕLs

RªrùUô¯ Y[of£dÏm, Rªrl TiTôhÓ §ÚYsÞYo §ÚSôs ®Zô®û] @WÑ


úUuûUdÏm ùRôiPôt±V RªZ±OoLs, ®ZôYôLf ºÚm £\l×UôL SPj§ YÚ¡\Õ.
L®OoLs, NêL¿§dÏ DûZj§hP @kR YûL«p §ÚdÏ\s ùS±TWl×m
ùT¬úVôoLs úTôu\YoL°p £XûWj úRoÜ ùTÚkRûLVô[o IÚYÚdÏ 1986 ØRp
ùNnÕ Nôuú\ôoLs ùTV¬p AiÓúRôßm AiÓúRôßm §ÚYsÞYo §ÚSôs ®Zô®p
RªrSôÓ @W£u Rªr Y[of£j Õû\«u ¸r Bq®ÚÕ YZeLlThÓ YÚ¡\Õ.
BVeÏm Rªr Y[of£ BVdLLm êXUôL 2020#Am AiÓ @nVu §ÚYsÞYo
®ÚÕLs YZeLlThÓ YÚ¡u\]. ®ÚÕ ùTt\Yo:
Bq®ÚÕLs §ÚYsÞYo §ÚSô°p YZeLl  S. ¨j§Vô]kR TôW§
TÓ¡\Õ.
TôúYkRo TôW§RôNu ®ÚÕ
 Bq®ÚÕLû[ ùTßúYôo RXô 1 XhNm
§Wô®P, TÏjR±Ü ùLôsûLLû[ R]Õ
ìTôndLô] LôúNôûX, IÚ NYWu ReLl TRdLm
TôPpL°p ûYjÕ ×Wh£LWUô] TôPp
Utßm ®ÚÕdLô] RϧÙûWf Nôu±Rr
L[ôL Rª¯p BVt±, Rªr ùUô¯dÏ ªLf
A¡VûY YZeLlTÓ¡u\].
£\kR úNûYVôt±V UôùTÚm L®Oo
 @kRYûL«p 2020#Am Ai¥tLô]
×Wh£dL®, TôúYkRo F] @ûZdLlTÓm
@nVu §ÚYsÞYo ®ÚÕ Øû]Yo
TôW§RôNu ùTVWôp ®ÚÕ YZeLl
S, ¨j§Vô]kR TôW§; RkûR ùT¬Vôo ®ÚÕ
TÓ¡\Õ. Bq®ÚÕ RªZL @WNôp 1978#Am
ùNg£ WôUfNk§Wu; @iQp @múTjLo
AiÓ ØRp YZeLlThÓ YÚ¡\Õ.
®ÚÕ Øû]Yo L, @ÚfÑ]u; úTW±Oo
AiÓúRôßm Rªrd L®Oo IÚYûWj
@iQô ®ÚÕ Øû]Yo úLô. NUWNm;
úRokùRÓjÕl TôúYkRo TôW§RôNu ®ÚÕ
ùTÚkRûXYo LôUWôNo ®ÚÕ Øû]Yo Uô.Ñ.
YZeLlTÓ¡\Õ.
U§YôQu ; ULôL® TôW§Vôo ®ÚÕ Øû]Yo
T. £YWô´; TôúYkRo ®ÚÕ R. úR²ûN 2019#Am AiÓ TôúYkRo TôW§RôNu
ùNpXlTô; RªrjùRu\p §Ú.®.L ®ÚÕ ®ÚûR ùTt\Yo:
Øû]Yo úN. ÑkRWWôNu; ØjRªrd LôYXo  R. úR²ûN ùNpXlTô
¡.A.ùT. ®ÑYSôRm ®ÚÕ Øû]Yo UÚjÕYo RªrjùRu\p §Ú.®.L. ®ÚÕ
U¦úULûX LiQu A¡úVôÚdÏ RªZL RªrjùRu\p §Ú.®.L ®ÚÕ YZeÏm
ØRpYo YZe¡]ôo. §hPm 1979#Am Ai¥p úRôtß®dLlThPÕ.
@nVu §ÚYsÞYo ®ÚÕ Bj§hPj§uT¥ AiÓúRôßm £\kR Rªr
RªrSôÓ @WÑ, DXLl ùTôÕUû\Vôm FÝjRô[o IÚYûWj úRoÜ ùNnÕ ®ÚÕ
§ÚdÏ\s ùS±TWl×m ùTÚúSôd¡p YZeLlThÓ YÚ¡\Õ.
AiÓúRôßm ûRj §eLs ØRp Sô°p 2019#Am AiÓ RªrjùRu\p §Ú.®.L.

 A-Series in
          ©lWY¬ 2020
®ÚÕ ùTt\Yo: Rª¯Ûm, Ae¡Xj§Ûm ªLf£\kR úTfNô[o.
 Øû]Yo úN. ÑkRWWôNu §Wô®P Øuú]t\ LZLjûR 1949#Bp ÕYd¡,
ULôL® TôW§Vôo ®ÚÕ 1967#1969 YûW RªZLj§u ØRXûUfNWôL
úRN ®ÓRûX, ùTi ®ÓRûXdLôL TôPp T¦Vôt±VYo. BYWÕ ùTVWôp úTW±Oo
BVt±V TôW§Vôo ùTV¬p TôW§Vôo ®ÚÕ @iQô ®ÚÕ F] RªZL @WÑ AiÓúRôßm
1979#Am AiÓ ØRp YZeÏm §hPm YZe¡ YÚ¡\Õ.
úRôtß®dLlThPÕ. TôW§Vôo TûPl×Lû[ 2019#Am AiÓ úTW±Oo @iQô
AnÜ ùNnúYôo, TôW§Vôo Tt±j §\]ônÜ ®ÚÕ ùTt\Yo:
ùNnúYôo, TôW§Vôo ×LûZ TWl×m YûL«p  Øû]Yo úLô. NUWNm
L®ûR# DûWSûP èp ùRôiÓ ùNnúYôo
@iQp @múTjLo ®ÚÕ
A¡úVôo Bq®Ú§û] ùT\j Rϧ
Bk§V @W£VXûUl×f NôN]jûR
TûPjRYoL[ôYo.
DÚYôd¡VYo, Sôh¥u ØRXôYÕ NhP
2019 #Am AiÓ ULôL® TôW§Vôo ®ÚÕ
@ûUfNo, ¾iPôûU I¯V úTôWô¥VYo,
ùTt\Yo:
RôrjRlThP ©tTÓjRlThP UdLÞdLô]
 Øû]Yo T. £YWô´
BPIÕd¸h¥tLôL úTôWô¥VYo TôTôNôúLl
ØjRªrd LôYXo ¡.A.ùT. ®ÑYSôRm
Fu\ûZdLlTÓm ÀmWôq Wôm´ @múTjLo.
®ÚÕ NêL¿§dLôL TôÓTÓm Nôuú\ôo IÚYÚdÏ
RªrSôÓ @WÑ ØjRªrd LôYXo BYWÕ ùTV¬p @iQp @múTjLo ®ÚÕ
¡.A.ùT. ®ÑYSôRm ®Ú§û] 2000#Am YZeLlTÓ¡\Õ.
Ai¥p úRôtß®jRÕ. Bq®ÚÕ 2019 #Am AiÓ @iQp @múTjLo
AiÓúRôßm £\kR RªZ±Oo IÚYÚdÏ ®ÚÕ ùTt\Yo:
YZeLlTÓ¡\Õ.
 Øû]Yo L, @ÚfÑ]u
2019#Am AiÓ ØjRªrd LôYXo
ùTÚkRûXYo LôUWô_o ®ÚÕ
¡.A.ùT. ®ÑYSôRm ®ÚÕ ùTt\Yo:
RªZLj§u ØRpYWôL TR®Y¡jÕ,
 UÚjÕYo U¦úULûX LiQu U§V DQÜj §hPjûR @ØpTÓj§,
RkûR ùT¬Vôo ®ÚÕ F°ûUdÏm úSoûUdÏm ùTVo ùTt\Yo
RªZL UdL°ûPúV TW®Ùs[ êPSm©dûL LôUWôNo. ùTÚkRûXYo, ¡eúUdLo
Lû[ @Lt\Üm, Nô§ I¯dL, ùTi @¥ûUj F] ×LZlThP BYWÕ ùTV¬p ùTÚk
R]jûR F§odL, §Wô®P B]jûR úUmTÓjR RûXYo LôUWô_o ®ÚÕ YZeLlTÓ¡\Õ
TÏjR±Ü ©WfNôWm ùNnRYo RªZo RkûR 2019 #Am AiÓ ùTÚkRûXYo LôUWô_o
ùT¬Vôo C.ùY.WôUNôª. BYo ùRôPoTô] ®ÚÕ ùTtú\ôo:
AWônf£ Utßm ùT¬Vôo LÚjÕLû[  Øû]Yo Uô.Ñ. U§YôQu
TWl×Y§p T¦Vôt±V IÚYÚdÏ
§ÚYsÞYo §ÚSôs Utßm RªZL
AiÓúRôßm ùT¬Vôo ®ÚÕ YZeLl @W£u Nôo©p ®ÚÕLs YZeÏm ®Zô
TÓ¡\Õ. ùNuû] §ÚYp-dúL¦«p Ds[ LûXYôQo
2019#Am AiÓ RkûR ùT¬Vôo ®ÚûR @We¡p SûPùTt\Õ.
ùTt\Yo: BkR ®Zô®p ØRp# @ûUfNo FPlTô¥
 ùNg£ WôUfNk§Wu TZ²Nôª LXkÕùLôiÓ ®ÚÕLû[ YZe¡
úTW±Oo @iQô ®ÚÕ LÜW®jRôo. úUÛm ®ÚÕ ùTt\YoLÞdÏ
ì.1 XhNjÕdLô] LôúNôûX Utßm ReLlTRdLj
úTW±Oo @iQô TÏjR±Ü, §Wô®P ûRÙm @Yo @¦®jRôo.
LÚjÕLû[ R]Õ FÝjRôt\p, úTfÑ, SôPL
TûPl×Ls úTôu\Yt\ôp TWl©VYo.

©lWY¬ 2020 A-Series in


  
@Pp _p §hPm
Ï_Wôj, a¬Vô]ô, LoSôPLô, Uj§Vl @Pp _p §hPjRôp ¸rLiP ®û[ÜLs
©WúRNm, ULôWôx¥Wô, Wô_vRôu Utßm GtTÓm:
DjRWl©WúRNm A¡V 7 Uô¨XeL°p, ¨XjR¥ ¿o LiLô¦l× @ûUl×Lû[
Øuà¬ûU @¥lTûP«p NØRôVl úUmTÓj§, TpúYß UhPeL°Ûm ¨XjR¥
TeL°l×Pu á¥V ¨XjR¥ ¿oúUXôiûU ¿o úNL¬l× ×s° ®YWeLû[ úUmTÓj§,
úUmTôhÓj §hPjûR ùNVpTÓjR Bj§hPm @Ytû\ T¡okÕ ùLôsYúRôÓ, TÏlTônÜ
YûL ùNn¡\Õ. ùNnÕ TWYXôdÏYÕ.
BkR §hPjûR ùNVpTÓjÕYRu êXm úUmThP ×s° ®YWeL°u @¥lTûP«p,
BmUô¨XeL°p Ds[ 78 UôYhPe úUmThP Utßm SûPØû\ Nôj§VUô]
LÞdÏ DhThP ÑUôo 8,350 ¡WôUl Ri½o §hPeLû[ U§lÀÓ ùNnYúRôÓ
TgNôVjÕLs TV]ûPÙm F] F§oTôodLl NØRôVm NôokR Ri½o TôÕLôl×j §hPe
TÓ¡\Õ. Lû[ TgNôVjÕ @[®p úUtùLôsYÕ.
TgNôVjÕLs Yô«XôL ¨XjR¥ ¿o Uj§V # Uô¨X @WÑL[ôp RtúTôÕ
úUXôiûUûV úUmTÓj§, úRûYdúLt\ ùNVpTÓjRlThÓ YÚm TpúYß §hPeLû[
úUXôiûUûV @¥lTûP úSôdLUôL ùLôiÓ IÚe¡ûQjÕ, ¿¥jR ¨XjR¥ ¿o úUXôiûUd
TZdL YZdL Uôt\jûR GtTÓjR @Pp _p Lô] ¨§ûV ¨VôVUôLÜm, §\mTPÜm
§hPm ùLôiÓ YWlThÓs[Õ. TVuTÓjÕRp.
ùUôjR IÕd¸Pô] ì.6,000 úLô¥«p, Öi½o TôN]m, Uôtßl T«o NôÏT¥,
50% DXL Ye¡ LP]ôL ùT\lThÓ, ªuNôW Eh¥ ©¬l× úTôu\ úRûYdúLt\
@Rû] Uj§V @WÑ §Úl©f ùNÛjÕm. SPY¥dûLLs êXm, ¡ûPdLdá¥V ¨XjR¥
Fg£V 50% ùRôûL Uj§V @W£u ¿o Y[jûR,§\mTP TVuTÓjÕRp.
YZdLUô] Thù_h IÕd¸PôL YZeLlTÓm.
Dsðo UdL°u TeL°l×Pu _p ËYu
DXL Ye¡d LPu Utßm Uj§V ¨§ÙR®
BVdLj§tLô] ¨§ ARôWm ¨WkRWUôL
ØÝYÕm Uô¨XeLÞdÏ ¨§ÙR®VôL
¡ûPdLf ùNnRp.
YZeLlTÓm.
®YNô«L°u YÚUô]jûR BWh¥lTôd
LiLô¦l× @ûUl×Ls, §\u DÚYôdLm,
ÏYÕ Fu\ BXdûL @ûPV DRÜm.
Ri½o DTúVô¡lúTôo NeLeLû[
YÛlTÓjÕYÕ DhTP, Uô¨XeL°p ¿¥jR TeL°l×Pu á¥V ¨XjR¥ ¿o úUXôiûUj
¨XjR¥ ¿o úUXôiûUdÏ @ûUl× §hPjûR EdÏ®dÏm.
çVô] GtTôÓLû[ YÛlTÓjÕYRtLô] úUmThP Øû\«p Ri½ûW £dL]UôL
@ûUl× Ã§VôL YÛlTÓjÕRp Utßm §\u TVuTÓjÕYRtLô], §hPjûR ùTÚU[®p
DÚYôdLj ùRôÏl×. ùNVpTÓj§, T«o NôÏT¥ Øû\Lû[
×s° ®YWl TWYp, Ri½o TôÕLôl× úUmTÓjÕRp.
§hPeLû[ RVô¬jRp, RtúTôÕ ùNVpTôh¥p ¨XjR¥ ¿o ARôWjûR £dL]UôLÜm,
Ds[ §hPeLû[ IÚe¡ûQjÕ N¬NUUôLÜm TVuTÓjÕYûR EdÏ®lTúRôÓ,
úRûYdúLt\ úUXôiûU SûPØû\ NØRôV @[®p TZdL YZdL Uôt\jûRÙm
Lû[ ©uTtßYRu êXm úUXôiûU GtTÓjÕm.
SûPØû\Lû[ ùNVpTÓjÕYÕ Ds°hP Uû\kR Øu]ôs ©WRUo YôwTô«u ©\kR
úUmTÓjRlThP ¨XjR¥ ¿o úUXôiûU §]Uô] ¥NmTo 25#Am úR§ ×ÕùPp-«p
SûPØû\L°p NôRû] TûPdL GÕYôL, SûPùTt\ ®Zô®p ©WRUo SúWk§W úUô¥
Uô¨XeLÞdÏ EdLj ùRôûL YZeÏY @Pp _p §hPjûR ùRôPe¡ ûYjRôo.
RtLô] EdLjùRôûL ùRôÏl×.

 A-Series in
          ©lWY¬ 2020
¿eLÞm NkRôRôWo ALXôúU!
NkRô ®YWm
LôXm ùUôjR ©W§Ls ®ûX ¥d ùNnVÜm
IÚ YÚPm 12 ì. 325/#
êuß YÚPm 36 ì. 865/#
HkÕ YÚPm 60 ì. 1260/#

 TQm @àl×m Øû\:  NkRô ®YWm @àl×m Øû\:


 Sd¸Wu Tl°úL`uv ùTV¬p @gNXL  ùTVo, ØÝûUVô] ØLY¬, ùRôûXúT£
êXm Money Order (M.O.) @àlTXôm.
Fi Utßm TQm ùNÛj§V ®YWjûR
 A/c. - Bp TQm ùNÛjR ¸rdLôÔm ùRôûXúT£
Fi¦p ùRôPo×ùLôs[Üm. ϱl©hÓ santha@nakkheeran.in B#ùU«p
 " Sd¸Wu Tl°úL`uv' Fu\ ùTV¬p @àlTXôm.
"ùNuû]' ùNÛjRRdL Demand Draft [DD]  @pXÕ ¸rdLôÔm álTû]j ùR°YôL éoj§
FÓjÕ @àlTXôm ùNnÕ NkRô ùRôûLÙPu @àl© ûYdLÜm.
 "Sd¸Wu Tl°úL`uv' Fu\ ùTV¬p  Ød¡VUôL NkRô TQm ùNÛj§VYoLs
LôúNôûX @àlTXôm. ùY°ëo LôúNôûXdÏ ¸rdLôÔm ùRôûXúT£«p ùRôPo× ùLôiÓ,
40 ìTôn áÓRXôL ùNÛjR úYiÓm. "@hTôo' NkRô @àl©VûR DߧlTÓj§d ùLôs[Üm.
LôúNôûXdÏ úYi¥V§pûX.

¸Ýs[ álTû]j ùR°YôL éoj§ ùNnÕ NkRô ùRôûLÙPu @àl© ûYdLÜm.

ùTVo : ................................................................................................. ©\kR úR§ : ..............................


®XôNm : ................................................................................................
................................................................................................
................................................................................................
ùRôûXúT£ (@ÛYXLm) : ................................................... (ÅÓ) ...................................................
Please find enclosed (DD/MO. No. ............................ Date: ................... favouring
NAKKHEERAN PUBLICATIONS.
Date : .......................... Signature : .........................................

Sd¸Wu Tl°úL`uv,
SP#76, 14#YÕ ùRÚ, @mTjço ùRô¯túThûP, ùNuû] # 600 058
ùRôûXúT£: 044# 43993014 / 43993000
Advt

B#ùU«p: santha@nakkheeran.in

©lWY¬ 2020 A-Series in


  
SPl× ¨LrÜLs

RªZLm
®ûWkÕ, RTôpLû[ ùP-Y¬
ùNnÙm Uô¨XeL°p, ØR-Pj§p,
ùPp- # 46 U¦ úSWm, Ds[Õ.
BWiPôªPj§p Ds[ RªZLm, 66
U¦ úSWj§tÏs ùP-Y¬ ùNn¡\Õ.
£j§WúU¯ LpùYhÓ LiÓ©¥l×
SôÓ ØÝYÕm, IÚ RTôp, NWôN¬VôL,
§ÚlTjço UôYhPm LpSôoNôm
82 U¦ úSWj§tÏs D¬VYÚdÏ
Th¥«p 1 3 . # A m è t \ ô i û P
ùP-Y¬ ùNnVlTÓYRôL, RTôp
úNokR £j§WúU¯ LpùYhÓ
Õû\ @±®jÕs[Õ. (_]Y¬ 16)
LiÓ©¥dLlThÓs[Õ.
ùNuû] HH¥ AWônf£Vô[oLs
úU¯ FuTÕ DZÜd LXlûT @pXÕ
NôRû]
Go Fuß ùTôÚsTÓm. £j§WúU¯
Fu\ôp @Z¡V LXlûTûV ϱdÏm LPp¿¬p BÚkÕ ûahW_u
ùNôpXôÏm. F¬ùTôÚs RVô¬dÏm SÅ] LÚ®ûV
Y¦LdÏÝ áhPm ùTÚmTôÛm ùNuû] H.H.¥. AWônf£Vô[oLs
£j§WúU¯ ùT¬VSôhPôo NûT LiÓ©¥jÕs[]o.
Fuß @ûZdLlTÓm. (_]Y¬ 10) BkR LÚ®«p FkR Ri½¬p
RªZLm BWiPôªPm BÚkÕm ûahW_û] RVô¬d
RTôp úNûY«u, LPkRôiÓ NôRû]l LXôm.
Th¥VûX, RTôp Õû\, NÁTj§p YojRL ç«p ûahW_u RVô¬dL
ùY°«hPÕ. 1000 ¥¡¬ ùNp£Vv ùYlTØm,
B§p, RTôp ùP-Y¬ ùNnV FÓjÕd 25 ×s°Ls @ÝjRØm úRûYl
ùLôsÞm NWôN¬ úSWm Ïû\kÕs[ TÓm.
RôLÜm, RôURªu± ®ûW®úXúV A]ôp BkR קV LÚ® @û\«u
ùP-Y¬ ùNnVlTÓYRôLÜm ùYlTj§Ûm, IÚ ×s° @ÝjRj
ùR¬®dLlThÓs[Õ. §Ûm BVeLdá¥VÕ. (_]Y¬ 17)
BVp ®ÚÕ 2019
L]Pô YZeLlTPÜs[Õ.
Rªr BXd¡Vj BYo FݧV ØRp SôYXô]
ú R ô h P m LôYpúLôhPm èÛdÏ 2011#Am
@ûUl× YZe¡ AiÓ Nô¡jV @LôPª ®ÚÕ
YÚm NoYúRN YZeLlThPÕ.
® Ú Õ L ° p Ñ.ùYeLúPNu RªrSôh¥Ûm,
ØRuûUVô]Õ ùY°SôÓL°Ûm úUûPL°p
BVp ®ÚÕ. Rª¯u úUuûUûVl TWl©
2019#Am AiÓdLô] Bq®ÚÕ YÚ¡\ôo. Rª¯u ùRôuûU Tt±Ùm,
Rªr BXd¡V DX¡p ¾®WUôL ¸Z¥ AnÜRWÜL°u RôdLm
BVe¡ YÚm L®OÚm, FÝj Tt±Ùm ùRôPokÕ ®¯l×QoÜf
Rô[Úm, @W£VpYô§ÙUô] ùNôtùTô¯ÜLs Atß¡\ôo.
Ñ . ù Y e L ú P N à d Ï (_]Y¬ 6)

 A-Series in
          ©lWY¬ 2020
úRNm
©]ôLô GÜLûQ úNôRû]
TôÕLôl× AWônf£ Utßm
úUmTôhÓ ¨ßY]Uô] ¥Ao¥J#
Yôp DÚYôdLlThP ©]ôLô BÕ SÅ] DNA ®YWdϱl× LÚ®Ls
GÜLûQ, I¥`ô Uô¨Xm Nk§léo Utßm DTLWQeLÞPu ®[eÏm.
LPtTϧ«p Ds[ IÚe¡ûQkR
BkR ûUVj§p Tô-Vp YuùLôÓûU
úNôRû] R[j§-ÚkÕ ùYt±LWUôL
©¬Ü Utßm TÓùLôûX ©¬Ü,
T¬úNô§dLlThPÕ.
RkûRY¯ ©¬Ü, U²R @ûPVô[ ©¬Ü
ÀWe¡ ùTôÚjRlThP BkR ©]ôLô Utßm ûUhúPôLôih¬Vp DNA
GÜLûQ, F§¬L°u TϧdÏs 75 ©¬ÜdLô] YN§Lû[ YZeÏ¡\Õ.
¡úXôÁhPo ùRôûXÜ YûW TônkÕ (¥NmTo 23)
ùNuß RôdLd á¥V §\u ùTt\ @Pp ¨XjR¥ ¿o §hPm
RôÏm.
¨XjR¥ ¿o úUXôiûUdLôL
©]ôLô, Fm. úL # II Wô d ù Lh ¥ u DÚYôdLlThP @Pp é_p úVôw]ô
EÓÚYp §\u, LhÓlTôÓ Utßm Fuß ùTV¬PlThÓs[ @Pp
Y¯LôhÓ SûPØû\Lû[ ¨XjR¥ ¿o §hPjûR ùPp-«p
IÚe¡ûQjÕ, Õp-VUô] RôdÏm ©WRUo SúWk§W úUô¥ ùRôPe¡
§\û] úUmTÓj§ BkR GÜLûQ ûYjRôo.
RVô¬dLlThÓs[Õ.
ì.6,000 úLô¥ U§l©Xô] @Pp
BkR GÜLûQ«u EÓÚYp ¨XjR¥ ¿o §hPm, 2024#Am
§\u, HAoFuFvFv DR®ÙPu AiÓdÏs IqùYôÚ ÅhÓdÏm
Y¥YûUdLlThÓs[Õ. (¥NmTo 20) ÏZôn êXm Ri½o ¡ûPlTûR
קV DNA TÏlTônÜ ûUVm Dߧ ùNnÙm.
a¬Vô]ô Uô¨Xm Ni¥L¬p ¨oTVô ¨XjR¥ ¿o úUXôiûUdLôL
¨§ §hPj§u ¸r ¨ßYlThP DÚYôdLlThÓs[ BkR §hPm
קV DNA TÏlTônÜ ûUVm §\kÕ ØRpLhPUôL Ï_Wôj, a¬Vô]ô,
ûYdLlThPÕ. Wô_vRôu, LoSôPLm, Uj§V ©WúRNm

@§®ûWÜ Yôu RôdÏRp GÜLûQ


F§¬ SôhÓ úTôo ®Uô]eLs,
ùa-LôlPoLs Ds°hPYtû\j
RûW«-ÚkÕ Rôd¡ @¯dLYpX
QRSAM @ § ® û W Ü G Ü L û Q
ùYt±LWUôL úNôRû] ùNnVl
ThPÕ.
BkR GÜLûQ«u úNôRû] I¥Nô
Uô¨Xm TôXôão UôYhPj§u FXdhúWô Al¥dLp ¥Wôd¡e
Nk§lé¬p Ds[ úNôRû] ûUVj§p LÚ®Ls êXm LiLô¦dLlThP]
SûPùTt\Õ. DRDO êXm ØÝYÕm
BkR QRSAM GÜLûQ, ®i¦p Bk§Vô®úXúV RVô¬dLlThP
GYlThP Utù\ôÚ GÜLûQûV, BkR GÜLûQ, 360 ¥¡¬ úLôQj§p
Õp-VUôL BûPU±jÕ Rôd¡ ÑZuß LiLô¦l× T¦Lû[
@¯jRÕ. úUtùLôsÞm §\u ùLôiPûY.
BkR úNôRû] úWPôo Utßm (¥NmTo 24)

©lWY¬ 2020 A-Series in


  
DX¡u ªL DVWUô] W«p TôXm
(C©s PY¬u DVWm 324 ÁhPo
L[ôÏm.) 1.3 ¡.Á., ¿[m ùLôiP
BkR TôXj§u DVWm 359 ÁhPo
AÏm.
RtúTôÕ º]ô®Ûs[ 275 Á.
DVWm ùLôiP µTôn W«púY
TôXúU DX¡u ªL DVWUô]
W«púY TôXm Fàm ùTÚûUûV
ùTtßs[Õ.
F]úY LôxÁ¬u BkR TôXm Lh¥
LôxÁ¬p Ds[ ùN]ôl S§«u Ø¥dLlThPôp, BÕúY DX¡u
ÏßdúL DX¡u ªL DVWUô] DVWUô] W«púY TôXm Fu\
W«púY TôXm LhÓm T¦Ls ùTÚûUûV Rh¥f ùNpÛm.
SûPùTtß YÚ¡u\]. LôxÁ¬p LhPlThÓ YÚm W«púY
ì.1110 úLô¥ ùNX®p LhPlTP TôXlT¦Ls 2021#m AiÓ
Ds[ BkR TôXm Tôãp Ds[ ¥NmTÚdÏs Ø¥dL BXdÏ
C©s PYûW ®PÜm DVWUô]Õ ¨oQ«dLlThÓs[Õ. (_]Y¬ 10)
A¡V Uô¨XeL°p ùNVpTÓjRlTP DX¡úXúV 2#YÕ DVWUô] £ûXVô]
Ds[Õ. (¥NmTo 26) NoRôo YpXTôn TúP-u £ûXûV
קV WôÔY ®YLôWeLs Õû\ @mUô¨X ØRpYo ®_n ìTô²
Uj§V TôÕLôl× @ûUfNLj§u ¸r §\kÕ ûYjRôo.
קRôL WôÔY ®YLôWeLs Õû\ 70,000 ¡úXô¡Wôm FûPÙs[ Utßm
Fu\ Õû\ DÚYôdLlThÓs[Õ. 50 @¥ DVWØs[ BkR ùYiLXf
BkR Õû\ ØlTûP RûXûU R[T§ £ûXVô]Õ ì.3.25 úLô¥ ùNX®p
©©u WôYj RûXûU«p ùNVpTÓm. LhPlThÓs[Õ.
WôÔYm, LPtTûP ®Uô]lTûP DX¡u ªL DVWUô] £ûXûV
A¡V 3 TûPLs NôokR T¦Lû[ Y¥YûUjR £t©Vô] Wôm Yg£
WôÔY ®YLôWeLs Õû\ LY²dÏm. ÑRo, DX¡u 2#YÕ DVWUô] BkR
ØlTûPLÞdLô] AÙReLs, £ûXûVÙm Y¥YûUjÕs[ôo.
R[YôP ùLôsØRpLû[Ùm (_]Y¬ 5)
NhP®§Ls, SûPØû\L°uT¥ ´Nôh#30 ùNVtûLdúLôs
LY²dÏm. BvúWô®u ´Nôh#30 ùNVtûLd
B² TôÕLôl× @ûUfNLm, 5 Õû\ úLôs, ©ùWgÑ LVô]ô®p Ds[
Lû[ ùLôiPRôL BÚdÏm. @ûY LqWq GÜR[j§p BÚkÕ §hP
TôÕLôl× Õû\, WôÔY ®YLôWeLs ªhPlT¥ ®i¦p ùNÛjRlThPÕ.
Õû\, WôÔY DtTj§ Õû\, WôÔY ùRu @ùU¬dLô®p Ds[ ©ùWgÑ
AWônf£ Utßm úUmTôhÓÕû\, LVô]ô®p BÚkÕ @§L FûPûV
JnÜùTt\ TûPÅWoLs SXu AÏm. ÑUkÕ ùNpÛm G¬Vôu#5 WôdùLh
(_]Y¬ 1) êXm GYlThPÕ.
2#YÕ DVWUô] NoRôo TúPp £ûX 2020#Am Ai¥p BvúWô ùNÛj§V
Ï_Wôj Uô¨Xj§u @LURôTôj ØRp ùNVtûLdúLôs BÕYôÏm.
SL¬p ûYxQúR® ûUVj§tÏ BRu FûP 3,357 ¡úXô AÏm.
@Ú¡Ûs[ NoRôoRm Y[ôLj§p (_]Y¬ 17)

 A-Series in
          ©lWY¬ 2020
º]ô®u µ´Vu#20 ùNVtûLdúLôs
º]ô R]Õ ªLlùT¬V ùNVtûLdúLôs
Rôe¡ ®iLX]ô] Xôe Uôof#5 H
NoYúRNm
ùLôiÓ µ´Vu#20 Fàm @§SÅ]
RLYp ùRôPo× ùNVtûLdúLôû[ UÚjÕYUû], Ts° F] @û]jÕ
ùYufNôe ®iùY° GÜR[j YûLVô] LhPPeLÞm UWeLs
§-ÚkÕ ùYt±LWUôL G®VÕ. ãrk§ÚdÏm.
BkR ®iLX]ô]Õ º]ô®u ªLÜm @RôYÕ Åh¥u TôpL², ùUôhûP
FûPÙs[ Utßm ªLÜm úUmThP Uô¥ F] FpXô BPeL°Ûm
RLYp ùRôPo× ùNVtûLdúLô[ôÏm. UWeLû[ ûYlTôoLs. 100 YûLVô]
BkR ®iLX]ô]Õ CZ#5 Fußm 10 XhNm UWeLs BkSLWjûR
@ûZdLlTÓ¡u\Õ. @XeL¬dLl úTô¡u\].
BÕ @§LThNUôL 25 Pu BkR UWeLs YÚPjÕdÏ 10 A«Wm
FûPdùLôiP ùNVtûLdúLôû[ Pu LôoTu ûP Adû^ûP D±g£
×®«u Rôr ÑtßYhP TôûRdÏm 14 900 Pu Ad£_û] DtTj§ ùNnÙm.
Pu FûP ùLôiP ùNVtûLdúLôû[ (_]Y¬ 3)
×®Nôo Ij§ûNÜ ÑtßlTôûRdÏm @YôuLôoÓ ûaToNô²d GÜLûQ
ùLôiÓ ùNpÛm §\u ùLôiPÕ. DX¡u FkR AÙReL[ôÛm RÓjÕ
(¥NmTo 31) ¨ßjR Ø¥VôR @YôuLôoÓ
DX¡u ØRp LôhÓ SLWm ûaToNô²d GÜLûQ ùYt±LWUôL
DX¡u ØRp LôhÓ SLWm ùRuº]ô úNôRû] ùNnVlThÓs[RôL WxVô
®p Ds[ ÏYôeµ UûXlTϧL°p ùR¬®jÕs[Õ.
@ûUkÕs[ ûXùN[ SLWj§p BkR SÅ] GÜLûQ U¦dÏ
@ûUdLlTPÜs[Õ. 33,000 ¡úXôÁhPo çWm úYLj§p
175 ùadúPo TWlT[®p TWkÕ ùNpXdá¥V §\u ùTt\Õ.
®¬kÕs[ BkR SLWj§u LhPûUl× BkR úNôRû]«u úTôÕ
Ø¥kÕ®hPôp ÑUôo 30 A«Wm úTo ûaToNô²d GÜLûQ I-«u
BeúL Y£dLXôm. úYLjûR ®P 27 UPeÏ úYLUôL
ÅÓ, úaôhPp, @ÛYXLm, ùNu\Õ. (_]Y¬ 8)

DX¡u ªLf£±V I° DQ¬


DPÛdÏs ÏZôûV ùNÛj§, @Ru
êXm @ßûY £¡fûN ùNnYRtÏ
Õp-VUô] úLUWôdLs úRûY.
@kR úLUWôdL°p BÚdÏm I°
DQ¬Ls Rôu DP-u Dßl×Lû[,
UÚjÕYoLs Õp-VUôL TôodL
DRÜ¡u\].
BúUw ùNuNôo F]lTÓm I°
DQ¬Ls @Poj§VôL BÚkRôp RVô¬jÕs[Õ.
Rôu, TPd Lôh£Ls Õp-VUôL BRu J®6948 Fu\ LÚ®, ¡u]v
BÚdÏm. DXL NôRû] ×jRLj§p, DX¡u
BRtLôL Jm² ®`u, DX¡úXúV ªLf £±V I° DQ¬VôL BPm
ªLf £±V I° DQ¬ûV ©¥jÕs[Õ. (¥NmTo 21)

©lWY¬ 2020 A-Series in


  
ùNVtûL ®¯j§ûW êXm Li TôoûY
@±®Vp#ùRô¯pÖhTm LiL°u ®¯j§ûW«p I°
@ÓdÏLs Tô§dLlThÓ Tô§
@[®p Li TôoûY BZl×Ls
LiQô¥ £p-p Tp GtTÓ¡u\].
@ùU¬dLô®Ûs[ J¬Lu BûR N¬ ùNnÙm An®p
TpLûXdLZL ®gOô²Ls, @ùU¬dLô®u vPôuúTôoÓ
ØRuØRXôL Tp §ÑdLû[ LiQô¥ TpLûXdLZLjûR úNokR
£pÛdÏs T§jÕ NôRû] ׬k ®gOô²Ls CÓTh¥ÚkR]o.
Õs[]o. RtúTôÕ @YoLs ùNVtûL
BkR £pÛdÏs Öi ¡ÚªLû[ ®¯j§ûWûV DÚYôd¡ @Ru êXm
ùNÛj§, Tp ùNôjûRVôYRtÏ, BZkR Li TôoûYûV ùT\ Ø¥Ùm
Tp-u úUúX Ds[ F]ôUp FlT¥, F] LiÓ©¥jÕs[]o.
Gu Y¯®Ó¡\Õ FuTûR AWôV ùNVtûL ®¯j§ûW«p Ds[ ªL
Ø¥Ùm. £±V ªu LPj§Ls GtL]úY
@úRúTôX Tp-u AúWôd¡Vm BÚdÏm ®¯j§ûW«p BÚdÏm
ùLÓm ©\ Y¯Lû[Ùm LiáPôL ùNpL°p ùNVpTôhûP DÚYôd¡
UÚjÕYoL[ôp úNô§jÕ @±V Li TôoûY GtTP ùNn¡\Õ.
Ø¥Ùm. קV Øû\«p ªuNôWm DtTj§
Buàm £X AiÓL°p, IÚ ªuNôWj§û] DtTj§ ùNnYRtÏ
úSôVô°«u TtL°p £±R[Ü TX Øû\Ls LôQlTÓ¡u\
Uô§¬ûV FÓjÕ @YWÕ TpÛdÏ úTô§Ûm BVtûL Øû\«p
Fu] Uô§¬Vô] £¡fûN ãZÛdÏ Tô§l©u± ªuNôWj§û]
RWXôm Fuß ¾oUô²dL Ø¥Ùm DtTj§ ùNnYRtLô] Y¯Øû\Lû[
F], J¬Lu TpLûXdLZL AWônf£Vô[oLs AnÜ ùNnÕ
AWônf£Vô[oLs ùR¬®j YÚ¡u\]o.
Õs[]o. Bk¨ûX«p RtúTôÕ ¿¬-

ùLôjRôLd LôndÏm RdLô°


ùLô¥ @pXÕ Gß ùLô¥VôLúYô
BpXôUp, BkRj RdLô°, ×Ro ùN¥
úTôX Ïß¡V BPj§p Y[od¡\Õ.
§WôhûN úTôX ùLôjÕd ùLôjRôLd
Lônd¡\Õ.
úLôph v©¬e AnYô[oLs,
RdLô°f ùN¥«u RiÓLs,
Lôm×Ls, LôndÏm §\u A¡VYtû\
¾oUô²dÏm UWTÔdL°p §ÚjRm
¨ëVôod¡Ûs[ úLôph v©¬e ùNn R R u ê X m B k R ® k ûR j
AnYLjûR úNokR ®gOô²Ls RdLô° DÚYô¡Ùs[Õ.
ØRpØû\VôL SLol×\ SôtTúR SôhL°p ®û[kÕ L²
úY[ôiûUj RdLô° Iuû\ RÚm BkR ùLôjÕj RdLô°«u TRm,
DÚYôd¡Ùs[]o. ÑûY, ¨\m Fuß FpXôúU RWUô]
YZdLUô] ùN¥VôLúYô, TPo RôL BÚlTÕ Ï±l©PjRdLÕ.

 A-Series in
          ©lWY¬ 2020
Uôov 2020 Eo§
SôNô, ùNqYôn úLôÞdÏ @àlT
Üs[ Uôov 2020 Eo§ûV
@iûU«p ùYt±LWUôL
úNô§jÕl TôojÕs[Õ.
L-úTôo²Vô Uô¨Xj§Ûs[
ù_h Dk§ AnYL UiPTj§tÏs,
BkR Eo§ Rô]ôLúY 10 U¦ úSWm
úUÓ, Ts[m, RûPLû[ LPkÕ
TV¦jÕdLôh¥, ®gOô²Lû[
U¡r®jÕs[Õ.
BRtÏ Øu SôNô @àl©V, DR®ÙPu IÚ ùNqYôn Sô°p
¡ë¬Vô£h¥, AlTofã²h¥ (24 U¦ 37 ¨ªPeLs) Uôov 2020
úTôu\ Eo§Lû[®P BÕ Eo§Vôp 200 ÁhPo ùRôûXÜ
TuUPeÏ §\u YônkRÕ. úUÛm, TV¦dÏm.
BkR Eo§dÏ Rôú]ôh¥j Uôov 2020 Eo§«p Ds[ áÓRp
§\u DiÓ. Õp-VØs[ YiQl TPd LÚ®,
B§p ùTôÚjRlThÓs[ ªL éªdÏ ùNqYô«u RûW, Tôû\Ls,
Nd§YônkR L¦² £p-u UûXLû[ TPm©¥jÕ @àl×m.

ÚkÕ ªuNôWj§û] DtTj§ ùR¬®dLlThÓs[Õ.


ùNnVdá¥V TRôojRm Iuß BkR ¡WLeLs éªûVl úTôXúY
LiÓ©¥dLlThÓs[Õ. BRu êXm RhTùYlT¨ûXÙm, 骫u
Åh¥u £X úRûYLÞdÏ úTô§V @[ûYÙm ùLôiÓs[]. úUÛm 219
ªuNôWj§û] DtTj§ ùNnV Ø¥Ùm ¡WLeLû[ D«¬]eLs YôZjRdL
F] ùR¬®dLlThÓs[Õ. Th¥V-p SôNô ®gOô²Ls
R®W úUX§L ªuNôWj§û] úNojÕs[]o.
@V--ÚdÏm ÅÓLÞdÏm úUÛm éªûVl úTôXj
T¡okR°dL Ø¥Ùm. RhTùYhT¨ûXÙm, Ri½ÚdLô]
Ïû\kR[Ü ¿¬û]l TVuTÓj§ Nôj§VdáßLÞm BkR ¡WLeL°p
ªuNôj§û] DtTj§ ùNnÙm t Fàm Ds[RôL SôNô ®gOô²Ls
BmØû\«û] LÜLôj§«Ûs[ ùR¬®jÕs[]o.
HH¥ ¨ßY]j§u AWônf£Vô[oLs Ïs[ ®iÁu §WsLs
LiÓ©¥jÕs[]o @ùU¬dLô®u ¨ë ùUd£úLô®p
ã¬Vû] Ñt±YÚm Ds[ Yôù]ô- ùRôûXúSôd¡ûV
éªûV úTôu\ ¡WLeLs TVuTÓj§ £X Ïs[ ®iÁu
SôNô®u ùLl[o YôuùY°j §WsL°p 13 ùTÚm LÚkÕû[Lû[
ùRôûXúSôd¡ êXm ®gOô²Ls AWônf£Vô[oLs LiÓ©¥j
BÕYûW 4,034 ¡WLeLû[d Õs[]o.
LiP±kÕs[]o. B§p éª úTôuú\ BkR ®iÁu §WsLs 骫-
DÚYm Utßm RhTùYlT¨ûX ÚkÕ TX I°VôiÓLs ùRôûX®p
ùLôiP ÑUôo 50 ¡WLeLs YûW @ûUkÕs[].
BÚlTRôL ùR¬®jR]o.
B§p £X ®iÁuL°p LÚkÕû[Ls
@§p 10 ¡WLeLs éªûVl
@Ru ûUVj§p BpXôUp Ds[].
úTôXúY ã¬Vû] Ñt±YÚYRôLÜm

©lWY¬ 2020 A-Series in


  
LÚkÕû[Ls ùLôiP ®iÁuL°u AWônkÕ YÚ¡u\]o.
DVo#ùR°Üj§\u TPeLs Bk¨ûX«p, ùNqYô«u Y°UiPXj
ªLlùT¬V Y¬ûN AnYLjûR §p LôQlTÓm ¿o LQd¡Pl
TVuTÓj§ FÓdLlThÓs[]. ThPûR ®P úYLUôL Uû\kÕ
ShNj§W úUôRp ¡[l©V @ûX YÚYRôL AWônf£Vô[oLs
BWiÓ ¨ëhWôu ShNj§WeLs ùR¬®jÕs[]o.
úUô§ Iu\ô]úTôÕ, LôXùY°«p ùNqYôn ¡WLj§u Col× ®ûN
GtThP ×®Âol× @ûXLû[, Ïû\YôL BÚdÏm LôWQj
@iûU«p ®gOô²Ls T§Ü §]ôúXúV ûahW_u Utßm
ùNnÕs[]o. Ad£_u êXdáßLÞd
@ùU¬dLô®p Ds[, -®evPu ¡ûPúVVô] ©ûQl× F°§p
-úLô AWônf£ ¨ûXVm, BkR رkÕ®ÓYRôL @YoLs
×®Âol× @ûXLû[ LPkR GlWp ùR¬®jÕs[]o.
2019#Bp T§Ü ùNnÕs[Õ. Y°UiPXj§p @§L ùN±îhPl
骫-ÚkÕ, 52 úLô¥ ªp-Vu ThP ¨ûX«p ¿o LôQlTÓYRôp,
I° AiÓLs ùRôûX®p BkR £X TÚYeL°p ¿o BZl× @§LUôL
úUôRÛm BûQl×m ¨LrkÕs[Õ. BÚlTRôLÜm AWônf£Vô[oLs
BûQkÕ DÚYô¡V קV ¨ëhWôu ϱl©hÓs[]o.
ShNj§Wm ã¬Vû]®P, 3.4 UPeÏ @ùU¬dL AnYô[oLs LiÓ©¥l×
¨û\ ùLôiPÕ F] ®gOô²Ls LPpYôr D«¬]Uô] LQYôn
LQd¡hÓs[]o. ÁuLÞdÏ U²RoLû[ úTôuß
ùNqYôn ¡WLm ¿o BZl× ØlT¬UôQ TPeLû[ Tôoj
ã¬Vd ÏÓmTj§p Ds[ úLôsLs R±Ùm Ti× Ds[RôL @ùU¬dL
ϱjÕm, ÕûQdúLôsLs AnYô[oLs LiP±kÕs[]o.
ϱjÕm, TX SôÓLû[f úNokR LQYôn ÁuLÞdÏ ØlT¬UôQ
AWônf£Vô[oLs ¾®WUôL LiQô¥Lû[ @¦®jÕ IÚ

DX¡úXúV ªLlùT¬V é
®Vp ®gOô²Ls ùR¬®j
Õs[]o.WlúX£Vô @o]ôp¥
édÏm ùN¥Ls IhÓi¦ RôYW
YûLûV úNokRûY AÏm.
BkR ùN¥LÞdÏ úYoLs BûXLs
FÕÜm ¡ûPVôÕ. Utù\ôÚ
RôYWj§p Y[Úm BkR ùN¥L°p
UXoLs UXokÕ ùY°úV YÚm.
@lúTôÕ Rôu, @ûY Utù\ôÚ
BkúRôú]£Vô®u úUtÏ ÑUjWô RôYWj§p IhÓi¦VôL BÚlTúR
¾®p DX¡úXúV ªLlùT¬V é ùR¬VYÚm F] ®gOô²Ls
UXokÕs[Õ. 4 @¥ @LXj§tÏ ùR¬®jÕs[]o.
©WUôiP úRôt\j§p Ds[ WlúX£Vô @o]ôp¥ DÚYj§p
BkR é®u ùTVo WlúX£Vô ùT¬RôL BÚkRôÛm, @kR é®p
@o]ôp¥. BÚkÕ @Ý¡V Bû\f£«u
DX¡p BÕYûW éjR UXoL°p ÕoSôt\m ÅÑm. F]úY BkR UXo
BÕúY ªLlùT¬VÕ Fuß RôYW ©QUXo Fußm @ûZdLlTÓ¡\Õ.

 A-Series in
          ©lWY¬ 2020
DÚÏm BUVj§p Y[Úm ùN¥Ls
ùTôÕYôL, LPp UhPj§p BÚkÕ,
6,000 ÁhPo DVWj§tÏ úUp
ùN¥, ùLô¥Ls Y[WôÕ FuTÕ
RôYW®Vp DiûU.
A]ôp, @ùU¬dL SôNô®u,
úXihNôh ùNVtûLdúLôsLs,
1993 ØRp, 2018 YûW FÓjR ×®l
TPeLû[ AWônkR ®gOô²Ls
AfN¬VUûPkR]o.
@kR ×ûLlTPeL°p ùR¬Ùm
T²l TPXm DÚ¡, S§VôL JÓYÕ
BUVUûX TϧL°p, T¥lT¥VôL
@§L¬j§Úd¡\Õ.
T² DÚ¡ YÚYûRÙm, @kR
BR]ôp, ã¬V I° TÓm BPeL°p,
BPeL°p RôYWeLs Øû[dLj
5,000 ØRp, 5,500 @¥L°p, @Po
ÕYe¡«ÚlTûRÙm, ©¬hPû]f
T²d Ï°Wôp RôYWeLs Y[WôUp
úNokR FdùNhPo TpLûXdLZL
BÚkR]. A]ôp, BlúTôÕ
®gOô²Ls LiP±kÕs[]o.
@kR DVWj§p ùYlT¨ûX
TÚY¨ûX UôßTôhPôp,
Uô± RôYWeLs Y[W Gt\Rô¡
BUVUûXûVl úTôoj§«ÚkR
«Úd¡\Õ.

LôùQô°ûV TôodL ûYjR]o. úLô¯ ØhûP«-ÚkÕ BuÑ-u


@kR LôùQô° @Yt±tÏ Av§úW-Vô Utßm _lTô²V
©¥jRUô] B\ôp ÁuLs ¿kÕYÕ ®gOô²Ls @û\ ùYlTj§p
úTôuß DÚYôdLlTh¥ÚkRÕ. BWiÓ SôhLÞdÏ úUp RôdÏl
§ûW«p ùR¬kR B\ôpLû[ DiûU ©¥dÏm BuÑ-u §WYjûR
F] ¨û]jÕ LQYôn ÁuLs DÚYôd¡Ùs[]o.
@Ytû\ RôdL ùRôPe¡]. B\ôp úUÛm, @jRûLV BuÑ-û] úLô¯
L°u BVdLj§túLtT ReL[Õ ØhûP«-ÚkúR RVô¬dL Ø¥Ùm
RôdÏRp Øû\ûV Uôt±dùLôiP]. FuTûRÙm @YoLs LiP±kÕ
@RôYÕ @Yt±u ùYqúYß Ds[]o.
ûLL[ôp RôdLj ùRôPe¡]. ùUpúTô²Ûs[ ×ú[ô¬
LQYôn ÁuLÞdÏ úLUWô YûL Buv¥¥ëh Utßm JNôLô
LiLs Ds[], @ûY Lôo²Vô, TpLûXdLZL ®gOô²Ls,
ùXuv, LÚ®¯ Utßm ®¯j§ûW ØhûP«-ÚkÕ RVô¬jR BuÑ-u,
A¡VYtû\d ùLôiÓs[]. Aß SôhLs YûW ùLh¥ RhPôUp
@ûYLs v¼¬úVôl£û^l RôdÏl ©¥d¡\Õ.
(BÚ®¯ BûVúLôQd Lôh£)
¡û[úLô BuÑ-u Fuß ùTV¬Pl
TVuTÓjÕYRtLô] §\û]d
ThÓs[ BkR UÚkRôp, TX úLô¥
ùLôiÓs[].
ìTôn ®WVm R®odLlTÓm.
@RôYÕ @ûYL[ôp çWjûR
@±V Ø¥Ùm, Gù]²p @ûYL°u úUÛm, ¾®W NodLûW úSôVô°LÞdÏ,
êû[ BÚ LiL°-ÚkÕm YÚm Tm× @ûPjÕd ùLôs[ôUp, @§L
NªdûOLÞdÏ BûP«Xô] SôhLs BuÑ-û] TVuTÓjRÜm
úYßTôÓLû[ ®[dÏ¡\Õ. Ø¥Ùm.

©lWY¬ 2020 A-Series in


  
LQd¡p, ùPôúUôhúPôLôûY
®û[VôhÓLs Årj§ Nôm©Vu ThPm ùYu\ôo.
ùTiLs Itû\Vo ©¬Ü Bߧ
AhPj§p F¡l§u a]ô WURôu,
¡[l DXL úLôlûT LôpTkÕ Be¡Xôk§u í£ûV Årj§
LjRôo RûXSLo úRôLô®p ¡[l Nôm©Vu ThPm ùYu\ôo. (¥N. 22)
@¦LÞd¡ûPúVVô] ©Tô úav¥ev NoYúRN ùNv
DXL úLôlûT LôpTkÕ ùRôPo 95#YÕ úav¥ev NoYúRN ùNv
SûPùTt\Õ. úTôh¥ Be¡Xôk§p SûPùTt\Õ.
BRu Bߧ AhPj§p Be¡XôkûR 9 ÑtßLs ùLôiP BkRj ùRôP¬p
úNokR -Yoép @¦Ùm, ©úW£-u Bk§Vô®u ©.UúLx Nk§Wu LûP£
©[ªeúLô @¦Ùm úUô§]. Ñt±p NLSôhûPf úNokR vPô²ûV
TWTWlTôL SPkR AhPj§u Ø¥®p F§ojÕ ®û[Vô¥]ôo.
-Yoép @¦, 1#0 Fu\ úLôp BkR AhPjûR 33#YÕ SLojR-u
LQd¡p ©[ªeúLô @¦ûV Årj§ úTôÕ ¥Wô ùNnRôo UúLx
ØRpØû\VôL Nôm©Vu ThPm Nk§Wu. BRu êXm 9 ÑtßL°u
ùYu\Õ. (¥NmTo 21) Ø¥®p 7.5 ×s°Ls ùTt\ UúLx
Bk§Vu åo vÏYôx Nk§Wu Nôm©Vu ThPm ùYu\ôo.
Bk§Vu åo vÏYôx ùRôPo (_]Y¬ 7)
Bk§Vô®u TpúYß SLWeL°p úT¥Fm ¥20 ùRôPo
SûPùTt\Õ. Bk§Vô®p ÑtßlTVQm
B§p ØmûT«p SûPùTt\ úUtùLôiP BXeûL ¡¬dùLh
ùRôP¬p TpúYß SôÓLû[ úNokR @¦ êuß úTôh¥Ls ùLôiP ¥20
48 ÅWo, ÅWôeLû]Ls TeúLt\]o. ùRôP¬p TeúLtß ®û[Vô¥VÕ.
BRu AiLs Itû\Vo ©¬Ü Bߧ B§p Bk§V @¦ 2#0 Fu\ LôQd¡p
AhPj§p Bk§Vô®u a¬kRo ùYt± ùTtß ùRôPûW ûLlTt±VÕ.
Tôp £e NôkÕÜm, _lTô²u BkR ùRôP¬u SôVL]ôL Bk§V
ùPôúUôhúPôLôÜm úUô§]o. úYLlTkÕ ÅfNô[o Sq¾l
TWTWlTôL SPkR AhPj§u ûN² úRoÜ ùNnVlThPôo.
Ø¥®p a¬kRo 3#0 Fu\ ùNh (_]Y¬ 10)
DXL úW©h ùNv ùRôPo
ùLôú]Ú am©, º ] ô ® u Ä
¥eËûV Årj§ Nôm©Vu ThPm
ùYu\ôo.
BRu êXm 2017#dÏ ©u BjùRôP¬p
ReLm ùYu\ Bk§Vo Fàm £\
lûT ùTt\ôo.

2017#Bp Bk§Vô®u ®vYSôRu


A]kj Nôm©Vu ThPm ùYu±
WxV RûXSLo UôvúLô®p DXL ÚkRôo.
úW©h ùNv Nôm©Vuµl ùRôPo AiLs ©¬®p SôoúY«u Uôd]v
SûPùTt\Õ. LôopNu Nôm©Vu ThPm
B§p ùTiLs ©¬®p Bk§Vô®u ùYu\ôo. (¥NmTo 29)

 A-Series in
          ©lWY¬ 2020
AdXôkÕ ¡[ô£d ùPu²v
¨ë³Xôk§u AdXôk§p ùTiLÞd
Lô] AdXôkÕ ¡[ô£d ùPu²v
ùRôPo SûPùTt\Õ.
BRu Bߧ AhPj§p @ùU¬dLô
®u ùNÃ]ô ®p-Vmv, NL SôhÓ
ÅWôeLû] ù_£Lô ùTÏXôûY
F§oùLôiÓ ®û[Vô¥]ôo.
TWTWlTôL SPkR AhPj§u
Ø¥®p ùNÃ]ô úSo ùNhL°p 2017#Am AiÓ Av§úW-V
ù_£LôûY Årj§ Nôm©Vu ThPm JT²p ThPm ùYu\ ©\Ï
ùYu\ôo. ùNÃ]ô ùYu\ ØRp ThPm
BÕYôÏm. (_]Y¬ 12)

vTô²x ãlTo úLôlûT LôpTkÕ BkR Th¥V-p BWiPôªPj§p


vùT«u Sôh¥u LôpTkÕ ¡[l º]ôÜm, êu\ôªPj§p
@¦LÞdLô], ãlTo úLôlûT @ùU¬dLôÜm Ds[]. (_]Y¬ 15)
ùRôP¬u Bߧ úTôh¥ Nܧ úaôToh NoYúRN ùPu²v
@úW©Vô®p SûPùTt\Õ. Av§úW-Vô®u LôuùTWô SL¬p
Bߧ úTôh¥«p úUô§V ¬Vp úaôToh NoYúRN ùPu²v ùRôPo
Uôh¬h # @jùX¥úLô Uôh¬h SûPùTt\Õ.
@¦Ls áÓRp úSWj§Ûm úLôp BRu ùTiLs BWhûPVo ©¬Ü
@¥dL®pûX. Bߧ AhPj§p Nô²Vô ªoNô#
ùT]ôp¥ ãh @Üh¥p 4#1 Fu\ Sô¥Vô ú_ô¥, º]ô®u Nôe
úLôp LQd¡p @jùX¥úLôûY `þYôn, ùTe `þYôn ú_ô¥ûV
Årj§ ¬Vp Uôh¬h Nôm©Vu úSoùNhL°p Årj§ Nôm©Vu
A]Õ. (_]Y¬ 13) ThPjûR ûLlTt±VÕ.
G¥© ùPu²v ùRôPo 2 AiÓ BûPùY°dÏl ©\Ï
Av§úW-Vô®p @¦LÞd¡ûPúV L[ª\e¡V Nô²Vô ªoNô, Rôu
Vô] G¥© úLôlûT ùPu²v TeúLt\ ØRp ùRôP¬úXúV
ùRôPo SûPùTt\Õ. Nôm©Vu úLôlûTûV ùYußs[Õ
£h²«p SPkR Bߧ úTôh¥«p ϱl©PjRdLÕ. (_]Y¬ 19)
ùNo©Vô, vùT«u @¦Ls úUô§]. Bk§Vô#Av§úW-Vô IÚSôs ùRôPo
B§p ùNo©V @¦, 2#1 Fu\ LQd¡p Bk§Vô®p ÑtßlTVQm
vùT«u @¦ûV Årj§ Nôm©Vu úUtùLôiP Av§úW-V ¡¬dùLh
ThPm ùYu\Õ. (_]Y¬ 14) @¦ 3 úTôh¥Ls ùLôiP IÚSôs
Bk§Vô ØR-Pm ùRôP¬p TeúLtß ®û[Vô¥VÕ.
NoYúRN ÕlTôd¡ ÑÓRp áhPûUl× ØmûT«p SPkR ØRp úTôh¥«p
2019#Am AiÓdLô] RWY¬ûN Av§úW-Vô ùYt± ùTt\Õ.
Th¥VûX ùY°«hÓs[Õ. @ûR ùRôPokÕ WôwúLôh¥p SPkR
B§p NoYúRN @[®p 21 ReLeLs, 2#YÕ Utßm ùTeLðÚ®p SPkR
6 ùYs°Ls Utßm 3 ©Wôuv 3#YÕ úTôh¥L°p Bk§Vô ùYt±
úLôlûTLÞPu Bk§V @¦ ùTtß 2#1 Fu\ LQd¡p ùRôPûW
ØR-PjûR ùTtßs[Õ. ûLlTt±VÕ. (_]Y¬ 19)

©lWY¬ 2020 A-Series in


  
@ùU¬dL PôXo U§l×Pu
ùTôÚ[ôRôWm DX¡u HkRôYÕ ªLlùT¬V
ùTôÚ[ôRôWUôL Ds[Õ.
ØRp SôuÏ BPeL°p @ùU¬dLô,
BXYN @§úYL BûQV BûQl× º] ô , _ l T ô u U t ß m ù_ o U ²
úLW[ Uô¨Xj§p YßûUdúLôh¥tÏ SôÓLs Ds[]. (_]Y¬ 1)
¸Ýs[ ÑUôo 2 ªp-Vu ÏÓmTe NSE @±Ü ûUVm
LÞdÏ BXYN @§úYL BûQV Uj§V YojRL Õû\ @ûUfNLm
BûQlûT YZeL LiQô¥ ØmûT«p NSE @±Ü ûUVjûR
BûZNôo YûXVûUl× §hPj§tÏ ùRôPe¡Ùs[Õ.
@mUô¨X @WÑ Il×Rp @°jÕs[Õ. BÕ IÚ ùNVtûL ÖiQ±®u
©uXôkÕ, FvúRô²Vô, ©Wôuv, ê X m (Artificial Intelligence# Al)
vùT«u, ¡Ãv, úLôvPô¬Lô ùNVpTÓjRlTÓm Lt\p ãZp
úTôu\ IÚ £X SôÓLs UhÓúU @ûUlTôÏm.
BûQVjûR @¥lTûP U²R D¬ûU BkR R[Uô]Õ úR£V TeÏ NkûR
Fuß @±®jÕs[]. (¥NmTo 21) «]ôp ØÝYÕm ¨oY¡dLlTÓm.
ªLlùT¬V ùTôÚ[ôRôW SôÓLs (_]Y¬ 8)
2026#Am Ai¥p ù_oU²ûV £\kR Lp® ¨ßY]eLs
©uàdÏ Rs°, Bk§Vô DX¡u Uj§V @WÑ ùY°«hP Bk§Vô®u
SôuLôYÕ ªLlùT¬V ùTôÚ[ôRôW £\kR Lp® ¨ßY]eLÞdLô]
UôL DÚYùYÓdÏm F]Üm, Th¥V-p @iQô TpLûXdLZLm
2034#Am Ai¥p _lTôû] 14#YÕ BPjûRÙm, úLôûY TôW§Vôo
©uàdÏ Rs° DX¡u êu\ôYÕ TpLûXdLZLm 21#YÕ BPjûRÙm
ùTôÚ[ôRôWUôL DÚùYÓdÏm F] ©¥jÕs[].
ùR¬®dLlThÓs[Õ. @úR úTôp Sôh¥u £\kR 200
Be¡XôkûR úNokR ùTôÚ[ôRôW ùTô±«Vp ¨ßY]eLÞdLô]
Utßm YojRL AWônf£ ûUVj§u Th¥V-p ùNuû] HH¥ ØR-Pj
AnY±dûL«p BkR RLYp ûRÙm, @iQô TpLûXdLZLm
ùY°«PlThÓs[Õ. 9#YÕ BPjûRÙm, §Úf£ NIT 10#YÕ
BlúTôÕ Bk§Vô 20.49 ¥¬p-Vu BPjûRÙm ©¥jÕs[Õ. (_]Y¬ 10)

RûX£\kR SôÓLs Th¥Vp


Tôq Fu\ DXL[ô®V AúXôNû]
ûUVØm úNokÕ ùY°«hÓs[].
BVtûLVô] ãZ-p YôZdá¥V
SôÓL°p ØR-Pj§p ÑÅPu,
B W i P ô Y Õ B P j § p
Ñ®hNoXôkÕ, êu\ôYÕ BPj§p
©uXôkÕm Ds[].
@û]jÕ ®Rj§Ûm £\kR
Lp®, YôrdûLjRWm, TôÕLôl×, SôÓLs Th¥V-p Ñ®hNoXôkÕ
ùTôÚ[ôRôWm úTôu\ @mNeLû[ ØR-Pj§Ûm, L]Pô BWiPôYÕ
ØuûYjÕ 2020#Am Ai¥tLô] BPj§Ûm, _lTôu êu\ôYÕ
RûX£\kR SôÓL°u Th¥VûX, BPjûRÙm ùTtßs[].
DXL @±dûL Fu\ Tj§¬dûLÙm, (_]Y¬ 15)

 A-Series in
          ©lWY¬ 2020
£\kR Fm.©.dLs ®ÚÕ
ULôWôx¥Wô Uô¨X Øu]¦ Á¥Vô
ÏÝUUô] úXôdUj Nôo©p, £\kR
®ÚÕLs
Fm.©.dLÞdÏ AiÓúRôßm ®ÚÕ
YZeLlThÓ YÚ¡\Õ. _ýùYuPv @¦ ùYpX DR®]ôo.
B§p £\kR Uô¨XeL[ûY Fm.©. úUÛm úS`uv Äd ThPjûR
®ÚÕ §Úf£ £YôÜdÏm, £\kR úTôofÑdLp ùYpXÜm ùWô]ôpúPô
UdL[ûY Fm.©. ®ÚÕ Øu]ôs LôWQUôL BÚkRôo.
Uj§V Uk§¬ NôLj WôndÏm ¡±v¥Vôú]ô ùWô]ôpúPô BkR
YZeLlThPÕ. ®ÚûR ùYpYÕ BÕ 9#YÕ Øû\
úUÛm, UdL[ûY £\kR ùTi FuTÕ Ï±l©PjRdLÕ.
Fm.© ®ÚÕ Ñl¬VôÑúXdÏm, UL°o £\kR ÅWôeLû] ®ÚûR
Uô¨XeL[ûY £\kR ùTi Be¡XôkÕ ÅWôeLû] í³
Fm.©. ®ÚÕ ®lXl RôáÚdÏm ©Wôuv ùTt\ôo. £\kR T«t£Vô[o
YZeLlThPÕ. ®ÚûR -Yoép úUXô[o _ýLu
ùPp-«p Ds[ @múTjLo ¡[ôl ùYu\ôo. (_]Y¬ 1)
NoYúRN ûUVj§p SPkR ®Zô®p
úR£V Y¥YûUl× ®ÚÕLs
ÕûQ _]ô§T§ ùYeûLVô SôÙÓ
LXkÕùLôiÓ ®ÚÕLû[ YZe ûaRWôTôj§p SûPùTt\ 34#YÕ
¡]ôo. (¥NmTo 22) Bk§V ùTô±«Vp UôSôh¥p
2019#Am AiÓdLô] úR£V
Ïú[ôl NôdLôo ®ÚÕ Y¥YûUl× Utßm AWônf£ ¨ßY]j
_ýùYuPv @¦ ÅWo ¡±v¥Vôú]ô §u úR£V Y¥YûUl× ®ÚÕLs
ùWô]ôpúPô 2019#Am AiÓdLô] YZeLlThP].
ÕûT Ïú[ôl NôdLo ®ÚûR §ÚY]kR×Wj§p Ds[ BvúWô#§WY
ùYußs[ôo. DkÕNd§ ûUV BVdÏSo ®gOô²
LPkR 2019 º¬ G Nôm©Vu ThPjûR SôWôVQàdÏ úR£V Y¥YûUl×

úLôpPu Ïú[ôl ®ÚÕLs


77#YÕ úLôpPu Ïú[ôl ®ÚÕLs
YZeÏm ®Zô @ùU¬dLô®u
L-úTôo²Vô UôLôQm Xôv
GgNpv SL¬p Ds[ ùTYo-
¶p£p SûPùTt\Õ.
B§p £\kR §ûWlTPUôL, ØRXôYÕ
DXLlúTôûW ®Y¬dÏm 1917 Fu\
aô-Üh TPm úRoYô]Õ. BkR
TPj§u BVdÏ]o Nôm ùUu¥v @úR úTôp £\kR S¥ûLdLô]
£\kR BVdÏ]ÚdLô] ®ÚûR ®ÚÕ _þ¥ TPj§p S¥jR ùW²
ùTt\ôo. ù_púYLÚdÏ YZeLlThPÕ.
DXLm ØÝYÕm ùTÚm YWúYtûT £\kR BûNVûUlTô[ÚdLô]
ùTt\ ú_ôdLo TPj§p S¥jR ®ÚÕ ú_ôdLo TPj§tLôL ¶pPo
ú_ôÏ«u úTô²dv £\kR Ï]ôúPôh¥ÚdÏ YZeLlThPÕ.
S¥LÚdLô] ®ÚûR Rh¥f (_]Y¬ 6)
ùNu\ôo.

©lWY¬ 2020 A-Series in


  
©££H ®ÚÕLs
B k ¨ûX «p © ££H «u D V ¬ V
®ÚRô] Tô- Dm¬Lo ®ÚÕ Bk§V
@¦«u úYLlTkÕ ÅfNô[o
_v©¬h ×mWôÜdÏ YZeLlThPÕ.
cLôkj @gÑm úNôlWô Hv©¬h ×mWô
Bk§V UL°o @¦«p Ød¡V
TkÕÅfNô[WôL Ds[ é]m
VôRÜdÏ £\kR ÅWôeLû] ®ÚÕ
YZeLlThPÕ..
YôrSôs NôRû]Vô[o ®ÚûR
Øu]ôs úLlPu cLôkjÕm,
é]m VôRq NTô- YoUô UôVed @LoYôp
@gÑm úNôlWôÜm ùTt\]o.
©££H NôoTôL AiÓúRôßm £\kR @±ØL ÅWÚdLô] ®ÚûR
NoYúRN ¡¬dùLh¥p £\kÕ UôVed @LoYôÛm, £\kR @±ØL
®[eÏm Bk§V ÅWo, ÅWôeLû] ÅWôeLû]dLô] ®ÚûR NTô-
LÞdÏ ®ÚÕLs YZeLlThÓ YoUôÜm ùYu\]o. (_]Y¬ 12)
YÚ¡\Õ.
®ÚÕ YZeLlThPÕ. ©WTX ®ÓRûXlúTôWôhP ÅWo
@d²#4#Bu §hP BVdÏSÚm úXôdUôuV §XLWôp ùRôPeLlThP
¥.Ao.¥.J. ®gOô²ÙUô] úLN¬ Sô°R¯u ùRôPdL Sôû[
¡ú`ôoSôjÕdÏ BVk§Wl ¨û]ÜáÚm YûL«p GtTôÓ
ùTô±«Vp Y¥YûUl× ®ÚÕm, ùNnVlThP ¨Lr®uúTôÕ
I¥Nô®p Ds[ Uj§V LhPP @YÚdÏ BkR ®ÚÕ YZeLlThPÕ.
AWônf£ ¨ßY]j§u ®gOô² (_]Y¬ 4)
@úNôdÏUôÚdÏ LhPPdLûXdLô] NoYúRN úVôLô §] ®ÚÕ
Y¥YûUl× ®ÚÕ YZeLlThPÕ. EPL ¨ßY]eLÞdLô] NoYúRN
¥.Ao.¥.J. ¨ßY]j§u Y[eLs# úVôLô §] ®ÚÕ ùNuû] çoRo`u,
úUXôiûU ùTôÕ BVdÏSo £jWô Rk§ ÏÝUjûR úNokR aúXô
Wô_úLôTôÛdÏ ùTi @±®VXô[ Fl.Fm. Yôù]ô- A¡VYt±tÏ
ÚdLô] ÑUu NoUô ®ÚÕ YZeLlThÓs[].
YZeLlThPÕ. (_]Y¬ 2) Bq®ÚÕ ùUôjRm 30 EPL
úXôdUôuV §XLo ®ÚÕ ¨ßY]eLÞdÏ YZeLlThPÕ
Øu]¦ Bk§ Sô°RZô] _dWu ϱl©PjRdLÕ.
RûXûU A£¬Vo NgNn ÏlRôÜdÏ _þu 2019#Bp קRôL DÚYôdLlThP
úXôdUôuV §XLo úR£V Tj§¬ûL BkR ®ÚÕ, EPLm êXm
®ÚÕ YZeLlThPÕ. úVôLôûY £\kR Øû\«p TWl×ûW
Tj§¬dûL Õû\«p @Yo At±V ùNnÙm EPL ¨ßY]eLÞdÏ
£\lTô] T¦dLôL BkR ®ÚÕ ùT\ YZeLlTÓ¡\Õ. (_]Y¬ 8)
@Yo úRoÜ ùNnVlThPôo. ØlTYWl× ùYeûLV SôÙÓ ®ÚÕ
Tj§¬dûL Õû\«p R²SToLs TÑûU ×Wh£«u RkûR Fu\ûZdLl
A t ß m T e L ° l × L û [ TÓm Fm.Fv. ÑYôªSôRàdÏ
@e¸L¬dÏm úSôd¡p ×ú]ûY ØlTYWl× ùYeûLV SôÙÓ £\l×
úNokR úLN¬#UWôjRô @\dLhPû[ úR£V ®ÚÕ YZeLlThÓs[Õ.
BkR ®ÚûR ¨ß®VÕ. (_]Y¬ 10)

 A-Series in
          ©lWY¬ 2020
©©u WôYj
©WRUo RûXûU«p SûPùTt\ Uj§V
@ûUfNWûY áhPj§p, RûWlTûP,
©WTXUô]YoLs
LPtTûP Utßm ®Uô]lTûPdÏm
úNojÕ IúW RûXûU R[T§ûV BYo RtúTôÕ ùTPWp Y¬ úNûY«u
¨VªdL Il×Rp @°dLlThPÕ. RûXYWôL TR® Y¡jÕ YÚ¡\ôo.
BûRVÓjÕ, WôÔY R[T§ ©©u WµV ©WRUo ¥ªh¬ ùUjYúRq
WôYjûR, ØlTûPLÞdLô] ØRp R]Õ TR®ûV §¼ùW] Wô´]ôUô
RûXûU R[T§VôL Uj§V @WÑ ùNnRûRVÓjÕ ×§V ©WRUWôL
¨VªjRÕ. ªdûLp ª`þv¥u ¨VªdLl
WôÔY R[T§ ùTôßl©p ThÓs[ôo.
BÚkÕ ©©u WôYj JnÜ ùTt\ ©WRUo TR®ûV Wô´]ôUô ùNnR
ûRVÓjÕ, ØlTûPL°u ØRp ¥ªh¬ ùUjYúRq WµVô®u
RûXûU R[T§VôL ùTôßlúTt\ôo. TôÕLôl× LÜu£-u ÕûQ
(_]Y¬ 1) RûXYWôL ¨VªdLlThÓ Ds[ôo.
G.©.UúLxY¬ (_]Y¬ 16)
DX¡u ªLlùT¬V ÕûQ WôÔYl ûUdúLp úRYlWRô TjWô
TûPVô] £Ao©FKl#Bu 1935#Am AiÓ GlWp 1, LpLjRô®p
RûXYWôL G.©.UúLxY¬ûV Uj§V ùRôPeLlThP Bk§V ¬Noq
T¦Vô[o @ûUfNLm ¨VªjÕs[Õ. Ye¡«u RûXûU«Pm ØmûT«p
Uj§V ¬Noq TôÕLôl× TûP Ds[Õ.
Uj§V DsÕû\ @ûUfNLj§u ¸r ¬Noq Ye¡«u RtúTôûRV AÞSWôL
ùNVpTÓ¡\Õ. Nd§ LôkR Rôv Ds[ôo.
G.©. UúLxY¬ Uj§V DsÕû\
Bk¨ûX«p ¬Noq Ye¡«u SôuLôYÕ
@ûUfNLj§u R²f ùNVXWôL TR®
ÕûQ AÞSWôL ûUdúLp úRYlWRô
Y¡jÕ YkRôo. (_]Y¬ 12)
TjWô ¨VªdLlThÓs[ôo.
ªdûLp ª`þv¥u
WµVô®u קV ©WRUWôL ªdûLp BYo êu\ôiÓ LôXm BlTR®ûV
ª`þv¥u ¨VªdLlThÓs[ôo. Y¡lTôo. (_]Y¬ 16)
UXôXô ëÑlNôn
LPkR 10 AiÓL°p DX¡u
ªLÜm ©WTXUô] B[mùTiQôL
Tô¡vRôû] úNokR UXôXô
ëÑlNôûV úRoÜ ùNnÕ H.Sô
LÜW®jÕs[Õ.
ùTiL°u Lp®dLôL úTôWô¥V
UXôXôûY LPkR 2012#Am
AiÓ RÄTôu TVeLWYô§Ls
ÕlTôd¡Vôp ÑhP]o. F²àm YZeLlThPÕ. 2017#Am AiÓ
BkR RôdÏR-p @Yo LôVeLÞPu H.Sô.®u @ûU§dLô] çRWô]ôo.
D«o Rl©]ôo. @Ru©\Ï @Yo Tô¡vRôu UhÓm
2014#Am AiÓ @YÚdÏm, Bu± NoYúRN @[®p ùTiL°u
Bk§Vô®u ûLXô‌‌x NjVôoj§dÏm Lp® D¬ûUdLôL ÏWp ùLôÓdL
áhPôL @ûU§dLô] úSôTp T¬Ñ ùRôPe¡]ôo. (¥NmTo 28)

©lWY¬ 2020 A-Series in


  
SPl× ¨LrÜLs ®]ô#®ûPLs
(¥NmTo # 2019)
51. ùY°SôhÓ IjÕûZl×j Õû\ Fu\ C) ¥NmTo 22
קV Õû\ûV DÚYôd¡V Uô¨Xm? D) ¥NmTo 24
A) a¬Vô]ô 57. YôL]eL°p ØRp RÓlé£ ¡°²d
B) úULôXVô FeÏ ùRôPeLlThÓs[Õ?
C) TgNôl A) ùLôf£
D) úLW[ô B) ùNuû]
52. 2020, _]Y¬ 1#Am úR§ @ÚkR§ C) ×ú]
vYoQô úVô_]ô §hPjûR FkR D) ØmûT
Uô¨X @WÑ ùRôPeLÜs[Õ? 58. 11#YÕ ©Wôk§V RW UôSôÓ FeÏ
A) @Nôm SûPùTt\Õ?
B) Nh¼vLo A) úPWôåu
C) RªrSôÓ B) RWmNôXô
D) I¥Nô C) úTôTôp
53. £û\ ¨oYôLj§p ºÚûP @¦kR D) ÚjWéo
ùTiLs ϱjR úR£V UôSôÓ FeÏ 59. 2 0 1 9 # A m A i Ó d L ô ] £ Ú µ
SûPùTt\Õ? LoUu ®ÚÕ FkR Uô¨Xj§tÏ
A) £mXô YZeLlThPÕ?
B) ùPp- A) úLW[ô
C) ×ú] B) LoSôPLm
D) ùNuû] C) RªrSôÓ
54. BYt±p FkR Sôh¥u ©WRUo Lôk¨ D) Ak§W©WúRNm
Ï¥Ù¬ûU Lp® ®ÚûR @±®jRôo? 60. ®iùY°«p Col× @ûXLs ϱjÕ
A) vùT«u AnÜdLô] ØRp ùNVtûLdúLôû[
B) RônXôkÕ FkR SôÓ G®Ùs[Õ?
C) úTôofÑLp A) º]ô
D) BjRô- B) WxVô
55. TÑûU F¬Nd§ ®ÚÕ 2019 ùYu\ C) L]Pô
¨ßY]m? D) _lTôu
A) BHEL 61. Bk§Vô®u ØRp Tiger Cell FeÏ
B) PôPô @ûUdLlThÓs[Õ?
C) Al©s A) úPWôåu
D) ¬ûXVuv B) UÜih @×
56. NoYúRN U²R ItßûU §]m Fuß C) Lôk§ SLo
@àN¬dLlTÓ¡\Õ? D) £mXô
A) ¥NmTo 20 62. FkR Uô¨Xj§p ØRpØû\VôL
B) ¥NmTo 21 NhPNûTdÏ 10 ùTi Fm.Fp.GdLs

 A-Series in
          ©lWY¬ 2020
úRoÜ ùNnVlThÓs[]o? B) I¥Nô
A) ÀLôo C) úLW[ô
B) _ôodLih D) _ôodLih
C) I¥Nô 69. ØRXûUfNo ÑúTô`u @©Vôu
D) úUtÏ YeLm FuTÕ FkR Uô¨Xj§u §hPUôÏm?
63. ùTiLû[TôÕLôlTôL Åh¥p A) Nh¼vLo
úNodÏm §hPUô] ABHEY FkR B) LoSôPLm
Uô¨Xj§p ùRôPeLlThÓs[Õ? C) ULôWôx¥Wô
A) RªrSôÓ D) @Nôm
B) Ak§W ©WúRNm 70. í§Vô]ô ×-Ls LôlTLm FkR
C) LoSôPLm Uô¨Xj§p @ûUkÕs[Õ?
D) ULôWôx¥Wô A) @Nôm
64. FkR SL¬p ùNg£ÛûY NeLj§u B) £d¡m
ùNV- ùY°«PlThPÕ? C) TgNôl
A) ùTeLðÚ D) §¬×Wô
B) §ÚY]kR×Wm 71. Bk§V UÚk§Vp èûX @e¸L¬jR
C) ûaRWôTôj ØRp SôÓ?
D) ùPp- A) AlLô²vRôu
65. @Pp é_p úVôw]ô Fàm §hPm B) CWôd
FkR úSôdLj§tLôL ùRôPeLl C) _lTôu
ThPÕ? D) º]ô
A) ¨XjR¥ ¿o úUXôiûU 72. ¨X®p ¿iPLôXm BÚkRRtLô]
B) UûZ¿o úNL¬l× NôRû]ûV @iûU«p رV¥jR
C) Ï¥¿o úUXôiûU úUtTWl× Eo§?
D) L¯Ü¿o Ñj§L¬l× A) Viper
66. §ÚSeûLLÞdLô] Bk§Vô®u ØRp B) Yutu 2
TpLûXdLZLm FkR Uô¨Xj§p C) Pragyan
@ûUdLlTP Ds[Õ? D) Lunokhad 1
A) ULôWôx¥Wô 73. @ùU¬dLô®u NhPUô] DASKA,
B) Wô_vRôu FkR Sôh¥tÏ F§WôL ùTôÚ[ôRôW
C) Dj§W©WúRNm RûPLû[ ®§d¡\Õ?
D) úLW[ô A) WxVô
67. FkR Uô¨X @WNôp _XN§ §hPm B) CWôd
ùRôPeLlThÓs[Õ? C) Bk§Vô
A) RªrSôÓ D) _lTôu
B) I¥Nô 74. FkR Sôh¥p aôo©u T² §Ú®Zô
C) ÀLôo ùLôiPôPlTÓ¡\Õ?
D) úLW[ô A) _lTôu
68. IußdÏ T§XôL 3 TWYXôdLlThP B) ©uXôkÕ
RûXSLWeLû[ DÚYôdÏYRtÏ C) º]ô
§hPªhÓs[ Uô¨X @WÑ?
D) BjRô-
A) Ak§W ©WúRNm

©lWY¬ 2020 A-Series in


  
75. LXTúLôv úR£VéeLô @ûUkÕs[ 82. UiÓ ®Zô®u ØRp T§l× FeÏ
SôÓ? SûPùTt\Õ?
A) CdÏúYPôo B) ©úW£p A) Uj§V ©WúRNm B) Wô_vRôu
C) ùTÚ D) £- C) ÀLôo D) úLW[ô
76. UôàúYp UoúWúWô Ïìv FkR 83. Bk§V WôÔYj§u FjRû]VôYÕ
Sôh¥u קV ©WRUWôL úRoÜ R[T§VôL Uú]ôw ØÏkj SWôYú]
ùNnVlThÓs[ôo? ùTôßlúTtßd ùLôiPôo?
A) £- A) 26 B) 27
B) LmúTô¥Vô C) 28 D) 30
C) ¡ëTô 84. 2019#20#Am Ai¥tLô] ¨ûXVô]
D) ù_oU² @©®Új§ BXdÏLs ϱÂh¥p
77. @Wôd @Ô DûX FkR Sôh¥p ØR-Pm ©¥jR Uô¨Xm?
@ûUkÕs[Õ? A) LoSôPLm
A) Bk§Vô B) úLW[ô
B) YPùLô¬Vô C) RªrSôÓ
C) CWôu D) I¥Nô
D) Tô¡vRôu 85. NoYúRN ùUôûTp LÚ® @ûP
78. FkR Uô¨Xj§p Ds[ úWôau Vô[jûR Gtßd ùLôiP ØRp SôÓ?
ÑWeLm @Pp ÑWeLlTôûR F] A) Av§úW-Vô B) NûU
ùTV¬PlThPÕ? C) _lTôu D) ùRuùLô¬Vô
A) BUôfNX©WúRNm 86. R à V ô j W ô ù L ô i P ô P l T Ó m
B) ÀLôo Uô¨Xm?
C) _ôodLih A) ÀLôo B) _ôodLih
D) @Nôm C) I¥Nô D) úLW[ô
79. NÁTj§p JnÜùTt\ FkR úTôo 87. 2020#Am Ai¥u YÚm IqùYôÚ
®Uô]m TLço Fußm @ûZdLl UôRj§u ØRp Sô°Ûm No Vechicle
TÓ¡\Õ? day LûPl©¥dLÜs[ Uô¨Xm?
A) úR_ôv A) Wô_vRôu
B) ªd 27 B) úLW[ô
C) ªWôw 2000 C) Ï_Wôj
D) BYt±p FÕܪpûX D) RªrSôÓ
80. YVô YkR]ô úVô_]ô®u ¸r TV] 88. FkR Uô¨Xm R]Õ ØRp T²f£ßjûR
ûPVd á¥VYoLs? LQdùLÓlûT SPjRÜs[Õ?
A) êjR Ï¥UdLs A) úUtÏ YeLm
B) ®RûYLs B) BUôfNX ©WúRNm
C) E]Øt\YoLs C) Dj§WLôih
D) A£¬VoLs D) TgNôl
81. TôÕLôl×j R[T§Ls @§LThNUôL 89. NÁTj§p DÚYôdLlThP ûNuVô
FjRû] YVÕ YûW T¦Vôt\ Uj§V LojRô ®Tôd¡u RûXYo?
@WÑ Il×Rp YZe¡Ùs[Õ? A) ©WRUo
A) 60 B) 62 B) ¨§VûUfNo
C) 64 D) 65 C) TôÕLôl×lTûPj RûXYo

 A-Series in
          ©lWY¬ 2020
D) RªrSôÓ A) Bkço B) úTôTôp
90. TYZlTôû\LÞdÏ ¾eÏ ®û[®dÏm C) ùNuû] D) ùLôf£
ã¬V I° Lôl×l éfûN RûPùNnR 96. 5 DRDO B[m ®gOô²Ls AnY
SôÓ? LeLs FkR Uô¨Xj§p @ûUdLlTP
A) ¨ë£XôkÕ B) Av§úW-Vô Ds[]?
C) ùS[ì D) TXôÜ A) úLW[ô B) LoSôPLô
91. 2020#Am Ai¥u ×jRôiûP C) I¥Nô D) Ï_Wôj
ùLôiPô¥V DX¡u ØRp ùT¬V 97. FkR Uô¨Xj§p 5,000 Ts°Ls
SLWm? ùNVtûLdúLôs ùRô¯pÖhTm
A) £h² B) AdXôkÕ êXm BûQdLlTP Ds[]?
C) ×ÕùPp- D) £úVôp A) Wô_vRôu B) I¥Nô
C) úLW[ô D) ULôWôx¥Wô
92. ¡ Ú µ L o U ô u ® Ú § u ê X m
®YNôVj§u FkR @mNj§p ªÏkR 98. 3#YÕ úLúXô Bk§Vô Bû[Oo
LY]m ùNÛjRlTÓ¡\Õ? ®û[VôhÓLs FeÏ SûPùT\
A) DtTj§ Üs[Õ?
B) ®tTû] A) ×ú]
B) ØmûT
C) ®²úVôLm
C) LÜLôj§
D) ¿o úUXôiûU
D) ùLôf£
93. Lôp Utßm Yôn úSôn LhÓlTÓjR
99. DXL ×jRL LiLôh£«u 28#YÕ
DX¡u ªLlùT¬V RÓlé£ §hPjûR
T§l× FeÏ SûPùT\ Ds[Õ?
ùRôPeLÜs[ SôÓ?
A) ×ÕùPp-
A) Bk§Vô
B) ùNuû]
B) UúX£Vô
C) úTôTôp
C) Al©¬dLô
D) ù_nléo
D) ªVôuUo
100. @Úi ù_h-«u ©\kR Sôû[
94. TôoûYVt\ SToLÞdLô] MAM
IqùYôÚ AiÓm IÚ Uô¨X
®iQlTjûR @±ØLlTÓj§V
®ZôYôL ùLôiPôP FkR Uô¨X
Ye¡
@WÑ Ø¥Ü ùNnÕs[Õ?
A) AXIS B) ICICI
A) Ï_Wôj
C) RBI D) SBI
B) a¬Vô]ô
95. çnûUVô] SLWeLs Th¥V-p C) ÀLôo
ØR-Pm ©¥jR SLWm? D) @Nôm

51. A 52. A 53. B 54. C 55. A 56. A 57. C 58. D 59. C 60. A 61. A 62. B

63. B 64. C 65. A 66. C 67. B 68. A 69. A 70. C 71. A 72. B 73. A 74. C

75. A 76. C 77. C 78. A 79. B 80. A 81. D 82. A 83. C 84. B 85. A 86. C

87. A 88. C 89. C 90. D 91. B 92. A 93. A 94. C 95. A 96. B 97. A 98. C

99. A 100. C

©lWY¬ 2020 A-Series in


  
SPl× ¨LrÜLs ®]ô#®ûPLs
(_]Y¬ # 2020)
1. _]Y¬ 2 ØRp Yô¥dûLVô[¬u Ls AûQVj§u ØRp ThÓ
ÅhÓ YôN-p UQûX YZeÏm TRlTÓjÕm ùRô¯tNôûX FkR
§hPjûR FkR Uô¨X @WÑ Uô¨Xj§p @ûUdLlTPÜs[Õ?
@±®jRÕ? A) Wô_vRôu B) ÀLôo
A) Ak§W ©WúRNm C) Ï_Wôj D) úLW[ô
B) ÀLôo 9. F k R U ô ¨ X ú T ô d Ï Y W j Õ û \
C) úLW[ô ùTiLÞdLô] Rôª² Fu\ DR®
D) RªrSôÓ ûUV FiûQ @±ØLlTÓj§Ùs[Õ?
2. BYoL°p úXôdUôuV §Xd úR£V A) DjRWLôih
Tj§¬ûLVô[o ®ÚûR ùYu\Yo? B) Dj§W ©WúRNm
A) NgNn ÏlRô C) LoSôPLm
B) @o]l úLôvYôª D) @Nôm
C) @ÚkR§ Wôn 10. 14#YÕ DXL ÑLôRôW áÓûL 2021 FeÏ
D) W®vÏUôo SûPùT\Üs[Õ?
3. ÀUô#úLô úWLôu úTo ¨û]Ü §]m A) ®NôLlTh¥]m
@àN¬dLlTÓYÕ? B) PôdLô
A) _]Y¬ 1 B) _]Y¬ 2 C) ùTn´e
C) _]Y¬ 3 D) _]Y¬ 4 D) ×ÕùPp-
4. B v ú W ô ® u × § V N k § W V ô u # 3 11. 107#YÕ Bk§V @±®Vp UôSôÓ FkR
§hPj§p ùNXÜ U§lÀÓ FqY[Ü? SL¬p SûPùTt\Õ?
A) ì. 610 úLô¥ B) ì. 612 úLô¥ A) ùNuû]
C) ì. 615 úLô¥ D) ì. 618 úLô¥ B) ùLôf£
5. BYt±p ѲRô XdWô ùRôPo×ûPV C) ùTeLðÚ
®û[VôhÓ? D) úTôTôp
A) ùPu²v B) ¡¬dùLh 12. A e ¡ X × j R ô i Ó § ] j § p
C) TôhªiPu D) aôd¡ DX¡úXúV @§L[®p ÏZkûRLs
6. NÁTj§p @±ØLlTÓjRlThP ©\kÕs[ SôÓ?
Bk§V W«púY«u קV A) Bk§Vô B) º]ô
IÚe¡ûQkR DR® ûUV Fi? C) ûSˬVô D) @ùU¬dLô
A) 138 B) 139 13. DX¡p NêL YûXjR[j§p @§L
C) 140 D) 141 úSWjûR ùNX®ÓY§p ØR-Pj§p
7. @ùU¬dL ®Uô]lTûP RôdÏR-p Ds[ SôÓ?
ùLôpXlThP Lô£m ÑûXUô² FkR A) Bk§Vô B) º]ô
Sôh¥u WôÔYj R[T§? C) ©Wôuv D) ©-lûTuv
A) CWôu B) CWôd 14. Bk§Vô®u ØRp SuÉo AûULs
C) £¬Vô D) BvúWp UßYôrÜ ûUVm FeÏ @ûUdLlTP
8. Lô§ Utßm ¡WôUl×\ ùRô¯tNôûX Üs[Õ?

 A-Series in
          ©lWY¬ 2020
A) LoSôPLm B) I¥Nô C) ùTiLs §\u
C) ÀLôo D) _ôodLih D) ®û[VôhÓ ÅWoLs §\u
15. úR£V úT¬Po Áh× TûP @LôPª 22. KTdNô T\ûYLs §Ú®Zô#2020 FkR
FkR Uô¨Xj§p @ûUdLlTPÜs[Õ? Uô¨Xj§p SûPùTt\Õ?
A) DjRW©WúRNm A) @Nôm
B) RªrSôÓ B) RªrSôÓ
C) ùRÛeLô]ô C) úUtÏ YeLm
D) úLW[ô D) LoSôPLm
16. SeL]ô Nô¡l ÏÚjYôWô @ûUkÕs[ 23. `ôeLôn IjÕûZl× @ûUl©u
SôÓ? 8 @§NVeLs Th¥V-p BPm
A) Tô¡vRôu B) Bk§Vô ùTtßs[ Bk§V ¨û]Üf £u]m?
C) éhPôu D) úSTô[m A) RôwULôp
17. ã¬V éeLôdLû[ ¨ßÜYRtLôL B) ItßûU«u £ûX
Bk§Vô®PªÚkÕ 75 ªp-Vu C) @_kRô ÏûLLs
PôXo ùTtßs[ SôÓ? D) RgûN ©WL¾vYWo úLô«p
A) £- 24. ØRXôYÕ úLúXô Bk§Vô
B) ¡ëTô TpLûXdLZL ®û[VôhÓLs 2020
C) ©úW£p FkR SL¬p SûPùT\Üs[Õ?
D) @où_u¥]ô A) ×Yú]vYo
18. Uô]v úR£V éeLô @ûUkÕs[ B) ®NôLlTh¥]m
Uô¨Xm? C) Bkço
A) úLW[ô D) ×ú]
25. úR£V Bû[Oo ®Zô#2020 FkR
B) LoSôPLm
Uô¨Xj§p SûPùTt\Õ?
C) @Nôm
A) DjRW ©WúRNm B) I¥Nô
D) Ï_Wôj C) Nh¼vLo D) Ï_Wôj
26. 31#YÕ NoYúRN LôjRô¥ §Ú®Zô
19. _p#ËYu# a¬Vô- §hPm FuTÕ FeÏ SûPùTt\Õ?
FkR Uô¨Xj§u ØRuûU LôX¨ûX A) ùLôf£
Uôt\ §hPUôÏm?
B) @LURôTôj
A) Nh¼vLo
C) ùNuû]
B) I¥Nô
D) ûaRWôTôj
C) ÀLôo
27. 34#YÕ DXL úTôdÏYWjÕ Utßm
D) Ï_Wôj
ÑtßXô úTôh¥ Th¥Vp 2019#Bp
20. DXL ©ùWn- §]m @àN¬dLl ØR-Pm ©¥jR SôÓ?
TÓYÕ? A) ©Wôuv
A) _]Y¬ 2 B) _]Y¬ 3 B) ù_oU²
C) _]Y¬ 4 D) _]Y¬ 5 C) Bk§Vô
21. Prodigy ®ÚÕ ¸rdLiP FRú]ôÓ D) vùT«u
ùRôPo×ûPV IÚ ®ÚRôÏm?
28. 2 0 2 0 U ô o £ p ª X u F u à m
A) ÏZkûRLs §\u Tu]ôhÓ LPtTûP T«t£ FkR
B) A£¬VoLs §\u SL¬p SûPùT\Üs[Õ?

©lWY¬ 2020 A-Series in


  
A) úLôYô Rôe¡ LlTXô] ®dWôkj FkR
B) ®NôLlTh¥]m Ai¥tÏs Bk§V LPtTûP«p
C) ØmûT BVdLlTÓm?
D) ùNuû] A) 2020 B) 2021
29. DX¡u ªL DVWUô] W«pTôXm C) 2022 D) 2024
FeÏ LhPlTP Ds[Õ? 36. ùRtLô£V YojRL Utßm TVQ
A) _mØ#LôxÁo T¬YojRû] FdvúTô#2020 #Bu
27#YÕ T§l× FeÏ SûPùTt\Õ?
B) XPôd
A) ×ÕùPp-
C) Wô_vRôu
B) ØmûT
D) @Nôm
C) ùNuû]
30. S¬Yôp UkRôûWûV R]Õ NhPUu\j
D) ùLôpLjRô
§u קV BXf£û]VôL Gtßd
ùLôiP Uô¨Xm? 37. FkR Uô¨Xj§p ×LrùTt\ ULôúU[ô
SûPùTt\Õ?
A) @Nôm
A) TgNôl
B) @ÚQôfNX ©WúRNm
B) Ï_Wôj
C) ªúNôWm
C) ÀLôo
D) úLW[ô
D) Dj§W ©WúRNm
31. e-PMP ù U ô û T p T V u T ô h û P
@±ØLlTÓj§V Uô¨Xm? 38. Bk§Vô®p ÙYô Yuawaah ù_]úW`u
@u-ªùPh ¨ßY]jûR @±ØLl
A) TgNôl
TÓj§V @ûUl×
B) Ï_Wôj
A) UNICEF
C) a¬Vô]ô
B) WHO
D) úUtÏ YeLm
C) WEF
32. 1901#Am Ai¥-ÚkÕ GZôYÕ
D) IMF
ùYlTUô] AiPôL FkR
AiÓ T§Ü ùNnVlThÓs[RôL 39. 2020 úVôLô @±®Vp áhPm FkR
Bk§V Yô²ûX AnÜ ûUVm SL¬p SûPùTt\Õ?
ùR¬®jÕs[Õ? A) ûaRWôTôj
A) 2016 B) 2017 B) ùTeLðÚ
C) 2018 D) 2019 C) Sô£d
33. ùaXu `oUôu FkR Sôh¥u ØRp D) ùLôf£
ùTi ®iùY° ÅWôeLû]? 40. BYt±p Uô]q RôdLo ùRôPo×ûPV
A) ùSRoXôkÕ ®û[VôhÓ?
B) º]ô A) úP©s ùPu²v
C) ©¬hPu B) ùNv
D) @ùU¬dLô C) ¡¬dùLh
34. Bk§Vô#2021#dLô] UdLsùRôûL D) Tôh ªiPu
LQdùLÓl× FlúTôÕ ùRôPeL 41. TÚl× YûLLs ϱjR áhPm#2020
Ds[Õ? Fu\ ùTV¬p BWiPôiÓdÏ
A) Uôof 1 B) GlWp 1 IÚØû\ SûPùTßm Tu]ôhÓ
C) úU 1 D) _þu 1 TÚl× UôSôÓ SûPùT\Üs[
Uô¨Xm?
35. Bk§Vô®u ØRp DsSôhÓ ®Uô]m
A) ULôWôx¥Wô

 A-Series in
          ©lWY¬ 2020
B) úLW[ô A) Wô_vRôu
C) RªrSôÓ B) DjRW©WúRNm
D) ùRÛeLô]ô C) ÀLôo
42. Lt\p Ïû\TôÓs[ ÏZkûRLÞd D) Ï_Wôj
Lô] £\l× DûW èpLû[ 47. NoYúRN §ûWlTP ®Zô®u T§ù]h
RVô¬dLÜs[ Uô¨X @WÑ? PôYÕ T§l× FeÏ SûPùTt\Õ?
A) LoSôPLm
A) PôdLô
B) Dj§W ©WúRNm
B) ÕTôn
C) TgNôl
D) úUtÏ YeLm C) ×ÕùPp-
43. ùPp- NhPUu\j§p Ds[ ùUôjR D) ùPôWôiúPô
BPeL°u Fi¦dûL? 48. êe¡p#Jo @§NV ×p FuTÕ Tt±V
A) 67 B) 68 T«XWeLm Utßm LiLôh£ FeÏ
C) 70 D) 72 SûPùTt\Õ?
44. IRDAI#Ap ùRôPeLlThP LhPôV A) _mØ#LôxÁo
¨ûXVô] R²STo ÑLôRôW LôlÀhÓ B) @Nôm
ùLôsûL«u ùTVo?
C) úLW[ô
A) AúWôd¡V NgºY²
D) RªrSôÓ
B) AúWôd¡V Wd`ô
49. T£©d Yû[Vj§u ùSÚl× Fuß
C) AúWôd¡V ÑWd`ô
@ûZdLlTÓm SôÓ?
D) AúWôd¡V Nm¬§
A) ©-lûTuv
45. 2020#Am Ai¥u DXL F§oLôX
F¬Nd§ Df£UôSôhûP SPjRÜs[ B) BkúRôú]µVô
SLWm?t C) RônXôkÕ
A) @×Rô©
D) º]ô
B) ¬Vôj
50. 骫u ªL AZUô] ¨XlTWl×
C) ÏûYj Tϧ FeÏ LiÓ©¥dLlThÓs[Õ?
D) CWôu
A) A£Vô
46. DZYoLs ReLs LÚm×l T«ûW
ùYhÓd¡° L°PªÚkÕ B) @iPôo¥Lô
TôÕLôlTÕ Ï±jR ®¯l×QoÜ C) Av§úW-Vô
TWl×ûWûV ùRôPe¡Ùs[ Uô¨Xm? D) Ao¥d

1. A 2. A 3. A 4. C 5. D 6. B 7. A 8. C 9. B 10. A 11. C 12. A

13. D 14. C 15. A 16. A 17. B 18. C 19. C 20. C 21. A 22. C 23. B 24. A

25. A 26. B 27. D 28. B 29. A 30. B 31. A 32. D 33. C 34. B 35. B 36. A

37. D 38. A 39. B 40. A 41. A 42. B 43. C 44. A 45. A 46. B 47. A 48. A

49. A 50. B

©lWY¬ 2020 A-Series in


  
SPl× ¨LrÜLs PV¬d ϱl×Ls
_]Y¬ # 2020
zz RªrSôÓ ¿oY[ ARôWeLs TôÕLôjRp
RªZLm Utßm S§Ls ºWûUjRp LZLUô]Õ
Bk§V LmùT²Ls NhPj§u¸r T§Ü
zz Bk§Vô®úXúV ØRuØRXôL C#váhPo
ùNnVlThÓs[Õ.
Fuß @ûZdLlTÓm @§SÅ] úTôdÏYWjÕ
LôYpÕû\dLô] úWôkÕ YôL]eLs zz LPp TôÕLôl× Utßm LPp AnÜ úUmTôÓ
ùNuû]«p @±ØLlTÓjRlThÓs[]. ϱjR NoYúRNl T«XWeLm ùNuû]
úUÛm Yi¥úVôhÓm ùTiLû[ Ts°dLWûQ«p @ûUkÕs[ úR£V LPp
T¬úNô§lTRtLôL £\l× ùTiLs Nôo ùRô¯pÖhT ¨ßY]j§p (FuHJ¥) 5
úTôdÏYWjÕdLôYp ©¬Üm ùRôPeLlThPÕ. SôhLs SûPùTt\Õ.
zz ùTp, §Úf£ ©¬Ü ×Õl©dLjRdL F¬Nd§ zz R ô r j R l T h P , T Z e Ï ¥ « ] Y Ï l û T f
DtTj§ §hPj§u IÚ TϧVôL 7.5 ùULôYôh úNokRYoLÞdÏ NhPlúTWûYL°Ûm,
§\u ùLôiP ã¬V ªuàtTj§ AûXûV UdL[ûY«Ûm 10 NRÅR BPIÕd¸hûP
¨ß®Ùs[Õ. BRtLôL, 2019#Am Ai¥u YZeL YûL ùNnÙm ¾oUô]m RªZL
TÑûU At\p ®Ú§û] §Úf£ ùTp NhPlúTWûY«p ¨û\úYt\lThPÕ.
¨ßY]m ùTtßs[Õ.
zz 2018#Am Ai¥p ªL @§L ùY°SôhÓ
zz úLW[ Uô¨X @W£u ªL DV¬V ®ÚRô] ÑtßXô TV¦Lû[ LYokR Bk§V Uô¨Xm
a¬YWôN]m ®ÚÕ BûNOô² Bû[V Fàm ùTÚûUûV RªrSôÓ ùTtßs[Õ.
Wô_ôÜdÏ YZeLlThÓs[Õ. 2012#Am
zz §ÚlTjço UôYhPm LpSôoNômTh¥«p
AiÓ ¨ßYlThP Bq®ÚÕ BûNÙXL
13.#Am èt\ôiûP úNokR £j§WúU¯
úUûRLû[ ùL[W®dÏm ®RUôL
LpùYhÓ LiÓ©¥dLlThÓs[Õ. úU¯
AiÓúRôßm YZeLlThÓ YÚ¡\Õ.
FuTÕ DZÜd LXlûT @pXÕ Go Fuß
zz ùNuû] RWU¦«p Ds[ Uj§V @±®Vp ùTôÚsTÓm. £j§WúU¯ Fu\ôp @Z¡V
Utßm ùRô¯pÖhT AWônf£ ûUV LXlûTûV ϱdÏm ùNôpXôÏm.
Y[ôLj§p @ÓjR RûXØû\ At\p
zz Bk§Vô®úXúV @§L PôdPoLs ùLôiP
úNªl×j ¾oÜdLô] קV ùRô¯pÖhT ûUVm
Uô¨XeL°p RªZLm 2#YÕ BPjûR
@ûUdLlTPÜs[Õ.
©¥jÕs[Õ. RªZLj§p ùUôjRm T§Ü
zz FiùQn ®jÕdLs DtTj§«p @§L ùNnVlThP PôdPoL°u Fi¦dûL 1,35,456
®û[fNûX ùTt\ RªrSôhÓdÏ £\kR AÏm. BkR Th¥V-p ULôWôx¥Wô Uô¨Xm
úY[ôi ùRô¯Xô[o ®ÚûR ©WRUo ØR-Pj§p Ds[Õ. @eÏ T§Ü ùNnVlThP
YZe¡]ôo. RªrSôÓ BkR ®ÚûR ùTßYÕ ùUôjR PôdPoL°u Fi¦dûL 1,73,384
BÕ 5#YÕ Øû\VôÏm. úUÛm FiùQn AÏm.
®jÕ DtTj§«p @§L ULãûX RkR
RªrSôhûP úNokR BWiÓ ®YNô«LÞdÏ zz UûZ¿o úNL¬l× Utßm ®YNôVjûR
Y[of£Nôo ®YNô«Ls ®ÚÕ YZeLlThPÕ. EdÏ®dÏm ®RUôL LôYpÕû\ûV
úNokR Bû[Oo U¦ØjÕ LiûQd
zz Bk§Vô®úXúV £\kR SpXôh£dLô] Lh¥dùLôiÓ 5 ¡.Á. ùRôûXÜ J¥ DXL
ϱÂh¥p RªZLm ØR-PjûR ùTtßs[Õ. NôRû] TûPjÕs[ôo.
RWY¬ûNl Th¥V-p RªZLj§tÏ 62
×s°Ls YZeLlThPÕ. zz RgNôî¬p Ds[ Bk§V DQÜ TR]

 A-Series in
          ©lWY¬ 2020
ùRô¯pÖhTdLZLm LÚl×dLܲ @¬£ @Pp é_p úVôw]ô Fuß ùTV¬PlThÓs[
èÓpv Utßm ©[ôv¥d ûTLÞdÏ Uôt\ôL @Pp ¨XjR¥ ¿o §hPjûR ùPp-«p
UdLôfúNô[j§p BÚkÕ ûTLû[ ©WRUo SúWk§W úUô¥ ùRôPe¡ ûYjRôo.
RVô¬jÕs[Õ. ì.6,000 úLô¥ U§l©Xô] @Pp ¨XjR¥ ¿o
§hPm, 2024#Am AiÓdÏs IqùYôÚ
zz DXL NôRû] ¨LrYôL ùNuû], Lôg£×Wm
ÅhÓdÏm ÏZôn êXm Ri½o ¡ûPlTûR
Utßm §ÚYsðo UôYhPeLû[ úNokR
Dߧ ùNnÙm.
10 XhNm Ts° UôQY, UôQ®Ls TeúLt\
ùS¡¯ Uô£pXô RªrSôÓ Fu\ ¨Lrf£ zz Uj§V TôÕLôl× @ûUfNLj§u ¸r קRôL
DߧùUô¯ GtûT RªZL ØRpYo ùRôPe¡ WôÔY ®YLôWeLs Õû\ Fu\ Õû\
ûYjRôo. DÚYôdLlThÓs[Õ. BkR Õû\ ØlTûP
RûXûU R[T§ ©©u WôYj RûXûU«p
zz UÕûW @ZLo UûX Utßm @Rû] NôokR
ùNVpTÓm.
Y]lTϧ RªZL Y]jÕû\dÏ ùNôkR
Uô]Õ F] DfN¿§Uu\m ¾ol× YZe¡Ùs[Õ. zz F§¬ SôhÓ úTôo ®Uô]eLs, ùa-LôlPoLs
Ds°hPYtû\j RûW«-ÚkÕ Rôd¡
zz ùRôûXjùRôPo× YN§Lû[ úUmTÓjÕm
@¯dLYpX QRSAM @§®ûWÜ GÜLûQ
YûL«p ùNuû]#@kRUôu ¾ÜLÞdÏ
ùYt±LWUôL úNôRû] ùNnVlThPÕ. BkR
BûPúV ì.1,224 úLô¥«p ÑUôo 2,250 ¡.Á.
GÜLûQ«u úNôRû] I¥Nô Uô¨Xm TôXôão
ùRôûXÜdÏ LPÛdL¥«p LiQô¥«ûZ
UôYhPj§u Nk§lé¬p Ds[ úNôRû]
Lm© YPm T§lTRtLô] §hPlT¦
ûUVj§p SûPùTt\Õ. BkR QRSAM
ùRôPeLlThÓs[Õ.
GÜLûQ, ®i¦p GYlThP Utù\ôÚ
zz L]Pô Rªr BXd¡Vj úRôhPm @ûUl× GÜLûQûV, Õp-VUôL BûPU±jÕ Rôd¡
YZe¡ YÚm NoYúRN ®ÚÕL°p @¯jRÕ.
ØRuûUVô]Õ BVp ®ÚÕ. 2019#Am
zz RLYp ùRôPo×, NhPm Utßm ¿§ Utßm
AiÓdLô] Bq®ÚÕ Rªr BXd¡V DX¡p
ªu]Ô®Vp Utßm RLYp ùRô¯pÖhT
¾®WUôL BVe¡ YÚm L®OÚm, FÝjRô[Úm,
@ûUfNo c W®NeLo ©WNôj ×ÕùPp-
@W£VpYô§ÙUô] Ñ.ùYeLúPNàdÏ
«p úR£V ©WôhúTih ª`û] (NBM)
YZeLlTPÜs[Õ.
ùRôPe¡ ûYjRôo.
zz ¸Z¥ AnY±dûLûV Rªr, Ae¡Xm,
zz 166 AiÓ LôX Bk§V W«púY YWXôt±p
NUv¡ÚRm, Bk§V Ds°hP 24 ùUô¯L°p
ØRuØû\VôL, SPl× ¨§Vôi¥p (2019#20)
èXôLj RªrSôh¥u ùRôp-Vp Õû\
TV¦Ls B\l× éw´VUôL Ds[Õ F]
ùY°«hÓs[Õ.
W«púY @ûUfNLm ùR¬®jÕs[Õ.

úRNm zz §ÚSeûLLs NêLjûR úNokRYoLÞdLôL


Sôh¥u ØRXôYÕ TpLûXdLZLm DjRWl
zz TôÕLôl× AWônf£ Utßm úUmTôhÓ ¨ßY] ©WúRN Uô¨Xm, ϵSLo UôYhPj§p
Uô] ¥Ao¥J#Yôp DÚYôdLlThP ©]ôLô §\dLlTP Ds[Õ.
GÜLûQ, I¥`ô Uô¨Xm Nk§léo zz ¶UôfNXl ©WúRNjûRÙm, XPôd ë²Vu
LPtTϧ«p Ds[ IÚe¡ûQkR úNôRû] ©WúRNjûRÙm BûQdÏm YûL«p
R[j§-ÚkÕ ùYt±LWUôL T¬úNô§dLlThPÕ. @ûUdLlThÓs[ úWôRe ÑWeLl TôûRûV
ÀWe¡ ùTôÚjRlThP BkR ©]ôLô GÜLûQ, ùTôÕUdLs TVuTôhÓdÏ ©WRUo SúWk§W
F§¬L°u TϧdÏs 75 ¡úXôÁhPo ùRôûXÜ úUô¥ §\kÕ ûYjRôo. LPp UhPj§p
YûW TônkÕ ùNuß RôdLd á¥V §\u ùTt\ BÚkÕ 3,000 ÁhPo DVWj§p ÑUôo 8
RôÏm. ¡.Á. ùRôûXÜdÏ BkR ÑWeLlTôûR
zz ¨XjR¥ ¿o úUXôiûUdLôL DÚYôdLlThP @ûUdLlThÓs[Õ.

©lWY¬ 2020 A-Series in


  
zz Ak§W Uô¨X ØRXûUfNo In.Fv._ zz קVRôL Y[okÕ YÚm Utßm êúXôTôV
LuúUôLu ùWh¥, @]kRéo UôYhPm ùRô¯pÖhTeLs (NEST) ©¬ûY Uj§V
RoUYôWj§p In.Fv.Ao úSRôu]ô ùSvRm ùY°®YLôW @ûUfNLm @ûUj
§hPjûR ùRôPe¡ ûYjRôo. Bj§hPj§u ¸r, Õs[Õ. Sôh¥u Uô¨XeL°ûPúVÙm,
Uô¨Xm ØÝYÕm ûLjR± ùSNYô[oLÞdÏ ùY°SôÓL°Ûm ØRÄhÓ IÚe¡ûQlûT
Ak§W Uô¨X @WÑ AiÓúRôßm ì.24,000 @§L¬dL BkR ©¬Ü ùNVpTÓm.
¨§ DR® YZeÏm. zz Bk§Vô®úXúV RVô¬dLlTÓm ØRp ®Uô]m
zz ×ÕùPp-«p £.Fu.´ £-iPoLÞPu Rôe¡d LlTXô] HFuFv ®dWôkj, @ÓjR
ùTôÚjRlThP Bk§Vô®u ØRp ¿iP çW AiÓ ùRôPdLj§p LPtTûP«p Øû\lT¥
£.Fu.´ Tv úNûY ùRôPe¡ ûYdLlThPÕ. BûQdLlTP Ds[Õ. BkRd LlTp úLW[
IúW ¨Wl©p 1000 ¡úXôÁhPo çWm TV¦dL Uô¨Xm ùLôf£«p Ds[ ùLôf£ µlVôoÓ
Ø¥Ùm. ØRp ¿iP çW BuPovúPh ¨ßY]jRôp LhPlThÓ YÚ¡\Õ.
£.Fu.´ Tv ùPp-«p BÚkÕ ùPyWôåàdÏ zz A£V T£©d TZléf£ (¥úWôúNô©Xô)
ùNpÛm. AWônf£ UôSôh¥u 5#YÕ T§l× ×ú]®p
zz ®Uô] ¨ûXVeL°p TôÕLôlûT @§L¬dÏm SûPùTt\Õ. BÕ Bk§V @±®Vp Lp®
YûL«p E¯VoLs DhTP ®Uô] Utßm AWônf£ ¨ßY]j§]ôp ØRuØû\
¨ûXVj§p T¦Vôtßm @û]jÕ RWl©] VôL Bk§Vô®p SPjRlThPÕ.
ÚdÏm B² TúVôùUh¬d ÖûZÜf ºhÓ zz ULôWôx¥Wô Uô¨XUô]Õ R]Õ ¨XlT§ÜLû[
YZeÏm §hPm ùRôPeLlThÓs[Õ. ©WRôu Uk§¬ TNp ÀUô úVô_]ô®u
zz £ ± V ù N V t û L d ú L ô s G Ü L X u L û [ BûQVR[jÕPu IÚe¡ûQjR Sôh¥u
®i¦p ùNÛjÕYRtLôL çjÕdÏ¥«p 3 ØRXôYÕ Uô¨Xm Fàm £\lûT ùTtßs[Õ.
A«Wm GdLo ¨Xj§p BWiPôYÕ WôdùLh zz ©WRôu Uk§¬ _u AúWôdVô §hPjûR
GÜR[m @ûUdLlTÓm F] BvúWô ùNVpTÓj§V Uô¨XeL°p £\kR Uô¨XUôL
ùR¬®jÕs[Õ. Ï_Wôj Uô¨Xm @±®dLlThÓs[Õ.
zz Uj§V @WÑ ùY°«hÓs[ Bk§Vô®u zz BvúWp Sôh¥-ÚkÕ ùT\lThP Anti Tank
çnûUVô] SLWm Th¥V-p ØR- Guided Missiles Spike (ATGMS) Fàm ÀWe¡
PjûR ùRôPokÕ 4#YÕ Øû\VôL Uj§V F§ol× GÜLûQLs Bk§V WôÔYj§p
©WúRN Uô¨Xm Bkço SLWm ùTtßs[Õ. BûQjÕd ùLôs[lThP].
BkRlTh¥V-p úUtÏ YeL Uô¨Xm zz ÏZkûRLû[ Ts°dÏ @àl×m RôndÏ
ùLôpLjRô LûP£ BPj§p Ds[Õ. AiÓdÏ ì.15,000 YZeÏm RônU¥
zz Sôh¥u ØRXôYÕ úR£V ¾VûQl×f (@mU ùYô¥) §hPjûR Ak§W ©WúRN Uô¨X
úNûYLs Lpí¬ ULôWôx¥Wô Uô¨Xm @WÑ ùRôPe¡Ùs[Õ. BRu êXm 42 XhNm
Sôdé¬p ùRôPeLlThPÕ. úR£V ¾VûQl× ÏÓmTeLû[f úNokR 82 XhNm UôQY
úNûYLs Lpí¬ Uj§V DsÕû\ UôQ®Ls TVu @ûPV Ds[]o.
@ûUfNLj§u ¸r ùNVpTÓm. zz úUtÏ YeLô[j§p @ûUkÕs[ ùLôpLjRô
zz Ï_Wôj ØRpYo ®_n ìTô² 2#YÕ Õû\ØLm, B² £VôUô ©WNôj ØLo´ Õû\
DVWUô] NoRôo YpXTôn TúP-u £ûXûV ØLm F] @ûZdLlTÓm F] Uj§V @WÑ
@LURôTôj§p §\kÕ ûYjRôo. @±®jÕs[Õ.
zz @W£u @û]jÕ UhPeL°Ûm LôQlTÓm zz H.Sô. Y[of£ §hPj§u êXm ØÓd¡
EZûX Tt± AWôV Ak§W Uô¨X @WÑ AnYLm ×ÕùPp-«p §\dLlThPÕ. BkR
Utßm Bk§V úUXôiûU Lp® ¨ßY]m, AnYLj§u Ød¡V ùNVpTôÓ Bk§Vô®p
@LURôTôj BûPúV ׬kÕQoÜ IlTkRm UôÑTôhûP F§oùLôsYRtLô] Y¯Øû\
úUtùLôs[lThÓs[Õ. Lû[ AWônYRôL BÚdÏm.

 A-Series in
          ©lWY¬ 2020
NoYúRNm GÜLûQûV Tô¡vRôu SôÓ ùYt±LWUôL
úNô§jÕs[Õ.
zz 25#YÕ TÚY¨ûX Uôt\m ùRôPoTô] zz DXL ÑLôRôW ¨ßY]j§]ôp @e¸L¬dLl
H.Sô.®u LhPûUl× UôSôÓ vùT«u Sôh¥u ThP KûPTônÓ RÓlé£Vô] (Typhoid
RûXSLWô] Uôh¬h SL¬p SûPùTt\Õ. conjugate vaccine (TCV)H SôÓ ØÝYÕm
BRû] £- SôÓ RûXûUúVtß SPj§VÕ. @±ØLlTÓj§Ùs[ DX¡u ØRp SôÓ Fàm
zz Ùù]vúLô DXL TôWmT¬V ÏÝ®u Dßl©] ùTÚûUûV Tô¡vRôu ùTtßs[Õ.
WôL Nܧ @úW©Vô SôÓ ØRpØû\VôL úRoÜ zz 2020#Am Ai¥p DX¡u ªL úYLUôL
ùNnVlThÓs[Õ. Y[Úm ùTôÚ[ôRôWjûR ùLôiP SôPôL
zz 40#YÕ Ùù]vúLô ùTôÕ UôSôÓ ©Wôuv (86%) LVô]ô SôÓ BÚdÏm F] NoYúRN
RûXSLo Tôãp SûPùTt\Õ. SôQV ¨§Vm L¦jÕs[Õ.

zz BkÕ úLô®pLs Ds[ 3,000 AiÓLs zz DdûWu Y¯VôL HúWôl©V SôÓLÞdÏ


TZûUVô] SLWm Tô¡vRô²u ûLTo ÏZôn êXm BVtûL F¬YôÙûY ùLôiÓ
TdÕudYô UôLôQj§u vYôh UôYhPj§p ùNpYRtLô] IlTkRj§p WxVôÜm,
LiÓ©¥dLlThÓs[Õ. BeÏ SôQVeLs, DdûWàm ûLùVÝj§hÓs[].
vçTm, Tôû]Ls Utßm @kRd LôXj§u zz DX¡p NêL YûXR[j§p @§L úSWjûR
AÙReLs Ds[]. ùNX®ÓY§p ©-lûTuv ØR-Pj§p
Ds[Õ F] An®p ùR¬®dLlThÓs[Õ.
zz 2040#Am Ai¥tÏs, DXL[®p 3142
31 SôÓLs BPmùTtßs[ BkR Th¥V-p
´Lô Yôh ã¬V ªuNd§ûV DtTj§ ùNnÙm
Bk§Vô 7#YÕ BPj§p Ds[Õ.
§\û] FhP úYiÓm F] NoYúRN
At\p ØLûU ùY°«hÓs[ @±dûL«p zz HúWôl©V ë²V²p BÚkÕ ©¬hPu
ùR¬®dLlThÓs[Õ. ùY°úVßYRtÏ Y¯YûL ùNnÙm ©ùWd³h
UúNôRôÜdÏ ©¬hPu SôPôÞUu\m Il×Rp
zz º]ô R]Õ ªLlùT¬V ùNVtûLdúLôs @°jRÕ. BRu êXm, 2020 _]Y¬ 31#Am
Rôe¡ ®iLX]ô] Xôe Uôof#5 H ùLôiÓ úR§dÏs HúWôl©V ë²V²p BÚkÕ
µ´Vu#20 Fàm @§SÅ] RLYp ùRôPo× ©¬hPu ùY°úVßYÕ DߧVô¡Ùs[Õ.
ùNVtûLdúLôû[ ùYufNôe ®iùY°
GÜR[j§-ÚkÕ ùYt±LWUôL G®VÕ. BkR zz @ùU¬dL ®iùY° TôÕLôl× TûPl©¬Ü
®iLX]ô]Õ º]ô®u ªLÜm FûPÙs[ 70 AiÓLÞdÏ ©\Ï ùRôPeLlThÓs[Õ.
Utßm ªLÜm úUmThP RLYp ùRôPo× @ùU¬dL ®Uô]l TûP«u ¸r BlTûPl
ùNVtûLdúLô[ôÏm. ©¬Ü ùNVpTÓm.

zz DX¡u ØRp LôhÓ SLWm ùRuº]ô®p Ds[ zz FhÓ úUtÏ Al©¬dL SôÓLs ùT²u,
ÏYôeµ UûXlTϧL°p @ûUkÕs[ ×o¡]ô TôúNô, ¡²Vô – ©Nôq, HY¬ úLôvh,
ûXùN[ SLWj§p @ûUdLlTPÜs[Õ. 175 Uô-, ûS_o, ùN]Lp Utßm úPôúLô ReL[Õ
ùadúPo TWlT[®p TWkÕ ®¬kÕs[ BkR ùTôÕYô] SôQVj§u ùTVûW CdúLô (Eco)
SLWj§u LhPûUl× Ø¥kÕ®hPôp ÑUôo 30 F] Uôt\ Il×d ùLôiÓs[].
A«Wm úTo BeúL Y£dLXôm. zz ùUd£úLô®p 1000 AiÓLs TZûUVô],
UôVu SôL¬LjûR úNokR TZmùTÚm
zz A d v ú T ô o Ó @ L W ô § « u 2 0 1 9 # A m
Uô°ûL Iu±u B¥TôÓLû[ ùRôpùTôÚs
Ai¥tLô] YôojûRVôL TÚY¨ûX
AWônf£Vô[oLs LiÓ©¥jÕs[]o.
@YNW¨ûX (Climate emergency) FuTÕ
Uô°ûL Fuß LÚRlTÓm @dLh¥Pm 20
@±®dLlThÓs[Õ.
@¥ DVWØm, 55 ÁhPo ¿[Øm, 15 ÁhPo
zz `ô¸u#1 Fu\ ùTV¬p ¨Xj§-ÚkÕ 650 @LXØm ùLôi¥ÚkRÕ.
¡.Á. ¨Xj§Ûs[ BXdûL RôdL YpX
zz @ùU¬dLôûY úNokR ¡±v¼]ô úLôf Fàm

©lWY¬ 2020 A-Series in


  
®iùY° ÅWôeLû] NoYúRN ®iùY° @±®Vp#ùRô¯pÖhTm
AWônf£ ûUVj§p 288 SôhLs Re¡ BÚkÕ
NôRû] TûPjÕs[ôo. zz @ùU¬dLô®u ¨ë ùUd£úLô®p Ds[
zz ùTn´e¡-ÚkÕ Nôe f£VôLôqÜdÏf ùNpÛm Yôù]ô- ùRôûXúSôd¡ûV TVuTÓj§
DVoúYL BÚl×l TôûR @§LôWléoYUôLl £X Ïs[ ®iÁu §WsL°p 13 ùTÚm
úTôdÏYWjÕ úNûYdÏ YkÕs[Õ. LÚkÕû[Lû[ AWônf£Vô[oLs
LiÓ©¥jÕs[]o. BkR ®iÁu §WsLs
zz BXeûL«u VôrlTôQm TXô- NoYúRN 骫-ÚkÕ TX I°VôiÓLs ùRôûX®p
®Uô] ¨ûXVjûRl úTôX UhPdL[l× @ûUkÕs[].
®Uô] ¨ûXVjûRÙm ®¬ÜTÓjR Bk§Vô
ØuYkÕs[Õ. zz BWiÓ ¨ëhWôu ShNj§WeLs úUô§
Iu\ô]úTôÕ, LôXùY°«p GtThP ×®Âol×
zz 2.7 ©p-Vu PôXo U§l×s[ 12 FKl #35© @ûXLû[, @iûU«p ®gOô²Ls
úTôo ®Uô]eLû[ £eLléÚdÏ ®tTû] T§Ü ùNnÕs[]o. @ùU¬dLô®p Ds[,
ùNnYRtÏ @ùU¬dL ùY°Ù\ÜjÕû\ -®evPu -úLô AWônf£ ¨ûXVm, BkR
Il×Rp @°jÕs[Õ. ×®Âol× @ûXLû[ LPkR GlWp 2019#Bp
zz × ú [ ô W u v û S h ¥ e ú L - u 2 0 0 # Y Õ T§Ü ùNnÕs[Õ.
©\kRSôû[ Øu²hÓ, DXL ÑLôRôW zz LPpYôr D«¬]Uô] LQYôn ÁuLÞdÏ
@ûUl× 2020#Am AiûP ùN®-Vo U²RoLû[ úTôuß ØlT¬UôQ TPeLû[
Utßm UÚjÕYf£ AiÓ (Year of Nurse TôojR±Ùm Ti× Ds[RôL @ùU¬dL
and Midwife) Fuß ùTV¬hPÕ. AnYô[oLs LiP±kÕs[]o. LQYôn
zz CWôu SôPô]Õ JCPOA (®¬Yô] áhÓ ùNVp ÁuLÞdÏ ØlT¬UôQ LiQô¥Lû[
§hPm (Joint Comprehensive Plan of Action) @¦®jÕ IÚ LôùQô°ûV TôodL
@ÔNd§ IlTkRj§-ÚkÕ Øt±ÛUôL ûYjR]o. @kR LôùQô° @Yt±tÏ
®X¡Ùs[Õ. CWôu Utßm ©5, ù_oU² ©¥jRUô] B\ôp ÁuLs ¿kÕYÕ úTôuß
& HúWôl©V Iu±Vm A¡VYt±tÏ BûPúV DÚYôdLlTh¥ÚkRÕ.
2015#Am AiÓ ®¬Yô] áhÓ ùNVp zz X # 57 Maxwell F]l ùTV¬PlThÓs[
§hPm F] @ûZdLlTÓm CWôu @ÔNd§ SôNô®u ØRXôYÕ ØÝYÕm ªuNôWj
IlTkRm ûLùVÝjRô]Õ. §]ôp BVdLlTÓm ®Uô]m @±ØLl
zz Lp®, YôrdûLjRWm, TôÕLôl×, ùTôÚ[ôRôWm TÓjRlThÓs[Õ.
úTôu\ @mNeLû[ ØuûYjÕ, zz 2020#Am AiÓ SYmTo UôRm Nk§WVôu#3
RûX£\kR SôÓL°u Th¥VûX @ùU¬dL ®iLXjûR ¨XÜdÏ @àlT Ds[RôL
ùNn§ Utßm DXL @±dûL Fu\ Tj§¬dûL BvúWô @±®jÕs[Õ.
ùY°«hÓs[Õ. B§p @û]jÕ ®Rj§Ûm
£\kR SôÓLs Th¥V-p ØR-PjûR zz Lôt±-ÚkÕ LôoTuûP AdûNûP R²VôL
Ñ®hNoXôkÕm, BWiPôªPjûR L]PôÜm, ©¬jÕ D±gÑm faradaic electro # swing
êu\ôªPjûR _lTôàm ùTtßs[]. reactive adsorption system Fàm LÚ®ûV
@ùU¬dLô®u Fm.H.¥. Lp® ¨ßY]
zz DX¡u BÚ ùTÚm ùTôÚ[ôRôW SôÓL[ô] AWônf£Vô[oLs DÚYôd¡Ùs[]o.
@ùU¬dLôÜdÏm, º]ôÜdÏm BûPúV ÑUôo 2
AiÓL[ôL YojRL úTôo SPkÕ YkRÕ. BkR zz U²R DQoÜLû[ LiP±Ùm ùNVtûL
¨ûX«p @ùU¬dLô Utßm º]ô BûP«Xô] úRôûX, ©¬hP²p Ds[ ©¬vPp
ØRtLhP IlTkRm ûLùVÝjRô]Õ. º]ô®u TpLûXdLZL AWônf£Vô[oLs
DVoUhP ©W§¨§Ls Øu²ûX«p BkR LiÓ©¥jÕs[]o. BkR ùNVtûL úRôp,
IlTkRj§p @ùU¬dL _]ô§T§ ¥Wml # º] £-dLôu êXm DÚYôdLlThÓs[Õ.
ÕûQ ÄÙ ¶ A¡úVôo ûLùVÝj§hP]o. zz NaôWu £pYo Fu\ ùTV¬Xô] DX¡p

 A-Series in
          ©lWY¬ 2020
ªL úYLUô] Fßm× YûL Õ²µVô F] @ûZdLlTÓm BÕ éª«u ¨û\ûV
Sôh¥p, ù_oU²V AWônf£Vô[oL[ôp ®P 10 UPeÏ @§L ¨û\ÙûPVÕ.
LiÓ©¥dLlThÓs[Õ. zz 50 AiÓLÞdÏ Øu]o 骫p ®ÝkR
zz ùSRoXôkûRf úNokR, áp Bu©²¥ RtúTôÕ, ®iLpÛdÏs T¥kÕs[ ShNj§Wj ÕLs,
Hv úYôoh 300 Fu\ קV Ï°oTR]l 700 úLô¥ AiÓLÞdÏ Øu]o DÚYô]
ùTh¥ûV DÚYôd¡Ùs[Õ. BRtÏ, Aß RôL @ùU¬dL ®gOô²Ls SÅ] LÚ®Ls
U¦ úSWm ªuNôWm BÚkRôp úTôÕm. @kR êXm RtúTôÕ LiP±kÕs[]o. SUÕ
úSWj§tÏs, ùTh¥ûVf Ñt± ¨WlTlThÓs[ TôpùY°UiPXm FqYôß DÚYô]Õ
NôRôWQ ¿ûW Hv Lh¥L[ôL Uôt± ®Óm. FuTûR @kRShNj§Wj ÕLs êXm
zz ùNqYôn ¡WLj§p éªûV úTôXúY Dl× @±kÕ ùLôs[ Ø¥Ùm Fußm @YoLs
G¬ Ds[ûR @ùU¬dL ®gOô²Ls á±Ùs[]o.
LiÓ©¥jÕs[]o.
zz DF#41 Fàm ùTV¬p DX¡p ªL Y-ûU
®û[VôhÓLs
YônkR LiPm ®hÓ LiPm Rôi¥ RôdÏRp zz WxV RûXSLo UôvúLô®p DXL úW©h
ùNnÙm GÜLûQûV º]ô ùYt±LWUôL ùNv Nôm©Vuµl ùRôPo SûPùTt\Õ. B§p
T¬úNô§jÕs[Õ. ùTiLs ©¬®p Bk§Vô®u ùLôú]Ú
zz L P p © [ ô v ¥ d û L @ L t \ × § V am©, º]ô®u Ä ¥eËûV Årj§ Nôm©Vu
ùRô¯pÖhTjûR ùSRoXôkÕ ®gOô²Ls ThPm ùYu\ôo.
LiÓ©¥jÕs[]o. BYoLs LP-p Ds[ zz ULôWôx¥W Uô¨Xm, ¨ëTuúYp SL¬p
©[ôv¥d ÏlûTLû[ @LtßYRtÏ SûPùTt\ Ao.Ao.XbVô úLôlûT Go
SLÚm Yû[Vm úTôu\ ùRô¯pÖhTjûR ûWK©s ÕlTôd¡ ÑÓRp úTôh¥«p 10 ÁhPo
DÚYôd¡Ùs[]o. º²Vo ©¬®p HvY¬ ©WRôl £e úRôUo
zz Av§úW-Vô Utßm _lTô²V ®gOô²Ls ReLm ùYu\ôo.
@û\ ùYlTj§p BWiÓ SôhLÞdÏ zz LjRôo RûXSLo úRôLô®p ¡[l @¦LÞd
úUp RôdÏl ©¥dÏm BuÑ-u §WYjûR ¡ûPúVVô] SûPùTt\ ©Tô DXL úLôlûT
DÚYôd¡Ùs[]o. úUÛm, @jRûLV LôpTkÕ ùRôP¬u Bߧ AhPj§p -Yoép
BuÑ-û] úLô¯ ØhûP«-ÚkúR RVô¬dL @¦, 1#0 Fu\ úLôp LQd¡p ©[ªeúLô
Ø¥Ùm FuTûRÙm @YoLs LiP±kÕ @¦ûV Årj§ ØRpØû\VôL Nôm©Vu
Ds[]o. ThPm ùYu\Õ.
zz SôNô Lôtß ®iùY°ûV Nk§dÏm BPjûR
zz Be¡Xôk§p SûPùTt\ 95#YÕ úav¥ev
AWôV HLôu Fu\ ùNVtûLdúLôû[
NoYúRN ùNv úTôh¥«p 9 ÑtßLs ùLôiP
G®Ùs[Õ. Ï°oNôR] ùTh¥ @[®Xô]
ùRôP¬p Bk§Vô®u ©.UúLx Nk§Wu 7.5
BkR ùNVtûLdúLôs @Vú]ôv©V¬p Ds[
×s°Ls ùTtß Nôm©Vu ThPm ùYu\ôo.
YôÙdL°-ÚkÕ DÚYôÏm Lôtú\ôhPjûR
Tt± AnÜ ùNnÙm. zz Bk§Vô®p ÑtßlTVQm úUtùLôiP
BXeûL ¡¬dùLh @¦ êuß úTôh¥Ls
zz DP-p Tôd¼¬Vô TWÜRûX LiP±Ùm
ùLôiP ¥20 ùRôP¬p TeúLtß
Ïû\kR ùNX®Xô] F°V ûLVPdL LÚ®ûV,
®û[Vô¥VÕ. B§p ØRp úTôh¥ UûZVôp
LÜLôj§«Ûs[ Bk§V ùRô¯pÖhT
WjÕ ùNnVlThPÕ. @ûR ùRôPokÕ SPkR
Lp® ¨ßY]j§u AWônf£Vô[oLs
BWiPôYÕ Utßm êu\ôYÕ úTôh¥L°p
DÚYôd¡Ùs[]o.
Bk§V @¦ ùYt± ùTtß ùRôPûW
zz ã¬V ÏÓmTj§tÏ @lTôp Ds[ Lô£úVô¥Vô ûLlTt±VÕ.
®iÁu ÏÝ®p IÚ ùY°dúLôû[
zz ¨ë³Xôk§u AdXôk§p ùTiLÞdLô]
®gOô²Ls LiÓ©¥jÕs[]o. ãlTo Foj
AdXôkÕ ¡[ô£d ùPu²v ùRôPo

©lWY¬ 2020 A-Series in


  
SûPùTt\Õ. BRu Bߧ AhPj§p 2019 º¬ G Nôm©Vu ThPjûR _ýùYuPv
@ùU¬dLô®u ùNÃ]ô ®p-Vmv, NL @¦ ùYpX DR®]ôo. úUÛm úS`uv
SôhÓ ÅWôeLû] ù_£Lô ùTÏXôûY Årj§ Äd ThPjûR úTôofÑdLp ùYpXÜm
Nôm©Vu ThPm ùYu\ôo. ùWô]ôpúPô LôWQUôL BÚkRôo.
zz Av§úW-Vô®p ÑtßlTVQm úUtùLôiP zz ûaRWôTôj§p SûPùTt\ 34#YÕ Bk§V
¨ë£XôkÕ ¡¬dùLh @¦ êuß úTôh¥Ls ùTô±«Vp UôSôh¥p 2019#Am AiÓdLô]
ùLôiP ùPvh ùRôP¬p TeúLtß úR£V Y¥YûUl× Utßm AWônf£
®û[Vô¥VÕ. B§p Av§úW-Vô @¦ ¨ßY]j§u úR£V Y¥YûUl× ®ÚÕLs
3#0 Fu\ LQd¡p ùRôPûW ûLlTt±VÕ. YZeLlThP]. B§p ¥.A.¥.J. ¨ßY]j§u
zz Av§úW-Vô®p @¦LÞd¡ûPúVVô] Y[eLs#úUXôiûU ùTôÕ BVdÏSo £jWô
G¥© úLôlûT ùPu²v ùRôPo SûPùTt\Õ. Wô_úLôTôÛdÏ ùTi @±®VXô[ÚdLô]
BRu Bߧ úTôh¥«p ùNo©V @¦, 2#1 ÑUu NoUô ®ÚÕ YZeLlThPÕ.
Fu\ LQd¡p vùT«u @¦ûV Årj§ §ÚY]kR×Wj§p Ds[ BvúWô#§WY
Nôm©Vu ThPm ùYu\Õ. DkÕNd§ ûUV BVdÏSo ®gOô²
SôWôVQàdÏ úR£V Y¥YûUl× ®ÚÕ
zz NoYúRN ÕlTôd¡ ÑÓRp áhPûUl×
YZeLlThPÕ.
2019#Am AiÓdLô] RWY¬ûN Th¥VûX
ùY°«hÓs[Õ. B§p NoYúRN @[®p 21 zz Øu]¦ Bk§ Sô°RZô] _dWu RûXûU
ReLeLs, 6 ùYs°Ls Utßm 3 ©Wôuv A£¬Vo NgNn ÏlRôÜdÏ úXôdUôuV
úLôlûTLÞPu Bk§V @¦ ØR-PjûR §XLo úR£V Tj§¬ûL ®ÚÕ YZeLlThPÕ.
ùTtßs[Õ. Tj§¬dûL Õû\«p @Yo At±V £\lTô]
T¦dLôL BkR ®ÚÕ ùT\ @Yo úRoÜ
zz Bk§V ÏjÕfNiûP Äd úTôh¥«u ØRp
ùNnVlThPôo.
Nôm©Vu ThPjûR Ï_Wôj ù_«uhv @¦
ûLlTt±VÕ. zz BXd¡Vj§p £\kR TeL°lûT At±VRtLôL
zz úTôTô-p SPkR 63#YÕ úR£V ÕlTôd¡ ÑÓRp YZeLlTÓm ®ÚRô] ùL[NªLWôw
Nôm©Vuµl úTôh¥«p a¬Vô]ôûY NôokR úR£V ®ÚRô]Õ ùYj WÍ FuTYÚdÏ
YVRô] Uà TôdLo ùTiLÞdLô] 10 ÁhPo YZeLlThÓs[Õ.
Go ©vP-p BWhûP ReLl TRdLeLû[ zz EPL ¨ßY]eLÞdLô] NoYúRN úVôLô
ùYu\ôo. R²STo UhÓªu± @¦Ls §] ®ÚÕ ùNuû] çoRo`u, Rk§
©¬®Ûm Uà TôdLÚdÏ ReLlTRdLm ÏÝUjûR úNokR aúXô Fl.Fm. Yôù]ô-
¡ûPjRÕ. A¡VYt±tÏ YZeLlThÓs[]. Bq®ÚÕ
zz WµVô RûXSLo UôvúLô®p SûPùTt\ DXL ùUôjRm 30 EPL ¨ßY]eLÞdÏ
©°hv ùNv úTôh¥ Uôd]v Lôop^u YZeLlThPÕ Ï±l©PjRdLÕ.
Nôm©Vu ThPm ùYu\ôo. UL°o ©¬®p zz H.Sô. NûT«u DXL LôX¨ûX Uôt\j§t
WµV ÅWôeLû] Xôdú]ô LhúWV]ô ùL§Wô] ùNVpTôÓLÞdLô] ®ÚÕ
Nôm©Vu ThPm ùYu\ôo. B k §Vô û Y Rû X û U «P U ô L ùL ô iP
zz 23 YVÕ ©¬Ü NoYúRN úP©s ùPu²v BuúTô£v ¨ßY]j§tÏ YZeLlThÓs[Õ.
RW Y¬ûN«p Bk§Vô®u B[m ÅWo zz TÑûU ×Wh£«u RkûR Fu\ûZdLlTÓm
Uô]q RôdLo ØR-PjûR ùTtßs[ôo. Fm.Fv. ÑYôªSôRàdÏ ØlTYWl×

®ÚÕLs
ùYeûLV SôÙÓ £\l× úR£V ®ÚÕ
YZeLlThÓs[Õ. Fm.Fv. ÑYôªSôRu
zz _ýùYuPv @¦ ÅWo ¡±v¥Vôú]ô ®YNôVjÕû\«p At±V TeL°l×dLôL
ùWô]ôpúPô 2019#Am AiÓdLô] ÕûT @YÚdÏ BkR ®ÚÕ YZeLlThPÕ.
Ïú[ôl NôdLôo ®ÚûR ùYußs[ôo. LPkR zz ©££H NôoTôL AiÓúRôßm NoYúRN

 A-Series in
          ©lWY¬ 2020
¡¬dùLh¥p £\kÕ ®[eÏm Bk§V ÅWo,
ÅWôeLû]LÞdÏ ®ÚÕLs YZeLlThÓ
¨VU]eLs
YÚ¡\Õ. Bk¨ûX«p ©££H«u DV¬V zz ©WRUo RûXûU«p SûPùTt\ Uj§V
®ÚRô] Tô- Dm¬Lo ®ÚÕ Bk§V @¦«u @ûUfNWûY áhPj§p, RûWlTûP,
úYLlTkÕ ÅfNô[o _v©¬h ×mWôÜdÏ LPtTûP Utßm ®Uô]lTûPdÏm úNojÕ
YZeLlThPÕ. Bk§V UL°o @¦«p IúW RûXûU R[T§ûV ¨VªdL Il×Rp
Ød¡V TkÕÅfNô[WôL Ds[ é]m VôRÜdÏ @°dLlThPÕ. BûRVÓjÕ, WôÔY R[T§
£\kR ÅWôeLû] ®ÚÕ YZeLlThPÕ.. ©©u WôYjûR, ØlTûPLÞdLô] ØRp
zz L]Pô®u §ûWlTPm Utßm ùRôûXdLôh£ RûXûU R[T§VôL Uj§V @WÑ ¨VªjRÕ.
¨ßY]jRôp YZeLlTÓm YôrSôs NôRû] zz @ùU¬dLô®Ûs[ ¨ëVôod ùUhúWô
Vô[o ®ÚÕdÏ Bk§V YmNôY°ûV Tô-hPu LûXd LiLôh£VLj§u
úNokR L]Pô §ûWlTPj RVô¬lTô[o ¾Tô Bk§VôûY úNokR ØRp @\eLôYXWôL ¨Rô
úUjRôÜdÏ YZeLlThÓs[Õ. @mTô² ¨VªdLlThÓs[ôo.
zz FuFp£ Bk§V ¨ßY]j§tÏ @¡X zz D X ¡ u ª L l ù T ¬ V Õ û Q W ô Ô Y l
Bk§V UdLs ùRôPo×j Õû\ Nôo©p 2 TûPVô] £Ao©FKl#Bu RûXYWôL
úR£V ®ÚÕLs YZeLlThÓs[Õ. £\kR G . © . U ú L x Y ¬ û V Uj§V T¦Vô[o
NêLlùTôßl×QoÜPu §LÝm ¨ßY]j @ûUfNLm ¨VªjÕs[Õ.
ÕdLô] ®ÚÕ Utßm RLYp @±Ùm zz W µ V ô ® u × § V © W R U W ô L ª d û L p
D¬ûUfNhPjûR £\lTôL @UpTÓj§ YÚm ª`þv¥u ¨VªdLlThÓs[ôo. BYo
®ÚÕ A¡V ®ÚÕLs YZeLlThP]. RtúTôÕ ùTPWp Y¬ úNûY«u RûXYWôL
zz ÕTô«p Ds[ Bk§Vu úUp¨ûXl Ts°«p TR® Y¡jÕ YÚ¡\ôo.
T¥dÏm £ßª Ñ´Rô N¾x 120 ùUô¯L°p zz ¬ N o q Y e ¡ « u S ô u L ô Y Õ Õ û Q
TôhÓ Tô¥ NôRû] TûPjRRtLôL. 100 Ïú[ôTp AÞSWôL ûUdúLp úRYlWRô TjWô
ÏZkûR úUûR ®ÚÕ YZeLlThÓs[Õ. ¨VªdLlThÓs[ôo. BYo êu\ôiÓ LôXm
BYo 12 YV§p 102 ùUô¯ TôPpLû[ 15 U¦ BlTR®ûV Y¡lTôo.
úSWjÕdÏ úUXôL Tô¥Ùm BÚ DXL NôRû]
Lû[ TûPjRYo FuTÕ Ï±l©PjRdLÕ. zz £.Ao.©.Fl RûXYWôL BkúRô ¾ùTj FpûX
LôYp TûP RûXYo Fv.Fv úRvYôÛdÏ
zz ©¬dv B[m LiÓ©¥lTô[o ®ÚÕ 2019, ÀLôûW áÓRp ùTôßl× YZeLlThÓs[Õ.
úNokR W® ©WLôxdÏ YZeLlThÓs[Õ.
ªLd Ïû\kR ùNX®Xô] TôûX Ï°oîhÓm zz úPôeLô Sôh¥tLô] Bk§Vô®u קV
LÚ®ûV LiÓ©¥jRRtLôL BkR ®ÚÕ ûa Lª`]WôL TjU_ô ¨VªdLlThÓs[ôo.
YZeLlThÓs[Õ. zz H.Sô. DXL ÑtßXô ¨ßY]j§u BVdÏ]WôL
zz LoSôPL TôP¡ Fv.ùN[ªVôÜdÏ Ne¸R Bk§VôûY úNokR H.G.Fv @§Lô¬ ÑUu
LXô¨§ ®ÚÕ YZeLlThÓs[Õ. BkR ©pXô ¨VªdLlThÓs[ôo.
®ÚRô]Õ AiÓúRôßm LoSôPL BûN zz ©WTX úY§«VXô[o ©×p ùTaô¬ Naô
LûXOÚdÏ YZeLlTÓm Io ®ÚRôÏm. 2020#23 LôXLhPj§tLô] Tu]ôhÓ
zz Am]v¥ BiPoúS`]p @ûUl©u R² Utßm TVuTôhÓ úY§«Vp Iu±Vj
U]Nôh£«u çÕYo ®ÚÕ 2019 ÑÅPu § u (International Union of Pure and
SôhûP úNokR ÑtßfãZp AoYXWô] Applied Chemistry (IUPAC)) T¦VL Dßl
¡úWhPô ÕuTod¡tÏ YZeLlThPÕ. ©]WôL úRokùRÓdLlThPôo. £.Fu.
zz ÕÚd¡ Sôh¥u NoYúRN ¡WôkR ¥e ®ÚÕ Ao.WôqdÏ ©\Ï IÚ èt\ôi¥p
2019 úULôXôVôûY úNokR U²R D¬ûU BkR TR®dÏ úRokùRÓdLlThP
AoYXo Ad]v Lôoµe¡tÏ YZeLl BWiPôYÕ Bk§Vo BYo FuTÕ Ï±l©Pj
ThÓs[Õ. RdLÕ.

©lWY¬ 2020 A-Series in


  
Ød¡V SPl× ¨LrÜLs
PV¬d ϱl×Ls # 2019
(_]Y¬ 2019 ØRp ¥NmTo 2019 YûW)

@±®Vp#ùRô¯pÖhTm ©WôuVôdLû[ úTôX áoûUVô] TtLs


BRtÏ BÚd¡\Õ.
_]Y¬ zz Bk§Vô®p ØRpØû\VôL TúVô ù_h
F¬ùTôÚs T V u T Ó j R l T h Ó W ô Ô Y
zz ùNqYôn ¡WLj§u úLôúWôúXôq Ts[j§p ®Uô]eLs ùYt±LWUôL T¬úNô§dLl
ØÝYÕUôL T² ¨û\k§ÚdÏm ThÓs[Õ. BkR D«¬ F¬ùTôÚ[ô]Õ
×ûLlTPùUôuß RtúTôÕ ùY°YkÕs[Õ. _ôhúWôTô FiûQ«-ÚkÕ RVô¬dLl
HúWôl©V ®iùY° ûUVj§u Uôov TÓ¡\Õ.
Fdv©Wv ª`u Bl×ûLlTPeLû[
zz @ùU¬dLô®u SôNô ®iùY° AnÜ ûUVm
FÓjÕs[Õ. úLôúWôúXôq Ts[m ùNqYôn
ùNqYôn ¡WLj§p קV úWôúTô Iuû\
¡WLj§u YPdÏ Rôr¨XlTϧ«p Ds[Õ
RûW«\d¡Ùs[Õ. § BuûNh F]lTÓm
LiP±VlThÓs[Õ. BlùTÚm Ts[m
@kR úWôúTô ùNqYôn ¡WLj§u AZUô]
T²Vôp ¨û\kRRpX, T²dLh¥L[ôp
Utßm Ds TϧLs ϱjÕ AWônf£ ùNnÙm
¨û\kRÕ.
úSôdLjÕPu @àlTlThÓs[Õ.
zz ùTuà ÏßeúLô°p Ri½o BÚlT
zz BÕYûW éªûV úSôd¡ LôQlTPôR
RtLô] @±Ï±Lû[ SôNô®u OSIRIS#REx
¨X®u Utù\ôÚ TϧVôL LÚRlTÓm
Spacecraft ®iLXm LiÓ©¥jÕs[Õ. BeÏ
úYôu LoUôu Fuàm Tϧ«p º]ô®u
Ad£_u Utßm ûahW_u @ÔdLs
Nôe B#4 ùNVtûLdúLôs RûW«\e¡VÕ.
BûQkÕ ûahúWôd£p Y¥®p Ri½o
¨X®u UßTdLjûR AnÜ ùNnYRtLôL
LôQlTÓYRôL LiP±VlThÓs[Õ.
@àlTlThP ®iLXjûR, LWÓØWPô]
zz Vespucci US Military Satellite Fu\ ùTV¬p Tϧ«p ùYt±LWUôL RûW«\d¡ º]ô
´.©.Fv#3 ùNVtûLdúLôû[ @ùU¬dLô®u NôRû] TûPjÕs[Õ.
vúTv Fdv ¨ßY]m ùYt±LWUôL ®i¦p
zz ã¬V ÏÓmTj§tÏ @lTôp Ds[ Lô£úVô¥Vô
ùNÛj§Ùs[Õ.
®iÁu ÏÝ®p IÚ ùY°dúLôû[
zz v ù T « ² Û s [ , S Y ô ú W ù T ô Õ l AnYô[oLs LiP±kÕs[]o. ãlTo Foj
TpLûXdLZLm Utßm ©¬hP²Ûs[, F] @ûZdLlTÓm BÕ éª«u ¨û\ûV
©¬vPôp TpLûXdLZLm A¡VYtû\ ®P 5 UPeÏ @§L ¨û\ÙûPVÕ.
úNokR ®gOô²Ls, I-d ¡Ód¡ûV
zz @ùU¬dLô®u éoÅL ùRôp-Vp R[j§p
DÚYôd¡Ùs[]o.
1000 AiÓLÞdÏ ØkûRV DX¡u ªLl
zz ©WôuVô úTôu\ IÚ Á²u ×ûRdLlThP TZûUVô] RôYW ûYWv YûLûV úNokR
Fg£Ùs[ ×ûRT¥Ym Iuû\ ®gOô²Ls ËVô úU dûWúNôûYWv 1 F] ùTV¬PlThP
LiÓ©¥jÕs[]o. BÕúY TZûUVô] ûYWû^ AWônf£Vô[oLs LiP±kÕs[]o.
Bû\f£ Nôl©Óm Áu Fuß ùR¬VYkÕs[Õ. BRtÏ Øu]o LiP±VlThP TZûUVô]
150 ªp-Vu AiÓLÞdÏ Øu× YôrkR ûYW^ô]Õ 750 YÚPj§tÏ ØkûRVRôÏm.
BkR D«¬]j§u FÛm× ùRtÏ ù_oU²«p
zz º]ô UAV# CH7 Fu\ ùTV¬p @§SÅ]
LiÓ©¥dLlThÓs[Õ. RtúTôÕs[
A°pXô ®Uô]jûR RVô¬jÕs[Õ. BkR

 A-Series in
          ©lWY¬ 2020
A°pXô ®Uô]j§u êXm AÙReLû[ LiÓ©¥jÕs[]o. BkR UÚkÕL[ôp
ÑUkÕ ùNuß RôdL Ø¥Ùm. BRu êXm £uÏuϲVô ûYWv 64 NRÅRm úSôn
@ùU¬dLôûY @ÓjÕ ªL DVWj§p RôdÏm RuûUûV BZlTÕPu, ùNpL°p
T\dLdá¥V Utßm @§SÅ] A°pXô Ds[ Ao.Fu.G ûYWvL°u @[Üm 45
®Uô]jûR RVô¬jÕs[ BWiPôYÕ SôÓ NRÅRm YûW Ïû\kÕs[Õ ùR¬VYkÕs[Õ.
Fàm ùTÚûUûV º]ô ùTtßs[Õ. zz @ûXqúLo Fu\ ùTV¬p UôWûPlûT
zz ã¬V²-ÚkÕ ªL ùRôûX®p BÚlTúRôÓ Øuáh¥úV LiP±Ùm ùUôûTp ùNV-ûV
UhÓUpXôUp ã¬Vd ÏÓmTj§u ªLlTûZV @ùU¬dLô®u BiPoûUÜiûPu BRV
ÖiúLôs @ph¥Uô ÕúX F] ®gOô²Ls AWônf£ ¨ßY]m @±ØLlTÓj§Ùs[Õ.
ùR¬®jÕs[]o. BkR @ph¥Uô ÕúX BkR ùNV- UôWûPl× Ï±jR RLYpLû[
ÖiúLô°u @ÚúL ùNuß @ùU¬dLô®u B£´ BVk§Wm @[®tÏ Õp-VUôL
¨ë aô¬Nu ®iLXm NôRû] TûPjÕs[Õ. RÚ¡u\].
zz DX¡u ØRp UWTÔ §Új§ DÚYôdLl zz ùT¬V ¿Útß úTôu\ קV LÚkÕû[
ThP ÏZkûRLû[ º] AWônf£Vô[o Iuû\ ®iùY° AWônf£Vô[oLs
¶ ´VôuÏ DÚYôd¡Ùs[ôo. LiP±kÕs[]o. BÕ éª«p BÚkÕ
zz K©hí Fu\ ùTV¬p úLuNo Utßm ¿¬¯Ü G\jRôZ IÚ ©p-Vu I° AiÓLÞdÏ
úSônLû[ LiÓ©¥dL á¥V vUôoh @lTôp BÚlTRôL ùR¬®dLlThÓs[Õ.
PônùXhûP @ùU¬dLô®u Fm.H.¥. ¨ßY]j
§u AWônf£Vô[oLs LiÓ©¥jÕs[]o.
©lWY¬
zz C£úTôu vPôo Fu\ ùTV¬p Bk§Vô®u zz ù_oU²«Ûs[ ©WôuúaôTo AWônf£
ØRp ÏZkûRLÞdLô] ùUôûTp úTôû] ¨ûXVjûRf úNokR ®gOô²Ls, ûLVPdL
BúSôYv FiPo©ûWNv Fu\ ¨ßY]m @Lf£Yl×d L§o LÚ®ûV DÚYôd¡
ùY°«hÓs[Õ. Ds[]o. ªu]Ô D§¬ TôLeLs RVô¬l©p
zz Mohammed VI#B Fu\ ùTV¬p ùUôWôdúLô GtThÓs[ Øuú]t\eLû[ TVuTÓj§
SôÓ R]Õ ×® LiLô¦l× ùNVtûLdúLôû[ DÚYôd¡ BÚlTRôp, BkRd LÚ®ûV, FYÚm
ùYt±LWUôL ®i¦p ùNÛj§VÕ. ûLVô[ Ø¥Ùm.
zz v©uúSdLo Fu\ ùTV¬p, DX¡u ªLlùT¬V zz LoSôPLô®u TôLpúLôh UôYhPj§u
U²R êû[ úTôuß ùNVpTPá¥V ãlTo TôRôª UûXL°-ÚkÕ BWiÓ ×§V
LmlëhPo Be¡Xôk§p UôuùNvPo B]eLû[ RôYW®VXô[oLs LiP±k
TpLûXdLZLj§p DÚYôdLlThÓs[Õ. Õs[]o. BkR B]eLs úTôohÓXôLô
TôRôªLô Utßm úTôohÓXôLô Xxª
zz ùTnúPô Y¯LôhÓm ùRô¯pÖhTjÕdÏ
SW£mUaô²Vô]ô F]l ùTV¬Pl
úUÛm YÛúNodÏm YiQm, BÚ ×§V
ThÓs[].
ùNVtûLdúLôsLû[ º]ô Uôof 38
WôdùLh¥u êXm ùYt±LWUôL ®i¦p zz £d¡ªu AnYô[oLs 120 AiÓLÞdÏ
ùNÛj§Ùs[Õ. ©\Ï LgNu_eLô úR£V éeLô®u Td¡m
zz º]ô®u BVt©Vp AWônf£Vô[oLs 100 Fu\ Tϧ«-ÚkÕ TÝl× ¨\ £±V UW
ªp-Vu ¥¡¬ ùYlTm ùLôiP ùNVtûL YiQjÕléf£ YûLûV LiP±kÕs[]o.
ã¬Vû] DÚYôd¡Ùs[]o. SmØûPV BkR TÝl× ¨\ £±V UW YiQjÕléf£
Yô]j§p Ds[ ã¬V²u ùYlT¨ûX Vô]Õ £d¡ªp ¥ ¨¡®p- Fàm @±®V
15 ªp-Vu ¥¡¬ ùNp£Vv UhÓúU Xô[Wôp ØRuØR-p 1887#Am Ai¥p
ùLôiPRôÏm. ×°VeùLôhûPL°p BÚkÕ LiÓ©¥dLlThPÕ.
zz ×°VeùLôhûP«p BÚkÕ FÓdLlTÓm zz êû[«p ªu @ûXLs RôßUô\ôL GtThÓ
×WRj§p BÚkÕ £dÏuϲVôÜdÏ UÚkÕ Y-l× YÚmúTôÕ, @ûR ºWôdL, Yôih
RVôo ùNnV Ø¥Ùm HH¥ ìoúL úTWô£¬VoLs Fu\ LÚ®ûV, L-úTôo²Vô TpLûXdLZL

©lWY¬ 2020 A-Series in


  
®gOô²Ls DÚYôd¡Ùs[]o. Y-l× úSôhÓLû[ @ûPVô[m LiÓ DeLs
NªdûO YÚmúTôúR, Yôih LÚ®, ªu ûL«p ûYj§ÚlTÕ FkR ìTôn úSôhÓdLs
@ûXLû[, êû[dÏs T§VlThÓs[ ªu FuTÕ Tt±V RLYpLû[ I- Y¥®p
Øû]LÞdÏ @àl×YRôp, úSôVô°dÏ YZeÏm.
Y-lûT DPú] ¨ßj§®P Ø¥Ùm F], zz U²R Y¥YûUl©p BÚkÕ Ntßm Uô\ôR
AWônf£Vô[oLs ùR¬®jÕs[]o. YûL«p ùTi úWôúTôh Iuß DÚYôdLl
zz Av§úW-VôûY úNokR Xô húWôl TÓ¡\Õ. BkR ùTi úWôúTôh¥tÏ HPô F]
TpLûXdLZLjûR úNokR ®gOô²Ls ùTV¬PlThÓs[Õ. BRû] ©¬hPû]f
×ûL«ûX ùN¥«Ûs[ édL°-ÚkÕ ×§V úNokR HùPu ùUpXo DÚYôd¡ YÚ¡u\ôo.
YûL Au¥TVô¥dÏLû[ DÚYôdL Ø¥Ùm zz DX¡u ªLlùT¬V ¥´hPp Yô]®Vp
F] ùR¬®jÕs[]o. AnÜj §hP @ûUl©PªÚkÕ ùT\lThP
zz Bk§Vô®u ØRp Rô²Ve¡ úWôTôh¥d RLYpL°u BWiPôYÕ T§l©û]
ùRôûXúSôd¡ _mØ LôxÁ¬u XPôd¡p @ùU¬dLô®u ®iùY° ùRôûXúSôd¡
@ûUkÕs[ Bk§V Yô²Vp AWônf£ @±®Vp Lp® ¨ßY]m Yô]®Vp
ûUVj§p TVuTôh¥tÏ YkÕs[Õ. Lp®dLô] aYôn TpLûXdLZLjÕPu
zz º]ô®u Vôuú` úLôLv Tϧ«p DX¡u BûQkÕ ùY°«hPÕ. BkR AnYLm
ªLÜm TZûUVô] ®Xe¡]eL°u 1.4 ©p-Vu ©dNp YN§ÙûPV ¥´hPp
ùRôpùTôÚs T¥YeLs LiÓ©¥dLl úLUWôÜPu 1.8 ÁhPo ùRôûXúSôd¡ûV
ThÓs[]. ùLôiÓs[Õ.

zz v ù T « ² u ù N ® p - T p L û X d L Z L zz ¿¬¯Ü úSôVô°LÞdÏ ReLs ÏÞdúLôv


AWônf£Vô[oLs ¿¬-ÚkÕ L¬U UôÑdLû[ @[ÜLû[ AnÜ ùNnY§p DR®Óm
©¬jùRÓlTRtLô] קV úY§lùTôÚû[ YûL«p IÚ ùNVtûLVô] LûQVj§tÏ
LiÓ©¥jÕs[]o. ûLúT£ ùNV- Iuû\ ®gOô²Ls
DÚYôd¡Ùs[]o. BkR ùNV- ÏÞdúLôv
zz L-úTôo²Vô TpLûXdLZLjûRf úNokR @[ÅhÓ LÚ®Ls, BuÑ-u DhùNÛjÕd
RôWô Np-Yu T\ûY B\¡u @ûUlûT LÚ®Ls Utßm Ø¥ÜLs FÓdÏm Y¯Øû\
AWônkÕ, @úRúTôX Ih¥l ©¬Ùm IÚ Ls A¡VYtßPu BûQdLlTÓm §\u
LÚ®ûV DÚYôd¡Ùs[ôo. IÚ B\¡p, ùLôiÓ RVô¬dLlThÓs[Õ.
SÓ Co Tϧ«u BÚ ×\Øm áWp Fàm
BûZLs DiÓ. áWpLs Iuú\ôùPôuß zz WxVô®u úR£V @ÔNd§ LZLUô]
Ih¥«ÚdÏm; @ûY Lôtß, Ri½o ThPôp úWôNôúPôm @Ô DûXLÞdLôL ®TjûR
©¬kÕ, ÁiÓm úNokÕ ùLôsÞm. RôeLdá¥V F¬ùTôÚû[ úNôRû]«P
AWm©jÕs[Õ. BkR F¬ùTôÚs @ÔNd§
zz Bk§Vô®p ØR-p LiÓ©¥dLlThP áPeL°p LÓûUVô] ®TjÕdLû[ Rôe¡
Au¥TVô¥d F§olûTd ùLôiP ãlToTd ¨tÏm YiQm Ds[Õ.
UWTÔYô]Õ DX¡u LûP£Vô] @fÑ
@NXô] BVtûLûLV ùLôiP ©WúRNeL°p Uôof
Iu\ô] Aod¥d¡p LiÓ©¥dLlThÓs[Õ.
BkR Au¥TVô¥d F§ol× UWTÔYô]Õ zz ùRu @ùU¬dL SôÓL°p Iu\ô] CdYPô¬p
ÖiÔ«¬L°p TX UÚkÕ F§o®û]ûV Ds[ LôXúTôúLôv ¾®p, @¬V YûL AûU
GtTÓjÕ¡u\Õ. B]Uô] ùTo]ôi¥Lô F] @ûZdLlTÓm
WôhNR AûU Øt±Ûm @¯kÕ®hPRôL
zz TôoûYVtú\ôÚdÏ ìTôn úSôhÓLû[ LÚRlThPÕ. ÑUôo 113 AiÓLÞdÏ ©\Ï
@ûPVô[m LôiTRtÏ DRÜm úWôµ¦ LôXúTôúLôv ¾®p ùTo]ôi¥Lô AûU
Fàm ùNV-ûL HH¥ úWôTôo DÚYôd¡Ùs[Õ. D«o YôrYûR AnYô[oLs RtúTôÕ
BkR ùNV-Vô]Õ TûZV Utßm קV ìTôn LiÓ©¥jÕs[]o.

 A-Series in
          ©lWY¬ 2020
zz GÜLûQûLû[j Rôe¡f ùNpÛm ¿oêr¡d @ûUfNo UúLx NoUôYôp ùRôPe¡
LlTXô] TúRy (TôWºL ùUô¯«p ùYt±Pm) ûYdLlThÓs[Õ.
H CWôu DÚYôd¡Ùs[Õ. BkR ¿oêr¡ zz TôÕLôl× AWônf£ Utßm úUmTôhÓ ¨ßY]j
LlT-p Ds[ GÜLûQ êXm ÑUôo 2000 §tLôL FªNôh F]lTÓm IÚ ªu]Ô
¡.Á YûW«Xô] BXdÏLû[ Õp-VUôL D[ÜjÕû\ ùNVtûLúLôû[ BvúWô
RôdL Ø¥Ùm. @ûUl× ®i¦p ùNÛjR Ds[Õ.
zz AntBot Fu\ ùTV¬p ´.©.Fv. DR®«pXôUp zz ùNôjÕdLû[ BûQVj§u Y¯VôL
SPUôÓm DX¡u ØRp úWôúTôhûP, ©ùWgÑ F°ûUVôL UôtßYRtLô] ùUôûTp ùNV-
SôhûPf úNokR CNRS (National Centre for e# Dharti #H Uj§V ÅhÓYN§ Utßm SLo×\
Scientific Research) @ûUl©u AWônf£ @ûUfNLm @±ØLlTÓj§Ùs[Õ. BkR
Vô[oLs DÚYôd¡Ùs[]o. R[j§u êXm ùNôj§u ÏjRûLLôWo
zz ®Uô]l úTôdÏYWjÕ @ûUfNLUô]Õ YûWTPj§u êXm @Ru BÚl©PjûRd
Bk§V ®Uô] ¨ûXV AûQVjÕPu LiP±YÕPu ùNôj§u @¥lTûP ®YWe
BûQkÕ úLv 2019 (GAS 2019) Fu\ Lû[Ùm LôQ Ø¥Ùm.
ùNV-ûV @±ØLlTÓj§Ùs[Õ. BkR zz ùTeLÞì®p קV U²R ®iùY° ®Uô]
ùNV-Vô]Õ BûQl ©W§¨§Ls Utßm ûUVjûR BvúWô @±ØLm ùNnÕs[Õ.
úTfNô[oLs ReLÞPu LXkÕûWVôÓm BkR ûUVm BvúWô®u RûXûUVLm
YN§ûV YZeÏm. @Ú¡p @ûUdLlThÓs[Õ.
zz Bk§V ùTÚeLPp FpûXL°p TôÕLôlûT zz No.£.®. WôUu LiÓ©¥lûT @¥lTûPVôLd
@§L¬dL, Bk§V ®iùY° AWônf£ ûUVm, ùLôiÓ, ÑtßfãZÛdÏ Tô§l©pXôR YûL«p
©ùWgÑ ®iùY° ûUVjÕPu BûQkÕ úLWh¥-ÚkÕ ºWô] úXNo L§ûW (Random
קV ùNVtûLdúLôsLû[ ®i¦p GÜm Laser Ray), ùNuû] H.H.¥ (Chennai –
קV IlTkRj§p ûLùVÝj§hÓs[Õ. IIT) AWônf£Vô[oLs LiP±kÕs[]o.
zz F¡l§p YWXôtßdÏ ØkûRV LôXLhPjûR LônL±«-ÚkÕ úXNo L§o DtTj§
úNokR 50 UmªLû[ ®gOô²Ls ùNnVlTÓYÕ DX¡úXúV BÕRôu ØRpØû\
LiP±kÕs[]o. ùLnúWô®u ùRtÏ FuTÕ Ï±l©PjRdLÕ.
Tϧ«p SPjRlThP @LZôn®u úTôÕ BkR zz ¶úWôµUô (_lTôu)#®p DX¡u ØRp
UmªLs LiÓ©¥dLlThP]. BYt±p 12 ùNVtûL ®iLp UûZûV @kSôhÓ @WÑ
UmªLs ÏZkûRLs FuTÕ ùR¬VYkÕs[Õ. DÚYôdL Ds[Õ. (World First Artificial Meteor
zz ©WTgNj§u (Universe) úRôt\m ϱjÕ ×¬kÕ Shower).ùNVtûL ®iLp UûZ DiPôdÏm
ùLôsYRtLôL IhÓùUôjR YôuTWlûTÙm ùNVtûLúLôs Fl£úXôu 4 Fu\ WôdùLh
AnÜ ùNnYRtLôL ªLÜm ×LrùTt\ êXm ®i¦tÏ ùNÛj§Ùs[Õ.
§hPUô] v©VùWv (SPHEREx) §hPjûR zz U²R]ôp DÚYôdLlThP TÚY¨ûX Uôt\d
SôNô ùRôPeL Ds[Õ. BkR §hPjûR SôNô LôWQeL°]ôp Øt±Ûm @¯dLlThÓs[
2023#Am Ai¥p ùNVpTÓjR Ds[Õ. DX¡u ØRp D«¬]UôL Av§úW-V
zz Y ° U i P X Ñ Z p , ® i ù Y ° ã Z p £ß ©WÜi F- @±®dLlThÓs[Õ.
LiLô¦l×, úT¬PoLû[ RÓjRp Utßm zz 3 AiÓLÞdÏ úUp Bh-, Dl×Uô
Ïû\jRp Utßm @±®Vp NôokR AnÜLû[ úTôu\ úYL ûYjR DQÜl ùTôÚhLû[
úUtùLôsYRtLôL º]ôYô]Õ Ùuaôn ùLPôUp TôÕLôlTRtLô] Àm úW¥úVNu
2 Fu\ ùNVtûLdúLôû[ ®iùY° Fu\ ùRô¯pÖhTjûR ØmûT TpLûXdLZL
ÑtßlTôûR«p ùYt±LWUôL ùNÛj§Ùs[Õ. BVt©Vp úTWô£¬ûV PôdPo ûY`ô-
zz úUtÏ YeLô[j§u ùLôpLjRô®p Ds[ TômúTôúX LiÓ©¥jÕ NôRû] TûPjÕs[ôo.
@±®Vp SLWj§p Bk§Vô®u ØRp BRtLôL FXdhWôu Àm L§oÅfÑ Fu\
Fulldome#3D §ûWVWeLm Uj§V LXôfNôW ùRô¯pÖhTm TVuTÓjRlThÓs[Õ.

©lWY¬ 2020 A-Series in


  
zz SôNô®u BuûNh ®iLXUô]Õ, R]Õ £pÛ RVô¬l©p Øu]¦ Y¡dÏm, BuùPp
ØRXôYÕ LÚ®Vô] ¨XSÓdL Uô²ûV Utßm @ùU¬dL F¬Nd§j Õû\ A¡VûY
ùNqYôn ¡WLj§p ùTôÚj§Ùs[Õ. BÕ BûQkÕ, @úWôWô @§§\u L¦²ûV
ùNqYôn ¡WLj§p GtTÓm ¨XSÓdLm RVô¬dLÜs[].
Utßm ¨XSÓdL @§oÜLû[ LiLô¦dÏm. zz ©úW£p Sôh¥p BWÜ úSWj§p Utßm
BÕúY ùNqYôn ¡WLj§u úUtTWl©p B]fúNodûLdLôL R]Õ DP-p I°ûV
ùTôÚjRlThP DX¡u ØRp LÚ®VôÏm. DªÝm @¬V YûL éN¦ úRûW YôrYÕ
zz ¨X®u TLpùTôÝÕ úRôußm Tϧ«p ùR¬V YkÕs[Õ. BûR @×Rô©«p Ds[
¿o êXdáßLs Ñt± YÚYRôL SôNô®u ¨ëVôod TpLûXdLZL AWônf£Vô[oLs
¨XÜ DXÜlT¦ ®iLXUô] LRO – (Lunar LiÓ©¥jÕs[]o.
Reconnaissance Orbiter) LiP±kÕs[Õ.
zz @ùU¬dLô®u Av¥²p Ds[ ùPd^ôv
¨X®p LiP±VlThÓs[ BkR ¿Wô]Õ
TpLûXLZLj§p BÚkÕ ¡ÙYô ë
F¬ùTôÚs RVô¬lTRtÏ @pXÕ ã¬Vd
RûXûU«Xô] AWônf£Vô[oLs, Lôt±p
L§oL°-ÚkÕ TôÕLôlTRtÏ U²RoL[ôp
BÚkÕ CWlTRjûR D±g£, ÑjRUô] Ï¥¿WôL
TVuTÓjR RϧÙûPVÕ.
Uôtßm ùRô¯pÖhTjûR LiP±kÕs[]o.
zz DVoRW Yôù]ô- Utßm ùRôûXdLôh£
zz ÏßeúLôs Iuß 6.6 úLô¥ AiÓLÞdÏ
TWl×ûL úNûYLû[ @°lTRtLôL, º]ôYô]Õ
Øu×, 骫p ®ÝkRúTôÕ ûPú]ôNoLs
º]ôNôh 6C Fu\ ùNVtûLúLôû[ Xôe
IhÓùUôjRUôL @¯kRRtLô] ×ûR
Uôof 3B Fu\ GÜ YôL]j§u êXm ×®
T¥UeLû[ ®gOô²Ls LiP±kÕs[]o.
ÑtßYhPl TôûRdÏ ùNÛj§Ùs[Õ.
@ùU¬dLô®u YPdÏ PdúLôhPô UôLôQj§p
GlWp SûPùTt\ @LrYôWônf£«uúTôÕ éªûV
Rôd¡V ÏßeúLô[ôp @¯Üt\ ÁuLs,
zz ùNqYô«p D«¬]eLs YôrYRtLô] Nôj§Vd UWeLs A¡VYt±u ×ûRT¥UeLs Yô«XôL
áß ªL Ïû\Ü Fuß TX ®gOô²Ls BÕ ùR¬VYkÕs[Õ.
ùR¬®jÕ YÚm ¨ûX«p, RtùTôÝÕ
zz DXL YWXôt±p ØRp Øû\VôL LÚkÕû[
SôNô®u ¡ë¬Vô£h¥ úWôYo ùNVtûLdúLôs
«u ØRp ×ûLlTPm FÓdLlTh¥ÚlTûR
ùNqYô«p D«oLs YôrYRtLô] ARôW
Yô²VXô[oLs DߧlTÓj§ Ds[]o.
×ûLlTPeLû[ T§Ü ùNnÕ @àl©Ùs[Õ.
AWônf£Vô[oL[ôp UôuvPo F]
zz UôQYoLÞdÏm @YoL[Õ ùTtú\ôodÏm @ûZdLlTÓm BkR LÚkÕû[ 骫p
úRoÜ ùRôPoTô] U] @ÝjRjûR Ïû\l BÚkÕ ÑUôo 5.5 úLô¥ I°VôiÓLs
TRtLôL, Ï_Wôj§u Lôk§ SL¬p Ds[ ùTôÕ ùRôûX®p BÚd¡\Õ.
SXj§tLô] Bk§V ¨ßY]m Conquer Exam,
zz @ùU¬dLô®Ûs[ Uj§V ×ú[ô¬Pô TpLûXd
Be a Warriors Fu\ ùUôûTp ùNV-ûV
LZLjûRf úNokR AWônf£Vô[oLs.
@±ØLlTÓj§Ùs[Õ.
vTô£Vp ûXh UôÓúXhPo Fu\, F°V
zz ã¬V At\ûXl TVuTÓj§ LPp¿¬-ÚkÕ LÚ®«u EPôL I°ûV ùNÛj§]o.
ûahW_u F¬ùTôÚû[ DtTj§ ùNnÙm @kRWj§úXúV @kR I°dL§ûW BVp×
Y¯Øû\ûV vPôuúTôoÓ AWônf£Vô[oLs úYLjûR ®P Tô§VôLd Ïû\dLÜm,
LiÓ©¥jÕs[]o. BYoLs LPp ¿¬-ÚkÕ 30 UPeÏ úYLUôL TV¦dLÜm, YkR
ûahW_u YôÙ Utßm Ad³_u YôÙdLû[ I°ûV @úR TôûR«p §ÚmTf ùNpX
R²jR²VôL ©¬lTRtLô] IÚ ×§V Y¯ûVd ûYdLÜm Ø¥kRRôL, T¬úNôRû]«p CÓThP
LiP±kÕs[]o. AWônf£Vô[oLs ùR¬®jÕs[]o.
zz @ùU¬dLô, 2021#dÏs BÕYûW BpXôR zz û U d ú W ô N ô l h ¨ ß Y ] m Y ô µ e P u
@[ÜdÏ @§§\u L¦²Vô], @úWôWôûY TpLûXdLZLjÕPu BûQkÕ, ¥´hPp
@±ØLlTÓjR §hPªhÓs[Õ. £-dLu RLYpLû[ ¥.Fu.G AL Uôtßm LÚ®ûV

 A-Series in
          ©lWY¬ 2020
LiP±kÕs[Õ. BkR ùUuùTôÚs Reconnaissance Orbiter) LiP±kÕs[Õ.
@ûUlTô]Õ 0 Utßm 1 Fu\ ¥´hPp ¨X®p LiP±VlThÓs[ BkR ¿Wô]Õ
RLYpLû[ DNA Y¬ûN«p Ds[ (@¥û]u, F¬ùTôÚs RVô¬lTRtÏ @pXÕ ã¬Vd
ûRVªu, ÏYôû]u, ûNhúPôûNu) AL L§oL°-ÚkÕ TôÕLôlTRtÏ U²RoL[ôp
Uôt± RWÜLû[ úNªd¡\Õ. TVuTÓjR RϧÙûPVÕ.
zz º]ô®u BWiPôYÕ RûXØû\ RWÜ zz ùThúWô-Vm ùTôÚhL[ôp UôÑThP Ui¦-
¬úX ùNVtûLúLô[ô] Tian lian II 01 ÚkÕ ©¬jùRÓdLlThP Tôd¼¬Vj§u
Fu\ ùNVtûLúLôs, Xôe Uôof 3B Fu\ R²jÕYUô] §¬×Lû[l TVuTÓj§,
WôdùLh êXm £eNôe ùNVtûLúLôs H.H.¥ TmTôn AWônf£Vô[oLs, ÑtßãZp
GÜR[j§-ÚkÕ ùYt±LWUôL ®i¦p SfÑjRuûU«-ÚkÕ, ùTuúNôúVh
ùNÛjRlThPÕ. (úNô¥Vm ùTuúNôúVh DQÜl TôÕLôl©tÏ
zz ûaRWôTôj§p Ds[ D«WÔ Utßm TVuTÓjRlTÓ¡\Õ), ùTu£p ApLaôp
êXdáß D«¬Vp ûUVjûRf úNokR Utßm SlRûXu úTôu\Ytû\ ©¬jùRÓdÏm
AWônf£Vô[oLs Tôd¼¬VôdL°u Ùd§ûV LiÓ©¥jÕs[]o.
ùNpÑYûW DûPlT§p DRÜ¡u\ ØùWnu zz ùTÚmTôÛm ëúW£Vô Utßm Al©¬dLô
FiúPô ùTh¥Vôv Fu\ ùSô§ûVd LôÓL°p Y£dÏm ϧdÏm £Xk§Ls ØRp
LiÓ©¥jÕs[]o. RPûYVôL úLW[ Uô¨Xm FoQôÏ[j§u
zz ¨XÜdÏ U²RoLû[ Gk§f ùNpÛYRtLôL Bp-ÕÓ LôÓL°p LiÓ©¥dLlThÓs[Õ.
@ùU¬dL ®iùY° AnÜ ûUVUô] úUÛm BÕ Habrocestum UWTÔ ÏÓmTjûR
SôNôYô]Õ FvFpFp Fu\ WôdùLh¥p NôokR £Xk§ B]m FuTÕm ùR¬VYkÕs[Õ.
TVuTÓjÕm AoFv#25 Fu\ Fu´û] zz ÅÓLÞdÏ húWôu F]lTÓm A°pXô
ùYt±LWUôL úNôRû] ùNnÕs[Õ. Ïh¥ ®Uô]eL°u êXm ùTôÚhLû[ ùP-
zz _lTô²u ®iLXUô] aVéNô 2 A]Õ Y¬ ùNnYRtLô] YojRL çVô] húWôu
NÁTj§p ¬VÏ Fu\ £ßúLô°u ÁÕ IÚ úNûYûV áÏs ¨ßY]j§u Rôn ¨ßY]
£±V ϯûV GtTÓjÕYRtLôL ùY¥UÚkûR Uô] ApTùTh Av§úW-Vô®u LôuùTWô
@Ru ÁÕ úUôRf ùNnÕs[Õ. BRu SL¬p @±ØLTÓj§Ùs[Õ.
úSôdLUô]Õ, ã¬V ÏÓmTj§u úRôt\m zz UWQ ùTôjRôu Fu\ ùTV¬p RôYWeL°p
ϱjÕ @±YRôÏm. úSôn F§olûT @§L¬lTRtLô] קV
zz SôNô®u ®iùY° ùRôûXúSôd¡Vô] ùRô¯pÖhTjûR º]ô®u £eÏYô
ùPv#Bu RWÜLû[l TVuTÓj§ 5 ªp- TpLûXdLZL AWônf£Vô[oLs LiÓ©¥j
Vu AiÓLs TZûUVô] Utßm éªûV Õs[]o.
®P 60 UPeÏ ùT¬V @[ÜûPV קV zz ùNqYôn ¡WLj§u ¨XjR¥ Tôû\«p
úLôû[ AWônf£Vô[oLs LiP±kÕs[]o. Õû[«hÓ @Ru Uô§¬Lû[ ØRu Øû\
TOI 1970 Fuß BkR úLôs @±VlTÓ¡\Õ. VôL úNL¬jÕ @ùU¬dLô®u ¡ë¬Vô£h¥
AnYô[oLs BûR ãPô] N² Fuß AnÜdLXm NôRû]l TûPjÕs[Õ.
@ûZd¡u\]o. úNL¬dLlThP Uô§¬Ls ¡ë¬Vô£h¥«p
zz vLôhXôk§u F¥uTod TpLûXdLZLjûRf @ûUdLlThÓs[ AnYLj§tÏ @àlT
úNokR AWônf£Vô[oLs Nd§ YônkR L¦² ThÓs[Õ. BmUô§¬Lû[d ùLôiÓ ùNqYôn
Il×ÚYôdLjûR TVuTÓj§ DÚ¡V Ne¡- ¡WLj§u ¨XjR¥ ¿o ϱjÕ AnÜLs
¨ûX Fu\ קV TÚlùTôÚ°u ¨ûXûVd úUtùLôs[lThÓ YÚ¡u\Õ.
LiP±kÕs[]o. zz @iPùY°«p úU£Vo 87 Fu\ TôpùY°
UiPXj§p Ds[ @§L @§L Col×
zz ¨X®u TLpùTôÝÕ úRôußm Tϧ«p
®ûN ùLôiP M# 87 Fu\ûZdLlTÓm
¿o êXdáßLs Ñt± YÚYRôL SôNô®u
LÚkÕû[ûV ®gOô²Ls ØRuØû\
¨XÜ DXÜlT¦ ®iLXUô] LRO (Lunar

©lWY¬ 2020 A-Series in


  
VôL TPm ©¥jÕs[]o. BkR LÚkÕû[dÏ ×Rû] AnÜ ùNnYRtLôL @àlT®tÏm
aôYôn ùUô¯«p ùTôúY¶ F]l BÚ ùNVtûLúLôsLû[ FnÕYRtLô]
ùTV¬PlThÓs[Õ. ª`u FuTÕ Ï±l©PjRdLÕ.
úU zz ×Õ ùPp-«p Ds[ _ªVô ª-Vô BvXôªVô
TpLûXdLZL AWônf£Vô[oLs ÏÝ
zz ØRuØû\VôL ®iùY°«p Ds[ SUÕ ¡WôÀu ÏYôiPm ×s°Lû[ TVuTÓj§
úTWiPj§p ªLl TZûUVô] Í-Vm @phWôùNu£h¥q ÏYôiPm ùRoúUô
ûahûWh (HeH+) @V²ûV @ùU¬dLô®u ÁhPûW DÚYôd¡Ùs[]o. 27 ¥¡¬ ùNp£Vv
SôNô LiP±kÕs[Õ. B[m ©WTgNj§p ØRp #196 ¥¡¬ ùNp£Vv YûW BkR
Ïû\kÕ YÚm ùYlT¨ûXVô]Õ, @V²VôdLl ùRoúUôÁhPWôp Õp-VUôL @[®P Ø¥Ùm
ThP ûahW_u Utßm SÓ¨ûXVô] Í-Vm FuTÕ Ï±l©PjRdLÕ.
@ÔdLs ®û]׬YRtÏ @àU§d¡\Õ.
zz _ l T ô ² u R ² V ô o @ û U l T ô ]
zz SôNôûYf (NASA) úNokR TESS (Transiting BuùPovùPpXo ùPd]ôX´ @RàûPV
Exoplanet Survey Satellite) 骫u @[ûYd ØRXôm WôdùLhPô] úUôúUô#3H
ùLôiP HD21749b Fu\ קV ¡WLjûR ùYt±LWUôL ®i¦p G®Ùs[Õ. A°pXô
LiP±kÕs[Õ. BÕ éª«-ÚkÕ 53 I° WôdùLhPô] úUôúUô#3 ùaôdûLúPô
AiÓ ùRôûX®p Ds[Õ. BRu ÑtßLôXm GÜR[j§-ÚkÕ GYlThPÕ.
36 SôhLs AÏm.
zz Bk§V ùRô¯pÖhTd LZLj§u (HH¥ TômúT)
zz BvúWp Sôh¥u ùPp @-q TpLûXLZLj ùTô±«VXô[oLs AJIT Fu\ûZdLlTÓm
§u AWônf£Vô[oLs DX¡u ØRp IÚ ÖiùNV-ûV DÚYôd¡Ùs[]o.
YôvÏXo´úPp ùTô±dLlThP 3D BRVjûR
zz ¡ëlNôh F]lTÓm Lôvªd L§oLs
DÚYôd¡Ùs[]o. BÕ úSôVô°L°u
Tt±V AWônf£dÏ @àlTTP®ÚdÏm
ùNp Utßm D«¬Vp ùTôÚhLs êXm
ùNVtûLdúLôû[ RVô¬dÏm ÏÝ®tÏ
RVô¬dLlThÓs[Õ.
RûXYWôL Bk§V @ùU¬dL UôQYo úLNq
zz U ¬ V ô ] ô @ L ¯ « p I Ú × Õ û U V ô ] WôLYu FuTYûW SôNô ¨VªjÕs[Õ.
FiùQûV DiÔm Tôd¼¬VjûR
zz UWQ ùTôjRôu Fàm ùTV¬p RôYWeL°p
@±®VXô[oLs LiP±kÕs[]o.
úSôn F§olûT @§L¬lTRtLô] קV
zz ûahúWô LôoTû]f £ûRdÏm BkR ùRô¯pÖhTjûR º]ô®u £eÏYô
Tôd¼¬Vô êXm FiùQn L£ÜLû[ ÑjRm TpLûXdLZL AWônf£Vô[oLs LiÓ©¥j
ùNnYRtLô] Y¯Lû[ LiP±V Ø¥Ùm. Õs[]o.
zz BXeûL«u ØRXôYÕ ùNVtûLúLô[ô] zz @ùU¬dL ®iùY° AnÜ ûUVm SôNô
WôY]ô 1 Fu\ ùNVtûLúLôs @ùU¬dLô 2022#Am AiÓ DART (Double Asteroid
®u SôNô®u DR®ÙPu ùYt±LWUôL Redirection Test) F u à m ® i L X j û R
®i¦p ùNÛjRlThPÕ. ùLôiÓ ¥¥êu @pXÕ ¥¥úUôv#© Fu\
zz @kRUôu Utßm ¨dúLôTôo ¾Üj ùRôPoL°u ûZdLlTÓm 150 ÁhPo DVWm ùLôiP
¡úWh ¨dúLôTôo ¾ÜL°p aoovÀpÓ ÏßeúLôû[ Rôd¡ @¯dLÜs[Õ. BÕ ¡WL
LÚmTfûN Ï«pLs LiÓ©¥dLlThÓs[]. TôÕLôl× ùRô¯pÖhTjûR ¨ì©dÏm ØRp
ØVt£VôÏm.
zz HúWôl©V ®iùY° ¨ßY]m _lTôu
GúWôvúTv Fdvlú[ôúWNu ¨ßY] zz Y° UiPX ÁjúRû] @lT¥úV LTÇLWm
j ÕPu (JAXA) BûQkÕ ùNVpTÓjÕm ùNnÙm IÚ Tôd¼¬VôûY HúWôl©V
©l© ùLôXmúTô (Bepi colombo) ª`²u ®gOô²Ls NÁTj§p LiÓ©¥jÕs[]o.
úNôRû] ØVt£ ùYt± ùTtßs[Õ. BÕ Lôtû\ DiÓ, Lôt±úXúV YôÝm,
ùUûRúXôúLlNô úLôoúLô]ô Fuß ùTV¬Pl

 A-Series in
          ©lWY¬ 2020
ThÓs[ BkR Tôd¼¬VôdLs, ÁjúRu, úYLj§p Nôow ALdá¥V קV YûL
ûShW_u, LôoTu úUô]ôdûNÓ, ûahW_u úThP¬Lû[ @ùU¬dL AWônf£Vô[oLs
A¡VYtû\ DhùLôiÓ D«oYôr¡u\]. LiÓ©¥jÕs[]o. BkR úThP¬«p -j§Vm
úLôTôph Adû^ÓdÏ T§XôL Yô]¥Vm
zz ©-lûTuûNf úNokR íúNôu ¾®p, IÚ
ûPNpKûTÓûY TVuTÓjR T¬kÕûW
קV U²R B]j§u FÛm×Ls LiÓ©¥dLl
ùNnÕs[]o.
ThÓs[]. úaôúUô ÛúNôù]£v Fuß
ùTV¬PlThÓs[ BkR U²R B]m, @kRj _þu
¾®p, 50 A«Wm AiÓLÞdÏ Øu×
Yôrk§ÚdL úYiÓm F] ®gOô²Ls zz @±®Vp ®Uô]lTûP AWônf£ AnYLm,
U§l©hÓs[]o. ¨ë ùUd³úLô®p @ùU¬dL WôÔYj§u
ùYsû[ Nôihv GÜLûQ úNôRû]
zz NÁTj§p ùY°Vô] An®uT¥, 9 XhNm
Tϧ«p ÑV#TôÕLôl× DVo F¬Nd§ úXNo
úLô¥ Pu @[ÜdÏ T²lTôû\Ls DÚ¡,
ùNnØû\Vô[o (Self – Protect High Energy
LP-p LXk§Úd¡u\]. AnÜdÏs[ô],
Laser Demonstrator (SHiELD)) Fuß @ûZdLl
55 AiÓL°p @§LUô] BZl×
TÓm úXNûW T¬úNô§jR]o. BÕ ×®«-ÚkÕ
@XôvLô Tϧ«p Rôu. BeÏ UhÓm
®i¦p BÚdÏm ùTôÚû[Ùm ®i¦
@kRd LôXLhPj§p, 3 XhNm úLô¥ Pu
-ÚkÕ ®i¦-ÚdÏm ùTôÚû[Ùm RôdL
DÚ¡ LP-p LXkÕs[].
áÓm FuTÕ Ï±l©PjRdLÕ.
zz ©uXôkûR úNokR, ®.¥.¥., ùRô¯pÖhT
zz _lTôu, ÑlÃm F] ùTV¬PlThÓs[ קV
AWônf£ ûUV ®gOô²Ls, UWm
×pXh W«ûX 400 ¡úXôÁhPo úYLj§p
¾l©¥dLôUp RÓdL, UWdá¯-ÚkúR IÚ
BVd¡ NôRû] TûPjÕs[Õ. N 700 Y¬ûN«p
ùTôÚû[ DÚYôd¡ NôRû] TûPjÕs[]o.
ÑlÃm F] ùTV¬PlThÓs[ BkR קV
ùalùNp Fu\ @kR úY§lùTôÚs, ©\ ¾
×pXh W«p Ïû\Yô] FûPÙPàm, Ïû\kR
RÓlTôuLû[®P 10 UPeÏ Nd§ YônkjÕ
F¬Nd§ûV TVuTÓjÕm YûL«Ûm
F] @ûRd LiÓl©¥jR ®gOô²Ls
¨XSÓdLm úTôu\ ãZûX RôeÏm áÓRp
ùR¬®d¡u\]o.
TôÕLôl× YN§Lú[ôÓ Y¥YûUdLlThÓs[Õ.
zz R²Vôo ®iùY° ¨ßY]Uô] vúTvFdv,
zz ã¬V²u ùY°TWlûT AWôV 2020#Am
NoYúRN ®iùY° ¨ûXVj§tÏ IÚ £l
AiÓ ª`u A§jVô §hPjûR BvúWô
êXm Dßl×Lû[ @àlTj §hPªhÓs[Õ.
ùNVpTÓjRÜs[RôL @Ru RûXYo úL.£Yu
zz HH¥ Lôuéo AWônf£Vô[oLs @§L ùR¬®jRôo.
ÅdLjûR LhÓlTÓjÕm SôYp êXdáû\
zz Bk§V LPtTûP Fm.Ao.Fv.G.Fm (MRSAM)
@ûPVô[m LiÓs[]o.
GÜLûQf úNôRû]ûV ùYt±LWUôL
zz ®iùY° AnÜ YWXôt±p ØRpØû\ SPj§VÕ. MRSAM RûW«-ÚkÕ Yôu
VôL IÚ ÏßeúLô°u ÁÕ ùNVtûL BXdûL Õp-VUôL Rôd¡ @¯dÏm.
Ts[jûR DÚYôd¡ _lTô²V ®iùY° zz XiP²p ©WTX R²Vôo ¨ßY]j§tÏ
AWônf£Vô[oLs NôRû] ׬kÕs[]o. ùNôkRUô] 55 @ÓdÏ Uô¥ Lh¥Pj§u
骫-ÚkÕ 300 ªp-Vu ¡.Á. ùRôûX Df£«p DX¡úXúV ØRu ©WªléhÓm
®Ûs[ ¬ÙÏ ÏßeúLô°u ÁÕ BkR Ts[m YûL«p 360 ¥¡¬ úLôQj§p ¿fNp
GtTÓjRlThÓs[Õ. Ï[m @ûUdLlThÓs[Õ. BkR Uô¥«u
zz ×Ru úLô°u Dh×\lTϧ«p §PlùTôÚs Df£«p @ûUdLlThÓs[ ¿fNp Ï[j§u
LôQlTÓYRôL @ùU¬dL ®iùY° SôuÏl×\ ÑYoLÞm LiQô¥Vôp
AWônf£ ¨ßY]m SôNô LiÓ©¥jÕs[Õ. A]RôÏm.
zz -j§Vm úThP¬Lû[ ®P ªL @§L zz º ] ô @ R u Ø R p A W ô V l T h P T X

©lWY¬ 2020 A-Series in


  
ùNVpTôÓLû[ ùLôiP JF#17 RiPo úTôo `ôol Fu\ Tϧ«p @ToúX¥, ¡púU¬
®Uô]jûR Tô¡vRôu ®Uô] TûPdÏ F] ùTV¬PlThÓs[ BÚ BPeL°p
YZe¡Ùs[Õ. Õû[«hÓs[Õ. BûR ùNpK© TPUôL
zz ReLjûRj úR¥l TPÚm קV égûN FÓjÕm ¡ë¬Vô£h¥ @àl©Ùs[Õ.
YûLûV A v § ú W - V ®gOô²Ls @Õ Õû[«hP BPeL°p L°Ui
LiÓ©¥jÕs[]o. Ui¦p U-kÕ ¡PdÏm L²UeLs @§L @[®p BÚkRÕ An®p
GWô[Uô] égûN YûLLÞs Iu\ô] LiP±VlThÓs[Õ.
×Nô¬Vm AdvúTôWm, úY§Vp ç«p zz @ùU¬dLô®p @XôdLô H ùPd]ôX´v Fàm
Õ¥lTô] TX DúXôLeLÞPu ®û] ¨ßY]m úTôdÏYWjÕ ùS¬NûX R®odÏm
׬Ùm FuTûR ØuúT ®gOô²Ls @±k ®RUôL @úR úYû[«p ÑtßfãZÛdÏ
§ÚkR]o. DLkR YûL«p ûahW_u F¬ùTôÚ°p
zz @ùU¬dLô®u SôNô ®iùY° AnÜ BVeÏm T\dÏm LôûW @±ØLlTÓj§Ùs[Õ.
ûUVm R]Õ Ao¥ªv (Artemis) §hPjûR vûL F] ùTV¬PlThÓs[ BkR T\dÏm
ùY°«hÓs[Õ. Bj§hPUô]Õ ¨Xô®tÏ Lôo 5 úTo @UWdá¥V YûL«p Y¥YûUdLl
U²RoLû[ 2024#Am AiÓdÏs ùLôiÓ ThÓs[Õ.
ùNpÛm ØVt£VôÏm. BRu A°pXô zz NÁTj§p úLôYô®u ÏPdÚªVô®p BÚkÕ
úNôRû] ØVt£Vô] Ao¥ªv I 2020#dÏm IÚ ×§V Ï[® B]jûR ®gOô²Ls
ùNVpTÓjRlTÓm FuTÕ Ï±l©PjRdLÕ. LiÓ©¥jÕs[]o. BRtÏ ÏPÏߪVô
zz UWTÔ AWônf£Vô[oL°p £Xo, RdLô°dÏ WeÏÉLô¬ Fuß ùTV¬PlThÓs[Õ.
ÑûY RÚm UWTÔdLs FûY F] AWônkR]o. zz I¥Nô Uô¨Xm Nk§lé¬p BÚkÕ ©WmúUôv
R² SToLs Y[odÏm RdLô°Ls ØRp, ãlToúNô²d LlTp GÜLûQ ùYt±LWUôL
LôÓL°p Rô]ôL Y[Úm RdLô°Ls YûW, úNôRû] ùNnVlThÓs[Õ. DRDO Utßm
725 YûL RdLô°Lû[ @YoLs AWônkR]o. NPOM (WxVô®u WôÔY ùRô¯pÖhT
BRu ®û[YôL RdLô°dÏ Ú£Ùm, UQØm ¨ßY]m) A¡VYt±tÏm BûPúV IÚ
RÚm UWTÔdLs DhTP, 4,873 קV UWTÔd áhÓ ØVt£Vôp ©WmúUôv DÚYô¡Ùs[Õ
Lû[ @YoLs @ûPVô[m LiÓs[]o. FuTÕ Ï±l©PjRdLÕ.
zz A£V £eLj§u IhÓùUôjR UWTÔYô]Õ zz IÚ SôW[dÏ 25 Ll Lô© Ï¥jRôp áP
ûaRWôTôj§p Ds[ £FvHAo ùNp BÚRVj§tÏ FkR ¾eÏm YWôÕ F] XiPu
Utßm êXdáß D«¬Vp ûUVjûR úNokR Ï«u úU¬ TpLûXdLZL AWônf£Vô[oLs
AWônf£Vô[oL[ôp ØRuØû\VôL Y¬ûN ùR¬®jÕs[]o.
VôdLlThPÕ. BÕ A£V £eLeL°u T¬QôU zz úR£V TôpùTôÚhLs úUmTôhÓ Yô¬VUô]Õ
Y[of£ûV @±kÕ ùLôsYRtÏm, BkR DX¡u ØRpØû\VôL NDDB ABRO Murrah
B]eL°u £\kR úUXôiûUûV GtTÓjÕ Fu\ûZdLlTPdá¥V S§¿o FÚûUL°u
YRtÏm AWônf£Vô[oLÞdÏ DRÜm. IÚ ØÝûUVô] NkR§«]o Yô¬Vô] UWTÔ
zz Bk§Vô®u CWlTRm ªdL TϧL°p ùRôÏlûT DÚYôd¡Ùs[Õ.
YP¡ZdÏ Bk§VôÜm Iu\ôÏm. A]ôp zz £LôúLô TpLûXdLZLjûR úNokR AWônf£
BÕ LPkR 30 AiÓL[ôL ªL úYLUôL Vô[oLs T² ÙLjûR úNokR LPp ¿¬u
Y\iÓ ùLôi¥Úd¡u\Õ. BÕ Ï±jÕ ÁRØs[Ytû\ LiP±kÕs[]o. BYoLs
AnÜ ùNnR AWônf£Vô[oLs Ïû\kR UôXj¾ûY DÚYôdÏm ÑiQôm×d Lp
TÚYUûZ ùTô¯Yô]Õ ÕûQ ùYlTUiPX T¥ÜLû[ AnÜ ùNnÙmúTôÕ BYtû\
T£©d LP-u BVtûL Uôt\eLÞPu LiÓ©¥jR]o.
ùRôPo×ûPVÕ F] ùR¬®jÕs[]o.
zz @ùU¬dLô ¨XlTWl©-ÚkÕ Yôu BXdûLj
zz ùNqYôn ¡WLj§p AnÜ ùNnÕ YÚm Rôd¡ @¯dÏm úR£V úUmTÓjRlThP
SôNô®u ¡ë¬Vô£h¥ ®iLXm, UÜih

 A-Series in
          ©lWY¬ 2020
GÜLûQ @ûUl× II (NASAMS – II) ùY°l×\ I°YhPjûR TPm ©¥dÏm IÚ
Bk§Vô®tÏ ®tTû] ùNnV Il×Rp Ød¡Vl T¦VôÏm.
YZe¡Ùs[Õ BÕ YôuùY°j RôdÏRp zz ùTp´Vm @ÔdLÚ AWônf£ ûUVjûRf
L°-ÚkÕ úR£Vj RûXSLol TϧûVl úNokR Ûd¬£Vô ùPo£ FuTYo £dLXô]
TôÕLôlTRtLô] úUmTÓjRlThP Yôu Bk§Vl TÚY UûZûVd L¦lTRtÏ
TôÕLôl× @ûUlTôÏm. @ÔdLÚ ùY¥l×Lû[ LiP±YRtLô]
zz YP¡ZdÏ Bk§Vô®p @§Ûm ϱlTôL NôR]eL°u RWÜLû[l TVuTÓj§Ùs[ôo.
@v^ôm Uô¨Xj§p ùSp RYû[ Fu\ BYo TÚYUûZûVd L¦lTRtÏ L§¬VdLf
IÚ ×§V B]j§û] AWônf£Vô[oLs £ûRÜ @ÔdLÚYô] ùT¬-Vm 7#u
LiÓ©¥jÕs[]o. BÕ ªd¬ùXhPô @[ûY @±kÕd ùLôsYRtLôL WxVô
@nNô² Fuß ùTV¬PlThÓ BÚd¡u\Õ Utßm Av§úW-Vô®p Ds[ CTBTO ®u
Micryletta aishani ªd¬ùXhPô úT¬]j§u ¸r RWÜLû[l TVuTÓj§Ùs[ôo.
DXL @[®p @e¸L¬dLlThP B]eL°p zz FjR]ôp F¬ùTôÚ°p BVeÏm ØRp
5#YÕ RYû[ DÚùYÓjÕs[Õ. ûTdûL ¥.®.Fv úUôhPôo ¨ßY]m
_þûX @±ØLlTÓj§Ùs[Õ. FjR]ôp @§Lm
¡ûPdL Yônl×s[ LÚm× @§Lm ®û[Ùm
zz B k § V ® i ù Y ° A n Ü û U V j § u Uô¨XeL[ô] ULôWôx¥Wm, Dj§W©WúRNm
RûXYo £Yu 2030#Am AiÓdÏs Utßm LoSôPLj§p BlקV WL YôL]m
Bk§V ®iùY° AnÜ ûUVm R]dùL] @±ØLlTÓjRlThÓs[Õ.
ùNôkRUôL ®iùY° ¨ûXVjûR @ûUdÏm
zz Be¡Xôk§u NoúW TpLûXdLZLjûRf
F] @±®jRôo. BÕ ®iÔdÏ @§LUô]
úNokR aoúRq TôkRô®u RûXûU«Xô]
®iùY° ÅWoLû[ @àlT ISRO#ÜdÏ
AWônf£Vô[oLs, úLôdNôd¡ ûYWv
DRÜm.
Fu\ NôRôWQ N°ûV DiPôdÏm
zz vúTvFdv ¨ßY]m, R]Õ êu\ôYÕ ûYWvLû[ ùNÛj§, £ß¿olûT ×tßúSôûV
Utßm £dLXô] §hPUô] @§L FûP ÏQlTÓjÕYÕ Ï±jÕ AWônkÕ
ùLôiP TôpLu ®iLXjûR ®i¦tÏf YÚ¡u\]o.
ùNÛj§Ùs[Õ.
zz Av§úW-Vô®Ûs[ ¼d¡u TpLûXdLZL
zz SôNô®u ùNqYô«p Ds[ ¡ë¬Vô£h¥ ®gOô²Ls UßÑZt£dÏ YWôUp úRe¡
YôL]m, @dúLô°p BÕYûW @[®PlTPôR ®Ó¡u\ LiQô¥d L¯ÜLû[ ùTô¥ ùNnÕ,
ªL @§L @[®Xô] ÁjúRû]d UQÛdÏ T§XôL, Tô-Uo Lôu¡ÃhûP
LiP±kÕs[Õ. BÕ 221 ppbv (per billion units TVuTÓj§ ùNeLp RVô¬jÕs[]o.
by volume L] @[®p IÚ ©p-Vu @X¡p
21 TϧLs) Fuß @[®PlThÓs[Õ. zz ù R t Ï © ú W £ - p ù Y o ù T o N q ì v
TWô]ôFu£v Fu\ ûPú]N¬u ×ûRT¥Ye
zz @ùU¬dLô®u ®iùY° ¨ßY]Uô] SôNô, Lû[ AWônf£Vô[oLs LiÓ©¥jÕs[]o.
YôrdûL«u @±Ï±Lû[j úRÓYRtLôL
T\dÏm Up¥ úWôYo YôL]Uô] ¥WôLuK©û[ zz ©-lûTuv Sôh¥u ¾Ül TϧL°p,
N²«u ¨XÜYô] ûPhPàdÏ @àlT LpûXf §uß ùN±jÕ UQp ÕLsL[ôL
§hPªhÓs[Õ. BÕ 2026#Am Ai¥p ùY°úVtßm @ÑWl ×Ý Iuû\, ®gOô²Ls
ùRôPeLlThÓ 2034#Am Ai¥p N²«u LiÓ©¥jÕs[]o. ¿ÚdL¥«p Tôû\Lû[
ªLlùT¬V Nk§W²p RûW«\eÏm. ùUpXj Õû[«hÓ ÏûPkÕ, DiÔm
YZdLm ùLôiPÕ, -úRô¬úPô @TôPô²Lô
zz ã¬Vû] TPm ©¥dÏm PUNCH §hPjûR
Fu\ ×Ý.
RûXûUúVtß Y¯ SPjÕYRtLôL ùPdNôû^f
úNokR ùRuúUtÏ AWônf£ ¨ßY]jûR zz B U V U û X « p B Ú k Õ T ² D Ú Ï Y Õ
NASA úRoÜ ùNnÕs[Õ. BÕ ã¬V²u LPkR 2000#Am AiûP úTôX BÚ

©lWY¬ 2020 A-Series in


  
UPeLô¡Ùs[RôL @ùU¬dLô®u ùLôXm©Vô ùR¬®jÕs[ôo.
TpLûXdLZLm R]Õ An®p ùR¬®jÕs[Õ. zz ©¥Fv 70 ShNj§Wm úLô[ôL UôßYûR
×®«u ùYlTm @§L¬jÕs[Rôp BkR AWônf£Vô[oLs ØRuØû\VôL TPm
¨ûX GtThÓs[Õ F] AnYô[oLs ©¥jÕs[]o. LPkR YÚPm LiÓ©¥dLlThP
ùR¬®jÕs[]o. קRôL úRôußm úLôÞdÏ RtúTôÕ ©¥Fv
zz ùNqYôn ¡WLjÕdÏ ØRpØû\VôL @ÓjR 70© F] ùTVo Uôt\m ùNnVlThÓs[úRôÓ,
AiÓ _þûX @pXÕ ALvÓ UôRYôd¡p RtúTôÕ LiÓ©¥dLlThP Utù\ôÚ ×§RôL
®iLXjûR @àlT º]ô §hPªhÓs[Õ. úRôußm úLôÞdÏ ©¥Fv£ F] ùTV¬Pl
@RuêXm, ùNqYôn ¡WLm DÚYô] ®Rm, ThÓs[Õ.
T¬QôU Y[of£ A¡VYtû\ ùR¬kÕùLôs[
Ø¥Ùm. ALvh
zz _lTô²V ®gOô²Ls, BÕYûW RôYW zz FúXôu Uvd¡u êû[ BVk§W BûPØLj
UWTÔdL°p ùNnV Ø¥VôR Iuû\ ùNnÕ §tLô] קV ùRô¯p ¨ßY]Uô] ¨ëWô-ed
Lôh¥Ùs[]o. úPôd¡Vô TpLûXdLZL Lô]Õ IÚ ST¬u U]ûRl T¥dLd á¥V
®gOô²Ls, ùSp Utßm LÓÏ A¡V YûL«p Lm©Vt\ ùTôÚjRlTPdá¥V
Yt±u ûUhúPôLôih¬Vô ¥.Fu.G.,dLû[ NôR]jûR DÚYôd¡ YÚYRôL ùR¬®jÕs[Õ.
ØRp Øû\VôL §ÚjRm ùNnÕ ùYt± zz 2024#Am AiÓ ¨X®tÏ ÅWôeLû]Lû[
LiÓs[]o. @àlT SôNô §hPªhÓs[Õ. Bj§hPj§tÏ
zz ídúLôv¡u Fu\ ùTV¬p ÛdúLôùPoUô AoùPªv Fuß ùTV¬PlThÓs[Õ.
úSôûV ÏQlTÓjÕm ê-ûL UÚkûR WôÔY zz ®iùY°j Õû\«p T¦×¬Ùm Tu]ôhÓ
AWônf£ ¨ßY]jûR úNokR AWônf£Vô[o Õ°o ¨ßY]eLû[ ColTRtÏm ®iùY°
úaUkj TôiúP LiÓ©¥jÕs[ôo. ùRôPoTô] ùRô¯pÖhTm AWônf£ Utßm
zz TmÀs F]lTÓm @vhúWôÀ úWôúTô úUmTôhÓdLô] ØRuûU ûUVUôL @ûR
®iùY°«p R]Õ ùNôkR At\-p DÚYôdÏYRtÏUôL §ÚY]kR×Wj§p
T\kR ØRp úWôúTô Fàm ùTVûW ùTtßs[Õ. Sôh¥u ØRp @§SÅ] ®iùY°
BÕ @ùU¬dL ®iùY° AnÜ ûUVUô] @ûUl×Ls éeLôûY @ûUdL úLW[ Uô¨X
SôNô®]ôp @àlTlThPRôÏm. @WÑ Ø¥Ü ùNnÕs[Õ.
zz LôoTu ÏYôiPm Pôhv Fu\ ùTV¬p zz Bk§V ®iùY° AWônf£ @ûUl× R]Õ
¨XdL¬«-ÚkÕ RVô¬dLlTÓm D«o UÚjÕY ØRp ã¬V TVQUô] A§jVô P1 H 2020 ØRp
ÕLsLû[ TVuTÓj§ úLuNo ùNpLû[ Tô§«p ùRôPeLlúTôYRôL @±®jÕs[Õ.
LiP±YRtLô] ùRô¯pÖhTjûR @±®Vp ã¬V²u ùLôúWô]ôûYl T¥lTRtLô] ØRp
Utßm AWônf£ LÜu£p YP¡ZdÏ @±®Vp @olT¦l× @±®Vp T¦ BÕYôÏm.
ùRô¯pÖhTjûR úNokR AWônf£Vô[oLs zz TôpùY° UiPXj§p 28 קV YûL
LiÓ©¥jÕs[]o. ®iÁuLû[ Bk§V ®gOô²Ls
zz 69 SôhLs ã¬Vu Uû\VôR @§NV ¾Ü LiÓ©¥jÕs[]o. éªdÏ @Ú¡p
Fàm ùTVûW HúWôl©V SôÓL°p Iu\ô] @ûUkÕs[ Fu´£ 4147 Fu\ ®iÁu
SôoúY«p Ds[ ùNômUôúWô« Fu\ ¾Ü ùRôϧ«p קV YûL«Xô] 28 ®iÁuLs
ùTtßs[Õ. LiÓ©¥dLlThÓs[].

zz Bk§Vô Nôo©p U²RoLû[ ®iùY°dÏ zz º]ô®u ùNVtûLdúLôs GÜRp YN§Lû[l


@àl×m ØRp §hPj§uT¥ LLuVôu TVuTÓj§ 2022#Am Ai¥p R]Õ ØRp
®iLXjûR 2021#Am Ai¥u Bߧ«p ®iùY° ÅWûW ®iùY°dÏ @àlT
®i¦p ùNÛjR SPY¥dûLLs FÓdLl úTôYRôL Tô¡vRôu @±®jÕs[Õ.
ThÓ YÚ¡u\] F] BvúWô RûXYo £Yu zz U²R]ôp DÚYôdLlThP TÚY¨ûX Uôt\d

 A-Series in
          ©lWY¬ 2020
LôWQeL°]ôp Øt±Ûm @¯dLlThÓs[ TX ®gOô²Ls SPjÕm, ®i¦Vp
DX¡u ØRp D«¬]UôL Av§úW-V £ß NeLj§u úNôRû] ØVt£Vô] ûXhùN«p # 2
©WÜi F- @±®dLlThÓs[Õ. JPm RtúTôÕ, @Ru ã¬V I° TônUWjûR
zz @ ù U ¬ d L ô ® Û s [ U j § V × ú [ ô ¬ P ô ùYt±LWUôL ®¬jÕ TV¦dL AWm©jÕs[Õ.
TpLûXdLZLjûRf úNokR AWônf£Vô[oLs. zz @iPôo¥Lô Utßm ¡ÃuXôk§p Ds[
vTô£Vp ûXh UôÓúXhPo Fu\, F°V T²UûXLs ªLÜm úYLUôL DÚ¡
LÚ®«u EPôL I°ûV ùNÛj§]o. YÚ¡u\]. @ÕÜm LPkR 25 AiÓL°p
@kRWj§úXúV @kR I°dL§ûW BVp× LPp¿o UhPm 7.5 ùN.Á. DVokÕs[RôL
úYLjûR ®P Tô§VôLd Ïû\dLÜm, AWônf£Vô[oLs ùR¬®jÕs[]o.
30 UPeÏ úYLUôL TV¦dLÜm, YkR zz ¨ë£Xôk§p 1 úLô¥úV 90 XhNm AiÓ
I°ûV @úR TôûR«p §ÚmTf ùNpX LÞdÏ Øu× YôrkR WôhNR ¡°«u
ûYdLÜm Ø¥kRRôL, T¬úNôRû]«p CÓThP ×ûRT¥YeLs LiùPÓdLlThÓs[Õ.
AWônf£Vô[oLs ùR¬®jÕs[]o. ¡°«u @NôRôWQ DVWm Utßm
zz F¡l§p YWXôtßdÏ ØkûRV LôXLhPjûR Y-ûUûV @e¸L¬dÏm YûL«p @kR
úNokR 50 UmªLû[ ®gOô²Ls ¡°dÏ ùaWôd¡sv BuvùTdúPPv F]
LiP±kÕs[]o. ùLnúWô®u ùRtÏ ùTV¬lThÓs[Õ.
Tϧ«p SPjRlThP @LZôn®u úTôÕ BkR zz Bk§V ®iùY° AWônf£ @ûUl× (ISRO)
UmªLs LiÓ©¥dLlThP]. BYt±p 12 DPu BûQkÕ ûaRWôTôj§p ØRp R²Vôo
UmªLs ÏZkûRLs FuTÕ ùR¬VYkÕs[Õ. ®iùY° @ÚeLôh£VLm §\dLlThÓs[Õ.
zz @où_u¥]ô Sôh¥p קV YûL ûPú]ôN¬u zz ºûUdLÚúYX UW I¯l× ùRôPoTô]
FÛm×dáÓ LiÓ©¥dLlThÓs[Õ. AWônf£Lû[ úUtùLôsÞm YûL«p 30
T_PôNôWv úWôàvûT]dv Fu\ ûPú] ¡Wôm FûPÙs[ ¿o ùNVtûLúLôû[ Lìo
ôNo YûL LÝj§p GWô[Uô] ùLôm×LÞPu UôQYoLs DÚYôd¡Vs[]o.
YôrkÕ YkRRôL áßlTÓ¡\Õ. BkR YûL
ûPú]ôNoLs 230 ªp-Vu AiÓLÞdÏ zz _lTôu Sôh¥u ¶úWôµUô®p DX¡u
ØtThPûY FuTÕ Ï±l©PjRdLÕ. ØRp ùNVtûL ®iLp UûZûV
@kSôhÓ @WÑ DÚYôdL Ds[Õ. ùNVtûL
zz SôNô®u BuûNh ®iLXUô]Õ, R]Õ ®iLp UûZ DiPôdÏm ùNVtûLúLôs,
ØRXôYÕ LÚ®Vô] ¨XSÓdL Uô²ûV Fl£úXôu 4 Fu\ WôdùLh êXm ®i¦tÏ
ùNqYôn ¡WLj§p ùTôÚj§Ùs[Õ. BÕ ùNÛjRlThÓs[Õ.
ùNqYôn ¡WLj§p GtTÓm ¨XSÓdLm
Utßm ¨XSÓdL @§oÜLû[ LiLô¦dÏm. zz vUôoh @d¬LôlPo Fu\ ùTV¬p ®YNôV
BÕúY ùNqYôn ¡WLj§u úUtTWl©p ¨XeL°p éf£dùLôp- UÚkÕLû[
ùTôÚjRlThP DX¡u ØRp LÚ®VôÏm. ùR°lTRtLô] Rô²Ve¡ £ß ùa-LôlPûW
HH¥ ùUhWôv AWônf£Vô[oLs
zz @×Rô©«p @ÁWL TpLûXdLZLj§u DÚYôd¡Ùs[]o.
AWônf£Vô[oLs ØlT¬UôQ ©W§ùVÓdÏm
ùRô¯pÖhTj§u @¥lTûP«p SÅ] úWôúTô zz ã¬V I° Nd§«p BVeÏm º]ô®u
Iuû\ DÚYôd¡ NôRû] TûPjÕs[]o. A°pXô ®Uô]m úUô³#2 R]Õ ØRp
BkR úWôúTô®u DPp TϧLs Utßm TVQjûR ùYt±LWUôL ¨û\Ü ùNnÕs[Õ.
ùNVpTÓm TôLeLs @û]jÕm 3 ¥ BkR ®Uô]m, TL-p ã¬V I°«p 8 U¦
©¬u¥e F]lTÓm ØlT¬UôQ ©W§ùVÓjRp úSWm ªuNd§ûV úNªjÕ ùLôiÓ BWÜ
Øû\«p DÚYôdLlThÓs[Õ. ªRUô] úYLj§p 12 U¦ úSWm YûW
@§LThNUôL 8 A«Wm ÁhPo DVWm YûW
zz vúTv Fdv @àl©V, TôpLu ùa® Yô²p T\dÏm §\u ùLôiPÕ.
WôdùLh êXm, ûXhùN«p # 2 Fu\ JPm,
ùYt±LWUôL ®iùY°«p ùNÛjRlTThPÕ. zz ú L m l ú W ô W ô v P o K T ô p L P v F u ß

©lWY¬ 2020 A-Series in


  
@ûZdLlTÓm vPôo Yôov ®iLXjûR ®iùY° ûUVj§u Uôov Fdv©Wv
IjR RûXÙPu á¥V A§LôX LPp D«¬]j ª`u Bl×ûLlTPeLû[ FÓjÕs[Õ.
§u ètßdLQdLô] ×ûRT¥YeLs úLôúWôúXôq Ts[m ùNqYôn ¡WLj§u
L]Pô®p LiÓ©¥dLlThÓs[]. BkR YPdÏ Rôr¨XlTϧ«p Ds[Õ LiP±Vl
D«¬]m Buû\V AojúWôTôh YûL ThÓs[Õ. BlùTÚm Ts[m T²Vôp
®XeÏL°u ÏÓmTjûR NôokRÕ Fußm, ¨û\kRRpX, T²dLh¥L[ôp ¨û\kRÕ.
@ûY 506 ªp-Vu AiÓLÞdÏ Øu zz A v § ú W - V ô ® Û s [ Ï Å u v X ô k Õ
úLml¬Vu LôXLhPj§p Yôrk§ÚdLXôm TpLûXdLZL ®gOô²Ls UôÓLs FÓjÕd
Fußm AWônf£«p ùR¬®dLlThÓs[Õ. ùLôsÞm ¾Y]j§p £XYûL LPp Tô£Lû[
zz AúWôq#3 Fàm ùTV¬p BvúWp Utßm úNojRôp, 99 ÁjúRu ùY°úVt\jûR ¨ßjR
@ùU¬dLô SôÓLs BûQkÕ DÚYôd¡Ùs[ Ø¥Ùm F], LiP±kÕ Ds[]o.
LiPm ®hÓ LiPm TôÙm RÓl× GÜLûQ
zz v ù T « ² Û s [ , S Y ô ú W ù T ô Õ l
ùYt±LWUôL úNô§dLlThPÕ. BkR
TpLûXdLZLm Utßm ©¬hP²Ûs[,
GÜLûQûV @ùU¬dLô®u úTô«e
©¬vPôp TpLûXdLZLm A¡VYtû\
¨ßY]Øm, BvúW-u GúWôvúTv ¨ßY]
úNokR ®gOô²Ls, I-d ¡Ód¡ûV
Øm BûQkÕ DÚYôd¡Ùs[].
DÚYôd¡Ùs[]o. úUÛm UÚjÕYj Õû\«p
zz ùadNô F] ùTV¬PlThÓs[ Itû\ STo I-d ¡Ód¡ ùRô¯pÖhTm F§oLôXj§p
TVQm ùNnÙm ®Uô]m @ùU¬dLô®p TVuTÓm F], ®gOô²Ls ùR¬®j
@±ØLm ùNnVlThÓs[Õ. BkR ®Uô]m Õs[]o.
Ri½¬p ªRdÏm YûL«Ûm, @YNW
zz ©WôuVô úTôu\ IÚ Á²u ×ûRdLlThP
LôXj§p D«o ©ûZdL TôWôãh DR®ÙPàm
Fg£Ùs[ ×ûRT¥Ym Iuû\ ®gOô²Ls
Y¥YûUdLlThÓs[Õ.
LiÓ©¥jÕs[]o. BÕúY TZûUVô] Bû\f£
zz Spektr # RG (Spectrum Roentgen Gamma) Nôl©Óm Áu Fuß ùR¬VYkÕs[Õ. 150
Fu\ ùTV¬p Y-ûU YônkR Fdv#úW ªp-Vu AiÓLÞdÏ Øu× YôrkR
LiLô¦l× ®iùY° ùRôûXúSôd¡ûV BkR D«¬]j§u FÛm× ùRtÏ ù_oU²«p
WxVô ®iÔdÏ @àl©Ùs[Õ. LiÓ©¥dLlThÓs[Õ. RtúTôÕs[
zz aVôé`ô#2 Fu\ _lTôu Sôh¥u ®iLXm ©WôuVôdLû[ úTôX áoûUVô] TtLs
BWiPôYÕ Øû\VôL ¬ëÏ ÏßeúLô°p BRtÏ BÚd¡\Õ.
RûW«\e¡Ùs[Õ. zz Bk§Vô®p ØRpØû\VôL TúVô ù_h
zz ¥WôLuK©û[ Fu\ ùTV¬p ®iLXjûR F¬ùTôÚs TVuTÓjRlThÓ WôÔY
N²dúLô°u ÕûQdúLô[ô] ûPhPû] ®Uô]eLs ùYt±LWUôL T¬úNô§dLl
AWônf£ ùNnV @àlT®ÚlTRôL SôNô ThÓs[Õ. BkR D«¬ F¬ùTôÚ[ô]Õ
@±®jÕs[Õ. BkR §hPm YÚm 2026#Am _ôhúWôTô FiûQ«-ÚkÕ RVô¬dLl
Ai¥p ùNVpTÓjRlTÓm. TÓ¡\Õ.
zz ûaToKúTôXô#1 Fu\ ùTV¬p º]ô®u zz BÕYûW éªûV úSôd¡ LôQlTPôR
ØRp Y¦L ç«Xô] WôdùLhûP À´eûL ¨X®u Utù\ôÚ TϧVôL LÚRlTÓm
úNokR R²Vôo ¨ßY]Uô] BuvùPpXo úYôu LoUôu Fuàm Tϧ«p º]ô®u
Ïú[ô¬ vúTv ùPd]ôX´ Fàm @ûUl× Nôe B#4 ùNVtûLdúLôs RûW«\e¡VÕ.
RVô¬jÕs[Õ. ¨X®u UßTdLjûR AnÜ ùNnYRtLôL
@àlTlThP ®iLXjûR, LWÓØWPô]
ùNlPmTo Tϧ«p ùYt±LWUôL RûW«\d¡ º]ô
zz ùNqYôn ¡WLj§u úLôúWôúXôq Ts[j§p NôRû] TûPjÕs[Õ.
ØÝYÕUôL T² ¨û\k§ÚdÏm ×ûLlTP zz ã¬V ÏÓmTj§tÏ @lTôp Ds[ Lô£úVô¥Vô
ùUôuß RtúTôÕ ùY°YkÕs[Õ. HúWôl©V

 A-Series in
          ©lWY¬ 2020
®iÁu ÏÝ®p IÚ ùY°dúLôû[ êXm ùYt±LWUôL ®i¦p ùNÛj§Ùs[Õ.
AnYô[oLs LiP±kÕs[]o. ãlTo Foj DXLm ØÝYÕm TVuTÓjRlTÓm
F] @ûZdLlTÓm BÕ éª«u ¨û\ûV @ùU¬dLô®u ´©Fv ùRô¯pÖhTj§tÏ
®P 5 UPeÏ @§L ¨û\ÙûPVÕ. Uôt\ôL º]ô DÚYôd¡Ùs[ ùRô¯pÖhTúU
BkR ùTnúPô AÏm.
zz º]ô UAV#CH7 Fu\ ùTV¬p @§SÅ]
A°pXô ®Uô]jûR RVô¬jÕs[Õ. BkR zz º]ô®u BVt©Vp AWônf£Vô[oLs 100
A°pXô ®Uô]j§u êXm AÙRe ªp-Vu ¥¡¬ ùYlTm ùLôiP ùNVtûL
Lû[ ÑUkÕ ùNuß RôdL Ø¥Ùm. ã¬Vû] DÚYôd¡Ùs[]o. SmØûPV
BRu êXm @ùU¬dLôûY @ÓjÕ ªL Yô]j§p Ds[ ã¬V²u ùYlT¨ûX
DVWj§p T\dLdá¥V Utßm @§SÅ] 15 ªp-Vu ¥¡¬ ùNp£Vv UhÓúU
A°pXô ®Uô]jûR RVô¬jÕs[ ùLôiPRôÏm.
BWiPôYÕ SôÓ Fàm ùTÚûUûV º]ô
zz BÕYûW LiP±VlTPô§ÚkR êû[«Ûs[
ùTtßs[Õ.
Uû\®Pm Iuß RtúTôÕ LiP±Vl
zz ã¬V²-ÚkÕ ªL ùRôûX®p BÚlTúRôÓ ThÓs[Õ. ùYßm 2 ªp-ÁhPo @[úY
UhÓUpXôUp ã¬Vd ÏÓmTj§u ªLlTûZV DûPV ϱjR Tϧ ùRôPoTô] AWônf£Ls
ÖiúLôs @ph¥Uô ÕúX F] ®gOô²Ls ùRôPokÕ SûPùTtß YÚYRôL ®gOô²Ls
ùR¬®jÕs[]o. BkR @ph¥Uô ÕúX ùR¬®jÕs[]o.
ÖiúLô°u @ÚúL ùNuß @ùU¬dLô®u
zz ù U d £ ú L ô û Y ú N o k R , @ ú R U _ ô d
¨ë aô¬Nu ®iLXm NôRû] TûPj
Ts[jRôdÏ TpLûXdLZL ®gOô²Ls,
Õs[Õ.
Y\iP 骫Ûm ®û[Ùm Lt\ôûZ
zz K©hí Fu\ ùTV¬p úLuNo Utßm ¿¬¯Ü L°-ÚkÕ ©[ôv¥d RVô¬dL Ø¥Ùm
úSônLû[ LiÓ©¥dL á¥V vUôoh F], LiP±kÕs[]o.
PônùXhûP @ùU¬dLô®u Fm.H.¥.
¨ßY]j§u AWônf£Vô[oLs LiÓ©¥j zz Av§úW-VôûY úNokR Xô húWôl
Õs[]o. TpLûXdLZLjûR úNokR ®gOô²Ls
×ûL«ûX ùN¥«Ûs[ édL°-ÚkÕ ×§V
zz C£úTôu vPôo Fu\ ùTV¬p Bk§Vô®u YûL Au¥TVô¥dÏLû[ DÚYôdL Ø¥Ùm
ØRp ÏZkûRLÞdLô] ùUôûTp úTôû] F] ùR¬®jÕs[]o.
BúSôYv FiPo©ûWNv Fu\ ¨ßY]m
ùY°«hÓs[Õ. zz Bk§Vô®u ØRp Rô²Ve¡ úWôTôh¥d
ùRôûXúSôd¡ _mØ LôxÁ¬u XPôd¡p
zz ¨ë£Xôk§p U²RûWl úTôu\ DVWm @ûUkÕs[ Bk§V Yô²Vp AWônf£
ùLôiP ùTuÏ®u T\ûY«u FÛm×dáÓ ûUVj§p TVuTôh¥tÏ YkÕs[Õ.
LiÓ©¥dLlThÓs[Õ. @kR FÛm×Lû[
AnÜ ùNnR§p @ûY TX úLô¥ zz º ] ô ® u V ô u ú ` ú L ô L v T Ï § « p
AiÓLÞdÏ Øu]o @¯kÕ úTô], DX¡u ªLÜm TZûUVô] ®Xe¡]
BÕYûW @±VlTPôR ùTuÏ®u B]jûR eL°u ùRôpùTôÚs T¥YeLs
úNokRûY FuTÕ ùR¬VYkRÕ. LiÓ©¥dLlThÓs[].

zz v©uúSdLo Fu\ ùTV¬p, DX¡u ªLlùT¬V zz vùT«²u ùN®p- TpLûXdLZL AWônf£


U²R êû[ úTôuß ùNVpTPá¥V ãlTo Vô[oLs ¿¬-ÚkÕ L¬U UôÑdLû[
LmlëhPo Be¡Xôk§p UôuùNvPo ©¬jùRÓlTRtLô] קV úY§lùTôÚû[
TpLûXdLZLj§p DÚYôdLlThÓs[Õ. LiÓ©¥jÕs[]o.

zz ùTnúPô Y¯LôhÓm ùRô¯pÖhTjÕdÏ zz êd¡Ûm, ùRôiûP«Ûm £X SpX Tôd¼¬Vô


úUÛm YÛúNodÏm YiQm, BÚ ×§V BÚkRôp lð ûYWv RôdÏR--ÚkÕ
ùNVtûLdúLôsLû[ º]ô Uôof 38 WôdùLh¥u SmUôp Rl©dL Ø¥Ùm FuTûR, @ùU¬dLô

©lWY¬ 2020 A-Series in


  
®Ûs[ ªf£Lu TpLûXdLZL ®gOô²Ls zz ¨ë£Xôk§p, DX¡u TZûUVô] T\ûY
LiP±kÕs[]o. B]eL°p Iu±u ×ûRT¥YeLû[
®gOô²Ls LiÓ©¥jÕs[]o. קRôL
@dúPôTo LiÓ©¥dLlThP LPtT\ûY ×ûRT¥Yj§tÏ
zz úLôphxªûPh Fu\ ùTV¬p קV ×úWôhúPôúPôuúPôTùP¬dv ÚjúR F]
RôÕlùTôÚû[ IÚ ûYWj§u DhTϧ«p ùTV¬PlThÓs[Õ.
L]Pô®u @pùToPô TpLûXdLZLj§u zz @ùU¬dL ®iùY° AnÜ ûUVm SôNô
AnYô[oLs LiÓ©¥jÕs[]o. Lôtß ®iùY°ûV Nk§dÏm BPjûR
zz ùNqYôn, ®VôZu A¡V úLôsLÞdÏ AWôV HLôu Fu\ ùNVtûLdúLôû[
BûPúV YXm YÚm £±V úLôÞdÏ G®Ùs[Õ. Ï°oNôR] ùTh¥ @[®Xô]
BkÕvRô² TôPLo Ti¥h _vWôw HLôu ùNVtûLdúLôs @Vú]ôv©V¬p
ùTVûW ãh¥ NoYúRN Yô²Vp ë²Vu Ds[ YôÙdL°-ÚkÕ DÚYôÏm Lôtú\ôh
LÜW®jÕs[Õ. BkR £±V úLôs LPkR PjûR Tt± AnÜ ùNnÙm.
2006#Am AiÓ SYmTo 11#Am úR§ zz éªûV úTôXúY RhTùYlT ¨ûXûV
LiP±VlThPÕ. ùLôiP Utù\ôÚ ¡WLUô] úL2#18©#«p
zz ùNqYôn ¡WLjûR YhPªhP UeLsVôu, ¿o BÚlTûR ®gOô²Ls LiP±kÕs[]o.
@eÏ ÅÑm ×Vp Lôh£Lû[ TPm T¥jÕ éª«-ÚkÕ 110 I°YÚP çWj§p BÚdÏm
éªdÏ @àl© @Nj§VÕ. @úRúTôX, úL2#18© ¡WLm, éªûV úTôp 8 UPeÏ
@ùU¬dLô®u SôNô @àl©V BuûNh ùT¬VÕ. BÕYûW LiP±VlThP ¡WLe
®iLXm, @iûU«p ùNqYôn ¡WLj§p L°úXúV D«¬]eLs Y£lTRtÏ Gt\
GtTÓm JûNLû[ éªdÏ @àl© Ds[Õ. RhTùYlT ¨ûXûV ùLôiÓs[, éª
zz Hd¡V @ÁWLjûRf úNokR @wUôu @pXôR IúW ¡WLm BÕYôÏm.
TpLûXdLZL UôQYoLs, ûYhPªu zz HR 5138 b F]l ùTV¬PlThÓs[ ªL
NjÕ Ïû\TôÓLû[ F°§p LiÓ©¥dL, ®£j§WUô] ÑtßYhPl TôûRÙPu
ûYhPô#úLm Fu\ ùUôûTp ùNV-ûV _þ©PÚdÏ @Ú¡p ÑZuß ùLôi¥ÚdÏm
Y¥YûUjÕ Ds[]o. קV úLô[ô]Õ LiÓ©¥dLlThÓs[Õ.
zz ùSRoXôkûR úNokR ×®«Vp YpÛSoLs @kR úLôs _þ©PûW Lôh¥Ûm 3 UPeÏ
HúWôlTô®u @¥«p Uj§V RûWdLPp ùT¬VÕ F] LQd¡PlThÓs[Õ.
©Wôk§Vj§p ¡úWhPo @h¬Vô F]l zz KùTúPôo Fàm ùTV¬p WxVô®u ØRp
ùTV¬PlThP IÚ ×§V LiPjûR U²R úWúTô úNôVv Fm.Fv#14 ®iLXj
LiÓ©¥jÕs[]o. BkR ¡úWhPo @h¬Vô §u êXm ùYt±LWUôL ®iùY°dÏ
¾Ü FuTÕ ¡ÃuXôk§u @[ûY ùLôiP @àlTlThÓs[Õ.
LiPj RLh¥u IÚ TϧVôÏm. zz ØmûT TpLûXdLZLj§u úTWô£¬ûVVô]
zz ã¬V ÏÓmTj§tÏ @lTôp Ds[ Lô£úVô¥Vô ûY`ô-, Fq®R WNôV]Øm úNodLôUp,
®iÁu ÏÝ®p IÚ ùY°dúLôû[ ÑUôo êuß AiÓLs YûW ùLhÓl
AnYô[oLs LiP±kÕs[]o. ãlTo Foj úTôLôUp BÚdÏm YûL«Xô] Bh-ûV
F] @ûZdLlTÓm BÕ éª«u ¨û\ûV LiÓ©¥jÕs[ôo. BRtLôL FXdhWô²d
®P 5 UPeÏ @§L ¨û\ÙûPVÕ. Àm úW¥úV`u Fàm ùRô¯pÖhTjûR
zz Yô²Vp AnYô[oLs ®VôZu úTôu\ TVuTÓj§Ùs[ôo.
IÚ YôÙ ¨û\kR úLô[ô] GJ 3512b F] zz ùRô¯p ¨ßY]eL°p TX T¦LÞdÏ
ùTV¬PlThP IÚ úLôû[ LiP±kÕs[]o. TVuTÓm YûL«Xô] SÅ] ùRô¯pÖhTe
BÕ ®VôZ²u ¨û\«p Tô§ûV ùLôiÓs[ LÞPu á¥V ¡Wôvl úUu F]l ùTV¬Pl
¨û\ÙPu á¥V IÚ ªLlùT¬V úLô[ôÏm. ThÓs[ úWôTôhûP ùNuû] HH¥ AWônf£
Vô[oLs Y¥YûUjÕs[]o.

 A-Series in
          ©lWY¬ 2020
zz SôNô®u ¡ë¬Vô£h¥ úWôYo ®iLXm, ùTôÚû[ DÚYôd¡Ùs[]o.
ùNqYôn ¡WLj§p TiûP LôXj§p
zz Ï_Wôj ÑtßfãZp Lp® Utßm AWônf£
¿úWôûPLs Utßm G¬Ls ¨û\kR Tϧ«p
@\dLhPû[«u AWônf£ @±Oo ÕÚq
L°Ui RôÕdLû[ LiP±kÕs[Õ.
©W_ôT§, úLW[ô, RªrSôÓ Utßm Ï_Wôj§p
zz 512 YVÕûPV DX¡u ªL اokR Ñ\ô YPdÏ LôQlTÓm BWiÓ ×§V £Xk§ B]eLû[
@hXôi¥d LPp Tϧ«p LiÓ©¥dLl LiÓ©¥jÕs[ôo. IÚ £Xk§ B]j§tÏ
ThÓs[Õ. Bk§V ¡¬dùLh ÅWo Nf£u ùPiÓpL¬u
zz Sôú]ôºÅh Fu\ ùTV¬p DX¡úXúV ùTVûW UúWeúLôNf£u ùPiÓpLo
ùUp-V ReLjûR Be¡Xôk§u Äh Fußm, Utù\ôuß ×²R ϬVúLôv F-Vôv
TpLûXdLZLj§u AWônf£Vô[oLs NYWô®u ùTVWôp BkúRôUùWeúLô
DÚYôd¡Ùs[]o. NYWThPo Fußm ùTV¬PlThPÕ
zz DP-p Tôd¼¬Vô TWÜRûX LiP±Ùm zz SôNô R]Õ ØRp @û]jÕ ªuNôW úNôRû]
Ïû\kR ùNX®Xô], F°V ûLVPdL ®Uô]eLû[Ùm X#57 úUdvùYp Fu\
LÚ®ûV @Nôm Uô¨Xm LÜLôj§«p Ds[ ùTV¬p @±ØLlTÓj§Ùs[Õ. BjRô-V
Bk§V ùRô¯pÖhT Lp® ¨ßY]j§u ùPd]m P 2006 T ®Uô]j§-ÚkÕ RÝ®
AWônf£Vô[oLs DÚYôd¡Ùs[]o. ¡hPjRhP 2 RNôlReL°p BÕ SôNô®u
ØRp ÏÝ X úNôRû] ®Uô]m AÏm.
zz BÕYûW LiP±VlTPô§ÚkR êû[«Ûs[
Uû\®Pm Iuß RtúTôÕ LiP±Vl zz £ùUuh LhPPeL°p GtTÓm SôsTP
ThÓs[Õ. ùYßm 2 ªp-ÁhPo @[úY ®¬NpLû[ RÓdL, @]p ªu ¨ûXVeL°p
DûPV ϱjR Tϧ ùRôPoTô] AWônf£Ls ÅQôÏm ¨XdL¬ NômTûX ûYjÕ Öi
ùRôPokÕ SûPùTtß YÚYRôL ®gOô²Ls DÚiûPL[ôL Ad¡, @Ytû\ £ùUuh
ùR¬®jÕs[]o. ÑY¬u ÁÕ éNXôm F] @ùU¬dLô®Ûs[
zz HH¥ HRWôTôjûR úNokR AWônf£Vô[oLs ¥ùWdùNp TpLûXdLZL ®gOô²Ls
Tô-p LXlTPjûR LiP±YRtLô] LiP±kÕs[]o.
vUôohúTôu NôokR ùNuNôûW DÚYôd¡ zz úRôp ÁÕ ã¬V I°TÓYRtÏm, Sm ÏPÛdÏs
Ùs[]o. BYoLs LiÓ©¥jÕs[ Øû\«u Y£dÏm XhNdLQdLô] SpX ÖiÔ«¬
êXm IÚ Lôh¥ Lô¡RjûR TVuTÓj§ LÞdÏm úSW¥ ùRôPo× BÚlTûR, ØRpØû\
Tô-p LXkÕs[ @ªXjRuûU«u @[ûY VôL L]PôûYf úNokR ®gOô²Ls
LiÓ©¥dLXôm. LiP±kÕs[]o. ã¬V L§¬Ûs[ ×\ERô
L§oLs, úRô-u ÁÕ TÓmúTôÕ, ûYhPªu #
SYmTo ¥, Sm DP-p DtTj§Vô¡\Õ. ÏP-p Ds[
zz DX¡úXúV @§L @[Ü JûNûV FÝl×m ÖiÔ«¬Ls Tp¡l ùTÚÏYRtÏ, BkR
T\ûYûV ®gOô²Ls LiP±kÕs[]o. ûYhPªu # ¥ ªLÜm @Y£Vm.
ùYiU¦l T\ûY (×úWd²Vôp ApTv)
zz HúWôl©V ®iùY° ¨ßY]m _lTôu
B]j§u Ai T\ûYLs FÝl×m JûN, Wôd
GúWôvúTv Fdvlú[ôúWNu ¨ßY]j
BûNd LfúN¬L°p ùY°lTÓm BûWfNp,
ÕPu BûQkÕ ùNVpTÓjÕm ©l©
UWm ùYhÓm BVk§W WmTm A¡VYtû\®P
ùLôXmúTô ª`²u úNôRû] ØVt£ ùYt±
@§L BûWfNûXd ùLôiPRôL BÚlTRôL
ùTtßs[Õ. BÕ ×Rû] AnÜ ùNnYRtLôL
D«¬VXô[oLs T§Ü ùNnÕs[]o.
@àlT®tÏm BÚ ùNVtûLúLôsLû[
zz @ùU¬dLô®Ûs[ YPdÏ LúWôûX]ô FnÕYRtLô] ª`u FuTÕ Ï±l©PjRdLÕ.
Uô¨X TpLûXdLZLjûRf úNokR
zz ©¬hPû]f úNokR F¥uTod TpLûXdLZL
®gOô²Ls, DúXôLm, £ùUuh, ©[ôv¥d
®gOô²Ls, úRôhPj§p, éfùN¥Lû[
Fuß FkRl TWl©u ÁÕ ûYjRôÛm,
TWôU¬dÏm §\u ùLôiP úWôúTô
@hûPl éf£ úTôX Ih¥dùLôsÞm

©lWY¬ 2020 A-Series in


  
Iuû\ DÚYôd¡Ùs[]o. h¬mTôh Fuß AWônf£Vô[oLs, úLôdNôd¡ ûYWv Fu\
@ûZdLlTÓm BkR úWôúTô, úRôhPj§u NôRôWQ N°ûV DiPôdÏm ûYWvLû[
YûWTPjûR U]lTôPm ùNnÕ, SôuÏ ùNÛj§, £ß¿olûT ×tßúSôûV ÏQlTÓj
NdLWeLÞPu YXm YÚ¡\Õ. BRu Itû\d ÕYÕ Ï±jÕ AWônkÕ YÚ¡u\]o.
LWm, édLû[ £ûRdLôUp, T±dLÜm, ×RoLû[
zz Fv#400 WL Yôu TôÕLôl× GÜLûQLs,
ùYh¥, Y¥YûUdLÜm YpXÕ.
Bk§Vô®Pm §hPªhPT¥ IlTûPdLlTÓm
zz vùT«û]f úNokR, ×úWôhPôu ¨ë F]o´ Fuß WxV @§To ®[ô§Áo קu
©ëfNo IÚ, Nd§ YônkR úUûNûV ùR¬®jRôo. WxVô®PªÚkÕ 43 ©p-Vu
DÚYôd¡«Úd¡\Õ. B # úTôoÓ BkR úUûN PôXo U§l©p (ì.39,000 úLô¥) Fv#400 Yôu
êXm, @ûXúT£, U¥dL¦², TXûLd L¦² TôÕLôl× GÜLûQLû[ ùLôsØRp ùNnYÕ
A¡VYtû\ ªuú]t\m ùNnÕ ùLôs[Xôm. ùRôPoTôL, BÚ SôÓLÞdÏm BûPúV LPkR
úUûN«u úUtTWl©p ã¬V ªu RLÓ AiÓ IlTkRm ûLùVÝjRô]Õ.
T§dLlThÓs[Õ.
¥NmTo
zz XiP²p Ds[ Wô¦ úU¬ TpLûXdLZL
AWônf£Vô[oLs Tô-Uo ©-mLs êXm zz ùTúWôlNWôu Bi¥Lm F]l ùTV¬PlThÓs[,
קV ûP FùXdh¬d ùLTô£Po LÚ® Bk§Vô®u ØRp £d]p Áu XhNj¾Ü
Iuû\ DÚYôd¡Ùs[]o. B§p Tô-Uo LP-p LiÓ©¥dLlThÓs[Õ.
©-mLs, ûP FùXdh¬d ùLTô£PûW Ñt± TX zz @ùU¬dLô®Ûs[ J¬Lu TpLûXdLZL
@ÓdÏL[ôL Ñt\lThÓ @ÝjRlThÓs[]. ®gOô²Ls, ØRuØRXôL Tp §ÑdLû[
zz ©-lûTuûNf úNokR íúNôu ¾®p, LiQô¥ £pÛdÏs T§jÕ NôRû]
IÚ ×§V U²R B]j§u FÛm×Ls ׬kÕs[]o. BkR £pÛdÏs Öi ¡ÚªLû[
LiÓ©¥dLlThÓs[]. úaôúUô ùNÛj§, Tp ùNôjûRVôYRtÏ, Tp-u úUúX
ÛúNôù]£v Fuß ùTV¬PlThÓs[ Ds[ F]ôUp FlT¥, Gu Y¯®Ó¡\Õ
BkR U²R B]m, @kRj ¾®p, 50 A«Wm FuTûR AWôV Ø¥Ùm.
AiÓLÞdÏ Øu× Yôrk§ÚdL úYiÓm zz ¨ëVôod¡Ûs[ úLôph v©¬e AnYLjûR
F] ®gOô²Ls U§l©hÓs[]o. úNokR ®gOô²Ls ØRpØû\VôL
zz éªûVl Tt± Õp-VUô] ×s°®YWeLû[ SLol×\ úY[ôiûUj RdLô° Iuû\
@±VÜm Col× ×XjûR Õp-VUôL DÚYôd¡Ùs[]o. YZdLUô] ùN¥VôLúYô,
@[®PÜm úLô[ Y¥®Xô] LiQô¥ TPo ùLô¥ @pXÕ Gß ùLô¥VôLúYô
ùNVtûLdúLôû[ @àlT WxV ®iùY° BpXôUp, BkRj RdLô°, ×Ro ùN¥ úTôX
ûUVm §hPªhÓs[Õ. ©-hv#Fm Ïß¡V BPj§p Y[od¡\Õ. §WôhûN úTôX
F]lTÓm BkR LiQô¥ ùNVtûLdúLôs ùLôjÕd ùLôjRôLd Lônd¡\Õ.
WxVô®u ©-hv ùNVtûLdúLô°p BÚkÕ zz LiL°u ®¯j§ûW«p I° @ÓdÏLs
úUmTÓjRlThPRôÏm. Tô§dLlThÓ Tô§ @[®p Li TôoûY
zz LôoTu ÏYôiPm Pôhv Fu\ ùTV¬p BZl×Ls GtTÓ¡u\]. BûR N¬ ùNnÙm
¨XdL¬«-ÚkÕ RVô¬dLlTÓm D«o UÚjÕY An®p @ùU¬dLô®u vPôuúTôoÓ
ÕLsLû[ TVuTÓj§ úLuNo ùNpLû[ TpLûXdLZLjûR úNokR ®gOô²Ls
LiP±YRtLô] ùRô¯pÖhTjûR @±®Vp CÓTh¥ÚkR]o. RtúTôÕ @YoLs ùNVtûL
Utßm AWônf£ LÜu£p YP¡ZdÏ @±®Vp ®¯j§ûWûV DÚYôd¡ @Ru êXm
ùRô¯pÖhTjûR úNokR AWônf£Vô[oLs BZkR Li TôoûYûV ùT\ Ø¥Ùm F]
LiÓ©¥jÕs[]o. LiÓ©¥jÕs[]o.

zz ©¬hP²u NoúW TpLûXdLZLjûRf zz BkúRôú]£Vô®u úUtÏ ÑUjWô ¾®p


úNokR aoúRq TôkRô®u RûXûU«Xô] DX¡úXúV ªLlùT¬V é UXokÕs[Õ.

 A-Series in
          ©lWY¬ 2020
4 @¥ @LXj§tÏ ©WUôiP úRôt\ Nôm©Vu ThPm ùYu\ôo.
j§p Ds[ BkR é®u ùTVo WlúX£Vô zz Av§úW-Vô®p ÑtßlTVQm úUtùLôiP
@o]ôp¥. DX¡p BÕYûW éjR UXoL°p Bk§V ¡¬dùLh @¦ 4 úTôh¥Ls ùLôiP
BÕúY ªLlùT¬VÕ Fuß RôYW®Vp ùPvh ùRôP¬p TeúLtß ®û[Vô¥VÕ.
®gOô²Ls ùR¬®jÕs[]o. B§p Bk§V @¦ 2#1 Fu\ LQd¡p ùYt±
zz SôNô®u ùLl[o YôuùY°j ùRôûXúSôd¡ ùTtß 72 AiÓLÞdÏ ©\Ï Av§úW-V
êXm ®gOô²Ls BÕYûW 4,034 ¡WLeLû[d Ui¦p ùPvh ùRôPûW ûLlTt± NôRû]
LiP±kÕs[]o. B§p éª úTôuú\ DÚYm TûPjRÕ.
Utßm RhTùYlT¨ûX ùLôiP ÑUôo 50 zz @ùU¬dLô®p SûPùTt\ _þ²Vo úLôpl
¡WLeLs YûW BÚlTRôL ùR¬®jR]o. @§p DXL Nôm©Vuµl úTôh¥«p Bk§VôûY
10 ¡WLeLs éªûVl úTôXúY ã¬Vû] úNokR 14 YVÕ £ßYu @o_þu Tô¥ ùYt±
Ñt±YÚYRôLÜm ùR¬®dLlThÓs[Õ. ùTtß NôRû] TûPjÕs[ôo.
zz LjRôo RûXSLo úRôLô®p, LjRôo JTu
®û[VôhÓLs ùPu²v ùRôPo SûPùTt\Õ. BRu
AiLs Itû\Vo ©¬Ü Bߧ úTôh¥«p
_]Y¬ vùT«²u WôToúPô Tô¥vPô ùNd Ï¥VW£u
RôUv ùTo¥fûN Årj§ Nôm©Vu ThPm
zz @ùU¬dLô®p Ds[ L-úTôo²Vô SL¬p ùYu\ôo.
NoYúRN NôXgf TôhªiPu ùRôPo zz T§]ôuLôYÕ DXL úLôlûT aôd¡
SûPùTt\Õ. BRu AiLs Itû\Vo ©¬Ü úTôh¥Ls I¥Nô Uô¨X RûXSLWô] ×Yú]
Bߧ AhPj§p _lTô²u úLô¡ YôPô]úT, vY¬p SûPùTt\Õ. BRu Bߧ úTôh¥«p
NLÅWo úLôPôn SúWôLôûY Årj§ Nôm©Vu ùTp´Vm ùSRoXôkûR Årj§ ØRpØû\
ThPm ùYu\ôo. VôL DXL úLôlûTûV ùYu\Õ.
zz D j § W © W ú R N U ô ¨ X m ù S ô n P ô ® p zz 2032 I-m©d úTôh¥ûV SPjR YWXôt±úXúV
ùTiLÞdLô] ×úWô úLôpl ùRôPo ØRpØû\VôL Bk§V I-m©d NeLm
SûPùTt\Õ. B§p Bk§V ÅWôeLû] ¬§Uô ®iQl©jÕs[Õ.
§XôYô¬ Nôm©Vu ThPm ùYu\ôo.
zz NoYúRN ¡¬dùLh LÜu£p ×LrùTt\
zz UúX£V RûXSLo úLôXôXmé¬p ë.Fv. ¡¬dùLh ÅWoLû[ aôp Al úTm Fu\
¡hv úLôpl DXL Nôm©Vuµl ùRôPo Th¥V-p úNojÕ LÜW®jÕ YÚ¡\Õ.
SûPùTt\Õ. B§p Bk§Vô®u ùSônPôûY RtúTôÕ BkR Th¥V-p Av§úW-V
úNokR @o_þu, 13#14 YVÕ ©¬®p Øu]ôs úLlPu ¬d¡ Tôi¥e úNodLl
ØR-PjûR ùTtß ReLlTRdLm ùYu\ôo. ThÓs[ôo.
zz NoYúRN ¡¬dùLh LÜu£p ùPvh ÅWoL°u zz DvùT¡vRô²p SPkR 2018 A£V _þ²Vo
קV RWY¬ûN Th¥VûX ùY°«hÓs[Õ. ùNv Nôm©Vuµl úTôh¥«p 18
B§p úThvúUuLs RWY¬ûN«p Bk§V YV§tÏhThP ùTiLÞdLô] ©¬®p
@¦ úLlPu ®Wôh úLôy- ØR-Pm Bk§Vô®u RVô² ThúPp BWiÓ ReLm
ùTtßs[ôo. TkÕ ÅfNô[oLs RWY¬ûN«p Utßm ùYiLX TRdLeLû[ ûLlTt±
ùRu]ôl©¬dLô®u Lô´úNô WTôPô ØR- NôRû] TûPjRôo.
Pm ùTtßs[ôo.
zz _mØ®p SPkR úR£V ùNv Nôm©Vuµl
zz UWôh¥V JTu NoYúRN ùPu²v úTôh¥ úTôh¥«p RªrSôhûP úNokR 19 YVRô]
×ú] SL¬p SûPùTt\Õ. BRu AiLs @W®kj £RmTWm ØR-Pm ùTtß úR£V
Itû\Vo ©¬Ü Bߧ AhPj§p ùNv Nôm©Vuµl ThPjûR ùYu\ôo.
ùRu]ôl©¬dLô®u ùL®u AiPoNu,
ÏúWôµVô®u BYô LôoúXô®fûN Årj§ zz 2018#Am Ai¥tLô] úR£V vÏYôx
Nôm©Vuµl úTôh¥«p AiLs ©¬®p

©lWY¬ 2020 A-Series in


  
UúLx UeLôquLo, ®dWm UpúaôjWôûY úUôúUôPôûY Årj§ R]Õ ØRXôYÕ
Årj§ ØRpØû\VôL Nôm©Vu ThPm BkúRôú]µV JTu ThPjûR ûLlTt±]ôo.
ùYu\ôo. ùTiLs ©¬®p ú_ôx]ô zz 2019 PôPô ØmûT UôWjRôu TkRVj§p
£u]lTô, EoY£ ú_ôµûV Årj§ Nôm©Vu AiLs ©¬®p ùLuVô®u LôvúUôv
ThPm ùYu\ôo. XLôhÓm, ùTiLs ©¬®p Fj§úVôl©Vô®u
zz @ùU¬dLô®u úVp TpLûXdLZLj§p ùYôodúSx @ùXØÜm Nôm©Vu ThPm
SûPùTt\ DXL _þ²Vo YôRj§\ûUdLô] ùYu\]o. TkRV çWm 42.195 ¡.Á. AÏm.
Nôm©Vuµl úTôh¥«p ùYußs[ ØRp
zz ùYvh Bi¼£p ÑtßlTVQm úUtùLôiP
Bk§Vo Fàm ùTÚûUûV ùNuû]ûV
Be¡XôkÕ ¡¬dùLh @¦, 3 úTôh¥Ls
úNokR 13 YVÕ £ßYu A§ Nôn ®_n¡Wi
ùLôiP ùPvh ùRôP¬p TeúLtß
ùTtßs[ôo.
®û[Vô¥VÕ. B§p ùYvh Bi¼v 2#1
zz Uj§V ©WúRN ¡¬dùLh ÅWo @_n ùWôúaWô Fu\ LQd¡p ùYt± ùTtß ùRôPûW
ØRp RW ¡¬dùLh¥p @±ØL AhPj§p 267 ûLlTt±VÕ.
WuLs FÓjÕ LûP£ YûW AhPªZdLôUp
zz Av§úW-V JTu ùPu²v ùRôPo
BÚkÕ DXL NôRû] TûPjRôo. Bkç¬p
ùUpúTôou SL¬p SûPùTt\Õ. BRu
ûaRWôTôj @¦dùL§Wô] Wg£ AhPj§u
AiLs Itû\Vo ©¬Ü Bߧ AhPj§p
úTôÕ @Yo BkR NôRû]ûV TûPjRôo.
vùT«²u WúTp SPôûX, ùNo©Vô®u
zz NoYúRN RPL[ áhPûUl× (International úSôYd ú_ôúLô®f úSo ùNhL°p F°RôL
Association of Athletics Federations (IAFF)) ùYuß GZôYÕ Øû\VôL Av§úW-V
2023#Am DXL Nôm©Vuµl úTôh¥Ls JTû] ùYu\ôo.
aeúL¬V RûXSLo ×PôùTv¥p SûPùTßm
F] @±®jÕs[Õ. zz ùSRoXôk§p Ds[ ®wd SL¬p PôPô v¼p
UôvPov ùNv ùRôPo SûPùTt\Õ. B§p
zz ¥#20 DXL úLôlûT ®û[VôhÓL°u
SPl× DXL Nôm©Vu SôoúY«u LôopNu
ùTVWô] World T20 FuTÕ T20 World Cup
9 ×s°LÞPu ØR-PjûR ©¥jÕ 7#YÕ
F] NoYúRN ¡¬dùLh LÜu£-]ôp ùTVo
Øû\VôL Nôm©Vu ThPjûR ûLlTt±]ôo.
Uôt± @±®dLlThÓs[Õ.
zz Av§úW-Vô®u £h² ûURô] ùL[WY
zz @où_u¥]ô®p SûPùTt\ DXL ¡d Tôd£e
Dßl©]o Fu\ £\lûT Bk§V ¡¬dùLh
Nôm©Vuµl úTôh¥«p LôxÁûW úNokR 15
ÅWo ®Wôh úLôy- Utßm T«t£Vô[o
YVÕ UôQYu A©j aªj ReLm ùYuß
W® Nôv§¬ A¡úVôo ùTtßs[]o.
NôRû] TûPjRôo.
zz NoYúRN ¡¬dùLh úTôh¥«p ªL ®ûWYôL zz B k § V ¡ ¬ d ù L h @ ¦ ¨ ë £ X ô k § p
19,000 WuLs FÓjR ÅWo Fu\ DXL ÑtßlTVQm úUtùLôiÓ 5 úTôh¥Ls
NôRû]ûV Bk§Vô®u ®Wôh úLôy- ùLôiP IÚSôs ùRôP¬p TeúLtß
Av§úW-VôÜdÏ F§Wô] £h² ùPvh ®û[Vô¥VÕ. B§p £\lTôL ®û[Vô¥V
úTôh¥«u úTôÕ TûPjRôo. BRu Bk§V @¦ 4#1 Fu\ LQd¡p ùRôPûW
êXm Nf£u ùPiÓpL¬u NôRû]ûV ûLlTt±VÕ.
رV¥jÕs[ôo. zz Av§úW-Vô®u ùToj SL¬p SûPùTt\
2019#Am Ai¥u úaôlUu úLôlûT
©lWY¬ ùPu²v úTôh¥«u Nôm©Vu ThPjûR
zz BkúRôú]µVô RûXSLo _LôojRô®p Ñ®hNoXôkÕ @¦ ùYußs[Õ.
BkúRôú]µVô JTu TôhªiPu ùRôPo zz @ùU¬dLô®u £Vôh¥p SL¬p NoYúRN
SûPùTt\Õ. BRu AiLs Itû\Vo vÏYôx JTu ùRôPo SûPùTt\Õ. BRu
©¬Ü Bߧ AhPj§p ùPuUôod¡u Bߧ AhPj§p Bk§Vô®u Wªj PôuPu,
AiPov @uPuùNu, _lTô²u ùLuúPô F¡l§u ØLUÕ Fp.ù`o©²ûV Årj§ R]Õ

 A-Series in
          ©lWY¬ 2020
4#YÕ Nôm©Vu ThPjûR ûLlTt±]ôo. Uôof
zz ÕÚd¡«u BvRôu×p SL¬p SûPùTt\ zz úPWôå²p SûPùTt\ @VoXôkÕdÏ F§Wô]
AyùUh úLôùUoh ÏjÕfNiûP ùRôP¬p NoYúRN ¥20 úTôh¥«p AlLô²vRôu
Bk§V ÅWoLs 3 ReLm, 2 ùYs°, 2 @¦ 278 Ï®jÕ DXL NôRû] TûPjRÕ.
ùYiLXm F] ùUôjRm 7 TRdLeLû[ BRtÏ Øu× Av§úW-V @¦ BXeûLdÏ
ùYu\]o. F§WôL 263 WuLs FÓjRúR NôRû]VôL
zz 2020 úPôd¡úVô I-m©d úTôh¥LÞdLô] BÚkÕ YkRÕ.
ØRp Rϧf Ñtß AhPUôL Ad¡ ùRôPo
zz LjRôo Sôh¥u úPôLô SL¬p SûPùTt\
(AiLs) 2019 _þu 6 ØRp 16 YûW I¥Nô
2019#Am Ai¥tLô] LjRôo JTu
Uô¨Xm ×Yú]vY¬p SûPùT\Üs[Õ.
ùTiLs NoYúRN ùPu²v úTôh¥«p
zz 36#YÕ úR£V ®û[VôhÓ úTôh¥Ls 2019 F-v ùUoùPuv (ùTp´Vm) £úUô]ô
Uôof, GlWp UôReL°p úLôYô®p SûPùT\ aôùXl#ûV (ÚúU²Vô) Årj§ Nôm©Vu
Ds[]. ThPjûR ùYußs[ôo.
zz BjRô- ãlTo úLôlûT LôpTkÕ úTôh¥«u
zz ùLuVô RûXSLo ûSúWô©«p _þ²Vo
Bߧ úTôh¥«p G£ ªXû] Årj§
NoYúRN JTu TôhªiPu ùRôPo
_ýùYuhv @¦ Nôm©Vu ThPm ùYu\Õ.
SûPùTt\Õ. BRu AiLs Itû\Vo ©¬Ü
zz ùNuû]«p SûPùTt\ ùNuû] JTu Bߧ AhPj§p Bk§Vô®u WôÏp TWjYôw,
ùNv ùRôP¬p _ôo´Vô ¡WôihUôvPo NL ÅWWô] @Uu TúWôdûL Årj§ Nôm©Vu
TuhÑûXVô ùXYu Nôm©Vu ThPm ThPjûR ûLlTt±]ôo.
ùYu\ôo.
zz ×ÕùPp-«p SûPùTt\ DXLdúLôlûT
zz NoYúRN ¡¬dùLh LÜu£p UL°o IÚSôs ÕlTôd¡ ÑÓRp úTôh¥«p, 10 ÁhPo Go
¡¬dùLh RWY¬ûN Th¥V-p Bk§V ûW©s ©¬®p Bk§Vô®u @éo® Ni¥Xô
ÅWôeLû] vªÚ§ UkRô]ô ØR-PjûR DXL NôRû]ÙPu ReLm ùYußs[ôo.
©¥jÕ NôRû] TûPjÕs[ôo. Wô_vRôu Uô¨XjûRf úNokR BYo 252.9
zz DXL ùPu²v ÅWo#ÅWôeLû]L°u ×s°LÞPu BkR NôRû]ûV TûPjÕs[ôo.
RWY¬ûN Th¥VûX NoYúRN ùPu²v APYo 10Á Go©vPp ©¬®p 16 YVRô]
NeLm ùY°«hÓs[Õ. B§p ùNo©Vô®u Dj§Wl ©WúRN Uô¨XjûR úNokR Bk§V
úSôYd ú_ôúLô®f ØR-PjûR ©¥jÕs[ôo. ÅWo ùN[Wq ùN[j¬ 245 ×s°Ls Ï®jÕ
Bk§V ÅWo ©Wwú]x Ïuú]vYWu 97#YÕ ×§V DXL NôRû]ÙPu ReLm ùYußs[ôo.
BPjûR ©¥jÕs[ôo.
zz Bk§Vô®u NÕWeL ¡Wôih UôvPWô]
zz úUtÏ Al©¬dLô®Ûs[ ùN]Lp Sôh¥p @©´j ÏlRô úLuv NoYúRN JTu
2022#Am AiÓ Bû[úVôo I-m©d NÕWeL úTôh¥«p (Cannes International Open
®û[VôhÓ úTôh¥Ls SûPùT\ Ds[Õ. trophy) 7.5 ×s°Ls ùTtß Nôm©Vu ThPm
zz L_LvRôu RûXSLo @vRô]ô®p ùTiLÞd ùYußs[ôo.
Lô] ùTPúW`u úLôlûT ùPu²v ùRôPo zz CWô²u NôTaôo SL¬p SûPùTt\ UdWôu
SûPùTt\Õ. B§p L_LvRôu, Bk§V úLôlûTdLô] ÏjÕfNiûP úTôh¥«u
@¦Ls úUô§]. B§p L_LvRôu @¦ Bߧf Ñt±p 49 ¡úXô FûPl©¬®p
3#0 Fu\ LQd¡p ùRôPûW ûLlTt±VÕ. Bk§Vô®u ¾Td £e ReLl TRdLm
zz úRôLô®p SûPùTt\ LjRôo JTu ùPu²v ùYußs[ôo.
úTôh¥«p DX¡u Øu]ôs SmTo 1
zz TgNôl @¦dÏ F§Wô] 47#YÕ úR£V
ÅWôeLû] £úUô]ô aúXlûT Årj§
º²Vo AiLs úaihTôp Nôm©Vuµl
ùTp´Vj§u Fp³ ùUoPuv Nôm©Vu
úTôh¥«p WôÔY @¦ ùYt± ùTtß
ThPm ùYu\ôo.

©lWY¬ 2020 A-Series in


  
Nôm©Vu ThPjûR ûLlTt±Ùs[Õ. Av§úW-V @¦dÏ F§WôL 514 WuLs
zz Õ T ô « p S û P ù T t \ A i L Þ d L ô ] FÓjÕs[ôo.
ÕTôn NoYúRN ùPu²v úTôh¥«p zz IÚ Sôs ¡¬dùLh úTôh¥«p IÚ Bu²e£p
Ñ®hNoXôkûRf úNokR úWô_o ùTPWo @¥dLlTh @§LThN £dNoLs (24) @¥jR
Nôm©Vu ThPjûR ùYußs[ôo. BRu @¦ Fu\ NôRû]ûV Be¡XôkÕ ¡¬dùLh
êXm, úWô_o ùTPWo Itû\Vo ©¬®p 100#YÕ @¦ ùTtßs[Õ. ùYvh Bi¼v @¦dÏ
NoYúRN ThPm ùTt\ BWiPôYÕ ÅWo Fu\ F§WôL BkR NôRû]ûV Be¡XôkÕ TûPjRÕ.
NôRû]ûVl TûPjÕs[ôo.
zz TpúL¬Vô NoYúRN UpÙjR úTôh¥ 2019#Bp zz WxVô®u ùN«uh ÀhPoTod SL¬p
APYo 65 ¡úXô lÃvûPp ©¬®p Bk§V SûPùTt\ WTA ùPu²v úTôh¥«p
ÅWo TwWôe ײVô @ùU¬dLô®u ú_ôoPôu ùSRoXôkÕ ÅWôeLû] ¡d¡ ùTohPuv,
@-YûW Årj§ ReL TRdLm ùYu\ôo. ÏúWôµVô®u úPô]ô úY¡dûL Årj§
Nôm©Vu ThPm ùYu\ôo.
zz º]ô®u aôeÌ SL¬p YÚm 2022#Am
AiÓ SûPùT\Üs[ A£Vl úTôh¥«p GlWp
ØRuØû\VôL JúN²Vô SôÓLs úNodLl
ThÓs[]. JúN²Vô SôÓLs FuTûY zz TôoØXô 1 LôoTkRVj§u ØRXôYÕ Ñt\ô]
Av§úW-Vô, ¨ë³XôkÕ, TÑ©d ùTÚeLPp Av§úW-V ¡Wôih©¬, ùUpúTôou
¾ÜLs A¡VYtû\ Ds[Pd¡VÕ. SL¬p SûPùTt\Õ. TkRV çWUô] 307.574
¡úXôÁhPo BXdûL ©uXôkÕ ÅWo YôpùP¬
zz H££ ùY°«hÓs[ ùTiLÞdLô] IÚ úTôhúPôv (ùUo£Pv @¦) 1 U¦ 25
Sôs ¡¬dùLh RWY¬ûN«p Bk§V ShNj§W ¨ªPm 27.35 ®]ô¥L°p LPkÕ ØR-Pm
ÅWôeLû] vm¬§ UkR]ô ØR-PjûR ùTt\ôo.
ùTtßs[ôo. TkÕ ÅfNô[oL°u RWY¬ûN«p
Bk§V ÅWôeLû] _ýXu úLôvYôª ØR- zz ¡WôihvXôm úTôh¥L°p Iu\ô] ©ùWgf
PjûR ©¥jÕs[ôo. JTu ùPuÉv úTôh¥Vô]Õ TôÃv SL¬p
úU 26#Bp ùRôPeÏ¡\Õ.
zz ©uXôk§p SûPùTt\ ´.© NoYúRN ÏjÕf
NiûP úTôh¥«p Bk§Vô 1 ReLm, 4 ùYs° zz aôeLôe¡p SûPùTt\ A£V Bû[úVôo
TRdLeLû[ ùYußs[Õ. BlúTôh¥«p RPL[ Nôm©Vuµl úTôh¥«u UL°ÚdLô]
APYo 56 ¡úXô FûPl©¬®p L®kRo £e ¿[m RôiÓRp úTôh¥«p RªrSôhûPf
©xh ReLm ùYußs[ôo. úNokR R©jRô ReLm ùYußs[ôo. BYo
BlúTôh¥«p BWiÓ ReLl TRdLeLû[
zz ùYvh Bi¼v ¡¬dùLh ÅWo ¡±v ùYu\ôo. BRtÏ Øu 100ÁhPo RûP RôiÓm
ùLnp IhÓùUôjR NoYúRN ¡¬dùLh¥p úTôh¥«p R]Õ ØRp ReLl TRdLjûR
(20 JYo, IÚ Sôs Utßm ùPvh) @§L ùYußs[ôo.
£dNo @¥jRYo (477 £dNo) Fu\ NôRû]ûV
TûPjÕs[ôo. zz úSTô[ Sôh¥u ©Wôh SL¬p ùRtLô£V
ùTiLs LôpTkÕ úLôlûT 2019 SûPùTt\Õ.
zz 2022#Am AiÓ úULôXVô Uô¨Xj§p 6 @¦Ls TeúLt\ BkR ùRôP¬u Bߧ
SûPùTßm 39#YÕ úR£V ®û[VôhÓ AhPj§p Bk§V @¦, úSTô[ @¦ûV
úTôh¥dLô] @§LôWléoY £u]UôL 3#1 Fu\ úLôp LQd¡p Årj§ Nôm©Vu
úULdL¦kR £ßjûR (Clouded Leopard) ThPm ùYu\Õ.
úRoÜ ùNnVlThÓs[Õ.
zz @ùU¬dLô®p SûPùTt\ Bu¥VuùYpv
zz NoYúRN ¥20 ¡¬dùLh¥p ϱl©hP @¦dÏ NoYúRN ùPu²v úTôh¥«p, AiLs
F§WôL 500 WuLs LPkR ØRp ÅWo Fu\ Itû\Vo ©¬®p Av§¬Vô®p ùPôª²d
NôRû]ûV Bk§V @¦ úLlPu ®Wôh ¾m Ñ®hNoXôk§u úWô_o ùTPWo#H ùYuß
úLôy- TûPjÕs[ôo. ®Wôh úLôy- Nôm©Vu ThPm ùYußs[ôo. ùTiLÞdLô]

 A-Series in
          ©lWY¬ 2020
Itû\Vo ©¬®p L]Pô®u ©VôuLô NoYúRN ¿fNp úTôh¥ SûPùTt\Õ. B§p
AuhÃvÏ ù_oU²«u GgN-d ùLoTûW 200 ÁhPo úTd vhúWôd ©¬®p Hd¡V @W×
ùYuß Nôm©Vu ThPjûR ùYu\ôo. FªúWhv Ts°«p T«Ûm 13 YVÕ RªZL
zz DXL ùPu²v ÅWo, ÅWoL°u RWY¬ûNl UôQYo ®úNx TWúUvYWu TkRV çWjûR
Th¥VûX NoYúRN ùPu²v NmúU[]m 2 ¨ªPm 15 ®]ô¥«p LPkÕ ×§V NôRû]
NÁTj§p ùY°«hÓs[Õ. AiLs Itû\Vo ÙPu ReLl TRdLm ùYu\ôo.
RWY¬ûNl Th¥V-p ùNo©Vô®u úSôYd zz @ùU¬dLô®u ªVôª SL¬p SûPùTt\
ú_ôúLô®f ØR-Pj§Ûm, vùT«²u WúTp ªVôª NoYúRN JTu ùPu²v ùRôP¬u
SPôp BWiPôYÕ BPj§Ûm Ds[]o. BߧlúTôh¥«p Øu]ôs SmTo Iu ÅWWô]
ùTiLs Itû\Vo RWY¬ûN«p _lTô²u Ñ®hNoXôk§u úWô_o ùTPWo, @ùU¬dLô®u
SúYôª INôLô ØR-Pj§p Ds[ôo. _ôu Bv]ûW Årj§ 4#YÕ Øû\VôL
zz Hd¡V @W× @ÁWLj§u @×Rô©«p, Nôm©Vu ThPjûR ûLlTt±]ôo. ùTPWo, R]Õ
SûPùTt\ £\l× I-m©d DXL úLôûPdLôX ùPu²v YôrdûL«p ûLlTt±V 101#YÕ
®û[VôhÓl úTôh¥L°p 85 ReLl ThPm BÕ FuTÕ Ï±l©PjRdLÕ.
TRdLeLÞPu ùUôjRm 368 TRdLeLû[ zz U ú X £ V ô J T u T ô h ª i P u ù R ô P o
ùYuß Bk§Vô YWXôtßf NôRû] úLôXôXmé¬p SûPùTt\Õ. BRu AiLs
TûPjÕs[Õ. Itû\Vo ©¬Ü Bߧ AhPj§p º]ô®u
zz TpúL¬Vô NoYúRN UpÙjR úTôh¥#2019#Bp -u Pôu, ùNu XôeûL Årj§ Nôm©Vu
APYo 65 ¡úXô lÃvûPp ©¬®p Bk§V ThPm ùYu\ôo.
ÅWo TwWôe ײVô @ùU¬dLô®u ú_ôoPôu zz ûRYô²p SûPùTt\ A£V GoLu ÕlTôd¡f
@-YûW Årj§ ReL TRdLm ùYu\ôo. ÑÓRp úTôh¥«p Bk§V @¦ 16 ReLm,
zz 2019#Am AiÓ TôhªiPu A£Vd 5 ùYs°, 4 ùYiLXm F] ùUôjRm 25
LXl× @¦Ls Nôm©Vuµl úTôh¥L°u TRdLeLû[ ùYußs[Õ. LûP£ Sô°p, 10
2#YÕ T§lTô]Õ aôeLôe¡p SûPùTt\Õ. ÁhPo Go ûW©s _þ²Vo AiLs ©¬®p
B§p _lTôû] Årj§ º]ô Nôm©Vuµl Bk§Vô®u Vôx YoRu DXL NôRû]ÙPu
ThPjûR ùYu\Õ. ReLl TRdLm ùYu\ôo.

zz Bk§V APYo aôd¡ @¦«u RûXûU zz DXL vÏYôx ÅWoL°u RWY¬ûNl Th¥VûX
T«t£Vô[WôL Av§úW-Vô®u Øu]ôs DXL vÏYôx NmúU[]m ùY°«hÓs[Õ.
ÅWo ¡Waôm Ãh úRokùRÓdLlThÓs[ôo. B§p, AiLs Itû\Vo ©¬®p DXL
BYo 2022#Bp SûPùTßm DXLúLôlûT Nôm©V]ô] F¡lûRf úNokR @- TWôd
úTôh¥ YûW T«t£Vô[WôL T¦Vôt\ ØR-PjûRl ©¥jÕs[ôo. Bk§Vô®u NÜWq
Ds[ôo. úLô`p 10#YÕ BPjûRl ©¥jÕs[ôo. BRu
êXm ØRp 10 BPeL°p BPmùTt\ ØRp
zz 1 7 Y V § t Ï h T h P ù T i L Þ d L ô ] Bk§V ÅWo Fu\ ùTÚûUûV ùTtßs[ôo.
DXLdúLôlûT LôpTkÕ úTôh¥ 2020#Am
AiÓ Bk§Vô®p SûPùT\Üs[Õ. zz ùUd£úLô®u UôuùPo¬ SL¬p SûPùTt\,
BRtÏ Øu, LPkR 2017#Bp Bk§Vô U#17 UôuùPo¬ JTu NoYúRN ùPu²v úTôh¥
AiLÞdLô] DXLdúLôlûT LôpTkÕ 2019#Bp ùTiLs Itû\Vo ©¬®p
úTôh¥ûV SPj§VÕ. vùT«²u Lôo©u ØÏÚ_ô ùTXôWv
Sôh¥u ®dúPô¬Vô @^ùWuLôûY Årj§
zz TgNôl @¦dÏ F§Wô] 47#YÕ úR£V Nôm©Vu ThPm ùYu\ôo.
º²Vo AiLs úaihTôp Nôm©Vuµl
úTôh¥«p WôÔY @¦ ùYt± ùTtß zz CFv©Fu Fu\ @ùU¬dL ùRôûXdLôh£
Nôm©Vu ThPjûR ûLlTt±Ùs[Õ. ¨ßY]m @iûU«p ùY°«hÓs[
DXLl ×LrùTt\ 100 ÅWoL°u Th¥V-p
zz UúX£V RûXSLo úLôXôXmé¬p 55#YÕ ¡±v¥Vôú]ô ùWô]ôpúPô ØR-Pm

©lWY¬ 2020 A-Series in


  
úYß FkR BVo×d èpLÞm
TNPSC úRoÜdÏ BqY[Ü
ùTôÚjRUô] RLYpLû[ ùLôÓlT§pûX.
 LPkR FhÓ AiÓLs TNPSC úRoÜ ®]ôdL°p Sd¸Wu
BVo×d @°jR RLYpLs ªLf £\lTô]ûY.
 LPkR GÝ AiÓL°p (2012#2019) TNPSC úRoÜL°p
úLhLlThP ùTôÕ@±Ü, SPl× ¨LrÜLs, @±®Vp &
ùRô¯pÖhTm, ùTôÚ[ôRôWm, UdLsùRôûL, Ød¡V §hPeLs,
Ød¡V §]eLs Ds[Pe¡V RLYpLs @¥lTûP«p
ØÝûUVôL RVô¬dLlThPÕ.

Sd¸Wu BVo×d 2020 : ùTôÚ[PdLm
§hPeLs
 
Fp.©.´. Uô²V TôLp §hPm
©WRU¬u DwYôXô §hPm
úT¥ TfNôúYô úT¥ TRôúYô
RªZL @W£u קV §hPeLs
Sn Uu³p §hPm
¨¥ AúVôd ùRôûXúSôdÏj §hPm
NôLo UôXô §hPm
©WRôu Uk§¬ KT^p ÀU úVô_]ô
¡Úµ £uNôn §hPm
¿Xl ×Wh£dLô] קV §hPm
اúVôo Jnî§Vd LôlÀhÓj §hPm
ùRôPe¡Ó Bk§Vô §hPm
Nôdbm §hPm
@Pp ùTuNu úVô_]ô ¥´hPp Bk§Vô §hPm
ËYuú_ô§ ÀUô úVô_]ô TôWmT¬V úUmTôÓ Utßm ®¬YôdLj §hPm
ÑLuVô Nm¬§ úVô_]ô ¾uRVôs DTôjVôVô ¡Wôm ú_ô§ úVô_]ô
@Yôv úVô_]ô _]² £Ñ ÑWd`ô LoV¡Wôm
DfNRôo @®vLôo úVô_]ô ÑLôRôW §hPeLs
úUd Bu Bk§Vô §hPm ¨oTVô ¨§j §hPm
_uRu úVô_]ô קV LôlÀhÓj §hPeLs
_u ANô§ úVô_]ô Bk§Vô®p RVô¬lúTôm §hPm
ReL úNªl×j §hPm @mÚj §hPm
FÝkÕ ¨p Bk§Vô §hPm vUôoh £h¥
SUôª LeûL §hPm RªZL @W£u NêL SXj§hPeLs
DwYôXô §hPm UdLsùRôûL 2011
Advt.

çnûU Bk§Vô §hPm Ød¡V §]eLs

 A-Series in
          ©lWY¬ 2020
Sd¸Wu BVo×d 2020
 ®¬Yô] LhÓûWL[ôL
Ød¡V ¨LrÜLs, ùTôÚ[ôRôWm, NhPm, §hPm, @±dûL,
ùLôsûL, ®ÚÕLs, @±®Vp & ùRô¯pÖhTm Buàm . . .
 Uj§V#Uô¨X @W£u Ød¡V §hPeLs LhÓûWL[ôL
 úRoÜ úSôd¡p 2019 Ød¡V ¨LrÜLs, SPl× ¨LrÜLs DhTP
 RªrSôÓ, Bk§Vô, DXLm, ®ÚÕLs, ®û[VôhÓLs, @±®Vp
F] ®¬Yô] ùTôÕ @±Ü RLYpLÞPu . . .
 TNPSC úRoÜL°u @¥lTûP«p RVô¬dLlThPÕ.
×jRLm Tt±V ®YWeLÞdÏ . . .
 ùNuû], §ÚYsðo
¡±vúPôTo # 96770#81375,
@ÛYXLm # 044#4399 3029
 LPío, ùTWmTío, §Úf£, RgNôîo,
SôûL., ×ÕdúLôhûP, §ÚYôìo,
§iÓdLp, Lìo, @¬Vío
TWU£Ym # 8939882454
 UÕûW, úR², ®ÚÕSLo, BWôUSôR×Wm,
£YLeûL, çjÕdÏ¥, SôLoúLô®p,
§ÚùSpúY-
LkRNôª # 8939882455
 ®Ýl×Wm, Lôg£×Wm, §,UûX, úYío,
¡ÚxQ¡¬, RÚU׬, úNXm
LQT§ # 8939882456
 úLôûY, §Úléo, CúWôÓ,
SôUdLp, ¿X¡¬
044#4399 3029 1120 TdLeLs ®ûX ì. 160/#
úUÛm ®TWeLÞdÏ
Sd¸Wu Tl°úL`uv,
105, _ô² _ôuLôu NôûX, WôVlúThûP, ùNuû]#14
Advt.

SP#76, 14#YÕ ùRÚ, @mTjço ùRô¯túThûP, ùNuû] # 600 058


ùRôûXúT£: 044 # 2688 1800

©lWY¬ 2020 A-Series in


  
©¥jÕs[ôo. Bk§V ¡¬dùLh ÅWo ®Wôh zz vùT«u Sôh¥p 2019#Am AiÓdLô]
úLô- GZôYÕ BPm ©¥jÕs[ôo. Xô -Lô úLôlûT LôpTkÕ ùRôPo SûPùTt\Õ.
zz º]ô®u aôeÌ SL¬p YÚm 2022#Am BRu Bߧ AhPj§p Tôo£úXô]ô @¦,
AiÓ SûPùT\Üs[ A£Vl úTôh¥«p ùXYu¥ @¦ûV 1#0 Fu\ úLôp LQd¡p
ØRuØû\VôL JúN²Vô SôÓLs Årj§ Nôm©Vu ThPjûR ùYu\Õ.
úNodLlThÓs[]. JúN²Vô SôÓLs FuTûY zz º]ô®u ´Vôu SL¬p A£V UpÙjR
Av§úW-Vô, ¨ë³XôkÕ, TÑ©d ùTÚeLPp Nôm©Vuµl úTôh¥Ls SûPùTt\Õ. B§p
¾ÜLs A¡VYtû\ Ds[Pd¡VÕ. CWôu 11 ReLm, 6 ùYiLXm DsTP 17
zz NoYúRN ¡¬dùLh LÜu£p (ICC) IÚSôs TRdLeLÞPu ØRp BPjûRÙm, º]ô 5
úTôh¥L°u RWY¬ûNl Th¥VûX ReLm, 5 ùYs°, 6 ùYiLXjÕPu 2#YÕ
ùY°«hÓs[Õ.@¦L°u RWY¬ûN«p BPjûRÙm ©¥jR].
Be¡XôkÕ @¦Vô]Õ ØR-Pm ©¥jÕs[Õ. zz º]ô®u À´e SL¬p DXL úLôlûT ÕlTôd¡
Bk§Vô BWiPôYÕ BPj§p Ds[Õ ÑÓRp ùRôPo SûPùTt\Õ. B§p TeúLt\
Bk§V @¦ 3 ReLm, IÚ ùYs° F]
úU ùUôjRm 4 TRdLeLû[ ùYuß ØR-PjûR
©¥jRÕ.
zz B k § V ô û Y f ú N o k R T w W ô e × ² V ô
AiLÞdLô] 65 ¡úXô¡Wôm lÃvûPp zz UúX£V RûXSLo úLôXôXmé¬p, A£V
©¬Üj RWY¬ûN«p DX¡p ØR-PjûRl R²STo vÏYôx Nôm©Vuµl ùRôPo
©¥jÕs[ôo. SûPùTt\Õ. B§p ùTiLÞdLô] Bߧ
úTôh¥«p SPl× Nôm©Vu Bk§Vô®u
zz º]ô®u îaôu SL¬p A£V TôhªiPu
ú_ôx]ô £u]lTô, aôeLôe¡u Au²
Nôm©Vuµl ùRôPo SûPùTt\Õ. BRu
@ÙûY Årj§ 2#YÕ Øû\VôL Nôm©Vu
AiLs Itû\Vo ©¬Ü Bߧ úTôh¥«p
ThPjûR ûLlTt±]ôo.
_lTô²u ùLuúPô ùUúUôhPô, º]ô®u
Ì V¡ûV Årj§ Nôm©Vu ThPm ùYu\ôo. zz H©Fp 2019 ùRôP¬u Bߧl úTôh¥
HRWôTôj§p SûPùTt\Õ. B§p ØmûT
zz ùUô]ôdùLô®p SûPùTt\ UôuúPô LôoúXô
Bk§Vuv @¦Ùm, ùNuû] ãlTo ¡ev
UôvPov ùPu²v úTôh¥«u Bߧ
@¦Ùm TXlTÃhûN SPj§]. TWTWlTôL SPkR
AhPj§p BjRô- ÅWo úT©úVô úTôd²
AhPj§u Ø¥®p IÚ Wu ®j§VôNj§p
ùNo©Vô®u PNu Xú_ô®fûN Årj§
ØmûT Bk§Vuv ùYt± ùTtß 4#YÕ
Nôm©Vu ThPm ùYu\ôo.
Øû\VôL úLôlûTûV ûLlTt±VÕ.
zz TgNôl Uô¨Xm í§Vô]ô®p, @û]jÕ
zz ùT-dv vPôm NoYúRN ÏjÕfNiûP
Bk§V LôpTkÕ áhPûUlTôp (AIFF)
úTôh¥«p Bk§Vô®u LÜWq úNôXe¡,
SPjRlThP NkúRôx úLôlûTdLô] LôpTkÕ
U½x Lܵd A¡úVôo ReLm ùYu\]o.
úTôh¥«p NoÅNv @¦ TgNôl @¦ûV
Årj§ úLôlûTûV ùYußs[Õ. zz Uôtßj§\]ô°LÞdLô] A£V ûNd°e
Nôm©Vuµl úTôh¥ 2019#Bp C5 ©¬®p
zz ùUôWôdúLô®u WTôj SL¬p NoYúRN UôWjRôu
Bk§V ÅWo §® `ô ùYs°Ùm, C4 ©¬®p
úTôh¥ SûPùTt\Õ. DX¡u TpúYß
ÏoXôp £e ùYiLXØm ùYu\]o.
TϧL°p BÚkÕm ÑUôo 24 A«Wm ÅWo,
ÅWôeLû]Ls TeúLt\]o. AiLÞdLô] zz @NoùTn_ôu ¡Wôih©¬dv 2019 Lôo
UôWjRôu úTôh¥«p ùLuV ÅWo Nª ¡Lu TkRV úTôh¥«p ©uXôkÕ SôhûP
TkRV çWjûR 2 U¦ 2 ¨ªPm 12 úNokR YôphúP¬ ®dPo úTôhPôv ùYt±
®]ô¥L°p LPkÕ ×§V NôRû]ÙPu, ùTt\ôo.
ØR-PjûR ©¥jRôo. zz H££ ùY°«hÓs[ ùTiLÞdLô] IÚ

 A-Series in
          ©lWY¬ 2020
Sôs ¡¬dùLh RWY¬ûN«p Bk§V ShNj§W ¡¬dùLh Nôm©Vuµl LÜWYjûR ®Wôh
ÅWôeLû] vm¬§ UkR]ô ØR-PjûR úLôy- RûXûU«Xô] Bk§V @¦
ùTtßs[ôo. TkÕ ÅfNô[oL°u RWY¬ûN«p ÁiÓm ûLlTt± NôRû] TûPjÕs[Õ.
Bk§V ÅWôeLû] _ýXu úLôvYôª ùPvh úTôh¥L°p ØR-PjûR ©¥jÕs[
ØR-PjûR ©¥jÕs[ôo. Bk§V @¦dÏ (116 ×s°Ls) LRôÙRj
ÕPu ì.7 úLô¥ T¬NôL YZeLlThPÕ.
zz Tl×Yô ¨ë ¡²Vô Sôh¥tÏ NoYúRN IÚ
Sôs ¡¬dùLh @kRvÕ Utßm NoYúRN _þu
¡¬dùLh LÜu£-u Dßl©]o @kRvÕm
YZeLlThÓs[Õ. zz ùUo£¥v @¦«u í«v aôªpPu
vùT«²u Tôo£úXô]ô®p SûPùTt\
zz I¥Nô Uô¨Xm ×Yú]xY¬p SûPùTt\
TôoØXô 1 Lôo TkRVj§u vTô²x ¡WôiP
BWiPôYÕ Bk§Vu ãlTo úLôlûT
©¬dv úTôh¥«p ùYt± ùTt\ôo.
LôpTkÕ ùRôP¬p úLôYô @¦ Nôm©Vu
ThPjûR ûLlTt±VÕ. zz NoYúRN ¡¬dùLh úTôh¥ SÓYoLs ÏÝ®p
ØRpØû\VôL Bk§VôûY úNokR G.S
zz aôeLôe¡p SûPùTt\ A£V TYo-l¥e
XzªûV H.£.£ ¨VªjÕs[Õ.
Nôm©Vu úTôh¥ UL°o ©¬®p ùNuû]ûV
úNokR Tp UÚjÕYo Aoj§ @Úi ReLl zz vùT«²u Uôh¬h SL¬p 64#YÕ HúWôl©V
TRdLm ùYu\ôo. LôpTkÕ ¡[l @¦LÞdÏ BûP«Xô]
Nôm©Vuv Äd ùRôPo SûPùTt\Õ. BRu
zz ùSRoXôk§p SûPùTt\ Pf NoYúRN
Bߧ AhPj§p -Yoép @¦, PôhPuaôm
TôhªiPu úTôh¥ 2019#Bp Bk§VôûY
@¦ûV 2#0 Fu\ úLôp LQd¡p Årj§
úNokR aoºp Pô Nôm©Vu ThPm ùYu\ôo.
Nôm©Vu ThPm ùYu\Õ.
zz TôoØXô 1 Lôo TkRVj§u 1000#YÕ úTôh¥«p
zz 2023# Am Ai¥p SPdLÜs[ A£V
Be¡XôkûR úNokR ùUo£Pv ÅWo í«p
úLôlûT LôpTkÕ úTôh¥ûV º]ô SPjR
aôªpPu Nôm©Vu ThPm ùYu\ôo.
Üs[Õ ùRu ùLô¬Vô Ru ØVt£ûV §ÚmT
zz úLôXôXmé¬p SûPùTt\ UúX£VôÜdÏ ùTt\ ©\Ï º]ô BkR YônlûT ùTtßs[Õ.
F§Wô] UL°o Ad¡ ùPvh ùRôPûW
Bk§Vô 4#0 Fu\ LQd¡p ûLlTt±VÕ. zz BjRô- JTu ùPu²v ùRôP¬u UL°o
Itû\Vo ©¬Ü Bߧ AhPj§p ùNd
zz vÏYôx ®û[VôhÓ úTôh¥«u NoYúRN Ï¥VWûN NôokR LúWô-]ô ©°vúLôYô
RWlTh¥V-p ØRp 10 BPjûR ùTtßs[ ©¬hPû] úNokR ú_ôau]ô úLôiPôûY
ØRXôYÕ Bk§Vo Fàm ùTÚûUûV NÜWq Årj§ Nôm©Vu ThPm ùYu\ôo.
úLôNp ùTtßs[ôo.
zz º]ô®p @¦LÞdÏ BûP«Xô] ѧoúUu
zz NoYúRN úP©s ùPu²v NmúU[]m úLôlûT TôhªiPu ùRôPo SûPùTt\Õ.
Nôo©p _lTô²p A£V úLôlûT úTôh¥ B§p AiLs Itû\Vo ©¬Ü, ùTiLs
SûPùTt\Õ. B§p Bk§V ÅWWô] RªrSôhûP Itû\Vo ©¬Ü, BWhûPVo ©¬Ü F]
úNokR NjVu Oô]úNLWu A\ôYÕ BPjûR @û]j§Ûm º]ô ùYt± ùTtß 11#YÕ Øû\
©¥jRôo. BRu êXm º]ô®p 2019 @dúPôTo VôL úLôlûTûV ûLlTt±VÕ.
UôRm SûPùTßm DXL úLôlûT úP©s
ùPu²v úTôh¥dÏ Rϧ ùTtßs[ôo. zz Tô o Ø X ô 1 L ô o T k R V j § u 6 # Y Õ Ñt ß
úTôh¥Vô] ùUô]ôdúLô ¡Wôih©¬ TkRVm
zz @où_u¥]ô®p SûPùTt\ DXL ¡d Tôd£e Uôuh LôoúXô®p Ds[ JÓR[j§p SPkRÕ.
Nôm©Vuµl úTôh¥«p LôxÁûW úNokR 15 B§p 5 Øû\ Nôm©V]ô] Be¡Xôk§u
YVÕ UôQYu A©j aªj ReLm ùYuß Ä®v aôªpPu (ùUo£Pv @¦) 1 U¦
NôRû] TûPjRôo. 43 ¨ªPm 28.437 ®]ô¥L°p ØRXôYRôL
zz 2019#Am AiÓdLô] H££#«u ùPvh YkÕ 25 ×s°Lû[ Rh¥f ùNu\ôo.

©lWY¬ 2020 A-Series in


  
zz Ñ®hNoXôk§u ¡¬uv Tϧ«p AiLÞd 4#Am BPjûR ùTt\Ru êXm Rϧ
Lô] NoYúRN JTu vÏYôx ùRôPo ùTtßs[ôo.
SûPùTt\Õ. BRu Bߧ AhPj§p zz 2019 Bu UEFA Nôm©VuµlûT -Yoép
Bk§Vô®u UúLx UuúLôVuLo, vùT«²u ùYußs[Õ. úTôhPuaôûU Årj§ ÄYoép
ùTo]ôhûP Årj§ Nôm©Vu ThPm @¦Vô]Õ 6#YÕ Øû\VôL HúWôlTô®u
ùYu\ôo. Nôm©V]ôL DÚùYÓjÕs[Õ. LPkR GÝ
zz ù_oU²«u زf SL¬p DXLd úLôlûT AiÓL°p -Yoép @¦dÏ ¡ûPjR
ÕlTôd¡ ÑÓm úTôh¥ SûPùTt\Õ. ØRp úLôlûT BÕYôÏm.
B§p TeúLt\ Bk§V @¦ ùUôjRm 5 zz @VoXôk§u Pl°u SL¬p SûPùTt\
ReLm, IÚ ùYs°l TRdLjÕPu, TRdL Bߧl úTôh¥«p @VoXôkûR ùYu\
Th¥V-p ØR-PjûR ©¥jÕ NôRû] Ru êXm Bk§V _þ²Vo UL°o aôd¡
TûPjRÕ. @¦ LôiPo K©hvù_Wôph U21 NoYúRN
zz ¡WôihvXôm úTôh¥L°p Iu\ô] ©ùWgf ThPjûR ùYu\Õ.
JTu ùPu²v ùRôPo TôÃv SL¬p zz HúWôl©V SôÓLÞdÏ BûP«p SûPùTt\
SûPùTt\Õ. B§p AiLs Itû\Vo ©¬Ü úS`uv Äd Bߧl úTôh¥«p ùSRoXôkûR
Bߧ AhPj§p vùT«²u WùTp SPôp, Årj§ úTôofÑdLp Nôm©Vu ThPm ùYu\Õ.
Av§¬Vô®u ùPôª²d §mûU Årj§
12#YÕ Øû\VôL ©ùWgf JTu ThPjûR zz IÚSôs úTôh¥«p @§L WuLs FÓjR
ûLlTt± NôRû] TûPjRôo. ùTiLs ÅWoLs Th¥V-p ¥Wô®h NôRû]ûV
Itû\Vo ©¬Ü Bߧ AhPj§p Av§úW- Bk§V úLlPu ®Wôh úLô- رV¥jÕs[ôo.
Vô®u Ax- Tôo¥, ùNdÏ¥VW£u Be¡Xôk§p SûPùTt\ DXLd úLôlûT
UôoùLhPô úYôuhúWôúNôYôûY Årj§ ¡¬dùLh úTôh¥«p Av§úW-Vô @¦dÏ
R]Õ ØRp ¡WôihvXôm Nôm©Vu ThPjûR F§WôL 29#YÕ Wuû] FÓjRúTôÕ @Yo
ùYu\ôo. BkR NôRû]ûV TûPjRôo.
zz ùL[aôj§«p SûPùTt\ 2019#Am AiÓ zz ùTiLÞdLô] FhPôYÕ DXL úLôlûT
NoYúRN JTu ÏjÕfNiûP úTôh¥«p ùRôPo ©Wôuv RûXSLo Tô¬£p ùRôPe¡VÕ.
Bk§Vô ùUôjRUôL 57 TRdLeLû[ ùYu\Õ. B§p ùUôjRm 24 SôÓLs TeúLt¡u\]. A£V
B§p 12 ReLm, 18 ùYs° Utßm 27 ùYiLX SôÓL°p º]ô, _lTôu, ùLô¬Vô, RônXôkÕ
TRdLeL[ôÏm. A¡V SôÓLs UhÓúU TeúLt¡u\].
zz Be¡XôkÕ, IÚSôs ¡¬dùLh YWXôt±p zz ùRu @ùU¬dLô®u LôpTkÕ SPjÕm
ùRôPof£VôL @§LØû\ 300 WuLs Ï®jR ÏÝYô] CONMEBOL, Av§úW-Vô Utßm
@¦ Fàm NôRû]ûV TûPjÕs[Õ. IÚSôs LjRôûW 2020 úLôTô @ùU¬dLô®p (COPA
¡¬dùLh YWXôt±p 7 Øû\ ùRôPof£VôL 300 America) TeúLtL @ûZl× ®ÓjÕs[Õ.
WuLÞdÏ úUp FÓjÕs[Õ. BRtÏ Øu zz úLôTô 2020#H @où_u¥]ô Utßm ùLôXm©Vô
Av§úW-Vô ùRôPof£VôL 6 Øû\dÏ úUp BûQkÕ SPjRÜs[].
300 WuLs @¥jÕ NôRû] ׬k§ÚkRÕ. zz Fl.H.Ff APYo aôd¡ º¬v BߧlúTôh¥«p
zz úR£V @[®Xô] Nl#_ý²Vo YÛçdÏm ùRu]ôl©¬dLôûY Årj§ Bk§Vô Nôm©Vu
úTôh¥«p RªrSôÓ AiLs ©¬Ü Nôm©Vu ThPm ùYu\Õ. BkR ùYt±«u êXm 2020
ThPm ùYu\Õ. I-m©d úTôh¥dÏ Bk§V @¦ Rϧ
zz Bk§VôûYf úNokR Uà TôdLo 2020 ùTtßs[Õ.
úPôd¡úVô I-m©d úTôh¥dÏ ÕlTôd¡f
ÑÓR-p Bk§Vô®u NôoTôL TeúLtÏm
_þûX
zz Av§úW-Vô®u ùUpúTôou SL¬p 2019
RϧûV ùTt\ôo. NoYúRN ÕlTôd¡f ÑÓRp
NoYúRN T£©d ©p-Vohv Nôm©Vuµl
áhPûUl©u DXL úLôlûT úTôh¥«p

 A-Series in
          ©lWY¬ 2020
ùRôPo SûPùTt\Õ. BRu Bߧ úTôh¥«p BߧlúTôh¥«p @ùU¬dLô, ùSRoXôkûR
£eLlé¬u ÀhPo ¡p¡±vh Bk§Vô®u Årj§ Nôm©Vu ThPm ùYu\Õ.
NÜWq úLôjRô¬ûV Årj§ úLôlûTûV zz XiP²p SûPùTt\ ®m©sPu JTu
ùYu\ôo. ùPu²v úTôh¥«p AiLs Itû\Vo
zz Be¡Xôk§p Ds[ Toªeaôm SL¬p ©¬®p ùNo©Vô®u úSôYd ú_ôúLô®f,
ùTiLÞdLô] Toªeaôm ¡[ô£d ùPu²v Ñ®hNoXôk§u úWô_o ùTPWûW Årj§
ùRôPo SûPùTt\Õ. Bߧ AhPj§p Nôm©Vu ThPm ùYu\ôo. ùTiLs Itû\Vo
Av§úW-Vô®u Ax- Tôo¥ úSo ©¬Ü Bߧ AhPj§p @ùU¬dLô®u
ùNhL°p ù_oU²«u _þ-Vô úLôo_ûN ùNÃ]ô ®p-Vmû^, ÚúU²Vô®u
Årj§ Nôm©Vu ThPm ùYu\ôo. £úUô]ô aôùXl Årj§ R]Õ ØRp
zz TôoØXô 1 Lôo TkRVj§u 8#YÕ Ñt\ô] ®m©sPu Nôm©Vu ThPjûR ùYu\ôo.
©ùWgf ¡Wôih©¬ TkRVm Ä LôvhùXh zz RônXôk§p SûPùTt\ NoYúRN ùPu²v
JÓR[j§p SûPùTt\Õ. B§p 309.60 ¡.Á. áhPûUl©u AiLs ©ëfNov ùPu²v
ùLôiP TkRV çWjûR 1 U¦ 24 ¨ªPm úTôh¥«p Bk§Vô®u £jRôoj WôYj,
31.198 ®]ô¥L°p LPkÕ Be¡Xôk§u Ä®v _lTô²u ¬úVô úSôÏf£ûV Årj§ Nôm©Vu
aôªpPu ØR-Pm ùTt\ôo. ThPm ùYu\ôo.
zz LjRôo RûXSLo úRôaô®p A£V vòdLo zz NoYúRN ¡¬dùLh úTôh¥L°p 20,000 WuLû[
Nôm©Vuµl ùRôPo SûPùTt\Õ. Bߧ @§úYLUôL LPkR ÅWo Fu\ DXL NôRû]
AhPj§p Bk§Vô®u TeLw @jYô², ûV Bk§V ¡¬dùLh ÅWo ®Wôh úLô-
6#3 Fu\ LQd¡p RônXôk§u R]ôYôj TûPjÕs[ôo. ®Wôh úLô- 417 úTôh¥L°p
§WúTôeûTúTôu Årj§ Nôm©Vu ThPm 20,000 WuLû[ LPkÕ ShNj§W ÅWo Nf£u
ùYu\ôo. ùPiÓpL¬u NôRû]ûV رV¥jÕs[ôo.
zz © - l û T u v R û X S L o U ¦ X ô ® p zz RônXôk§u ThPôVô SL¬p SûPùTt\
A£V UL°o Wd© Nôm©Vuµl ùRôPo A£Vô £pYo ùYpPoùY«h ÏjÕfNiûP
SûPùTt\Õ. BRu Bߧ AhPj§p º] úTôh¥«p Bk§V ÅWo ûYTq VôRq
@¦, ©-lûTuû^ Årj§ Nôm©Vuµl Nôm©Vu ThPm ùYu\ôo.
ThPjûR ùYu\Õ. zz NoYúRN I-m©d Lªh¥«u קV RûXûU
zz @ùU¬dLô®u L-úTôo²Vô®p Ds[ VLm I-m©d LÜv Fàm ùTV¬p
ùTl©s Àf úLôpl -ed³p 2019#Am Ñ®hNoXôkÕ Sôh¥u XôNôuú] SL¬p
AiÓdLô] ÙFv JTu úLôpl ùRôPo §\dLlThÓs[Õ.
SûPùTt\Õ. B§p @ùU¬dLô®u úL¬ zz UeúLô-Vô®u DXuTôRo SL¬p SûPùTt\
ÜhúXih Nôm©Vuµl ThPm ùYu\ôo. A£V ´m]ôv¥d Nôm©Vuµl úTôh¥«p
zz ©úW£p Sôh¥p 46#YÕ úLôTô @ùU¬dLô Bk§Vô®u ©W]ô§ SôVd ùYiLX TRdLm
LôpTkÕ úTôh¥ 2019, SûPùTt\Õ. ùYu\ôo.
BߧúTôh¥«p ©úW£p @¦ 3#1 Fu\ zz A£V Lôi¥]uPp ùNv Nôm©Vuµl
úLôp LQd¡p ùTÚûY Årj§ 9#YÕ Øû\ 2019 úTôh¥«p Bk§VôûY úNokR 14 YVÕ
VôL Nôm©Vu ThPm ùYu\Õ. ¡WôihUôvPo ¨ap `¬u Nôm©Vu
zz º]ô®u UdLôq SL¬p SûPùTt\ A£V ThPm ùYu\ôo.
_þ²Vo vÏYôx Nôm©Vu úTôh¥«p zz ÏúWôµVô®u DUôd¡p SûPùTt\ DXL
Bk§V ÅWo úNôh Wô², NL ÅWWô] Vôx Bû[Oo úLôlûT LWôjúR 2019 úTôh¥«p
TúRûV Årj§ Nôm©Vu ThPm ùYu\ôo. 12 YVÕdÏhThP ©¬®p 10 YVÕ £ßª
zz FhPôYÕ ùTiLs DXLd úLôlûT LôpTkÕ @¬g£Rô úP ùYs°l TRdLm ùYußs[ôo.
úTôh¥ 2019 ©Wôuv Sôh¥p SûPùTt\Õ. zz BjRô-«u SôúTô-«p SûPùTt\ 100 Á

©lWY¬ 2020 A-Series in


  
JhPlTkRVj§p Bk§V ÅWôeLû] åh¥ ©W£ùPuh úLôlûTdLô] ÏjÕfNiûP
Nkj ReLlTRdLm ùYu\ôo. DXL @[®Xô] úTôh¥ SûPùTt\Õ. B§p 51 ¡úXô
100 Á JhPj§p ReLm ùYu\ ØRp Bk§V FûPl©¬Ü Bߧl úTôh¥«p Aß Øû\
ÅWôeLû] åh¥ Nkj AYôo. DXL Nôm©Vu ThPm ùYu\ Bk§Vô®u
zz A£V úLôlûT LôpTkÕ úTôh¥Ls 2023#Am ú U ¬ ú L ô m , Av§úW-Vô®u Gl¬p
AiÓ º]ô®p SûPùT\Üs[Õ. ©Wôueû^ F°RôL Årj§ ReLl TRdLm
ùYu\ôo.
zz Be¡Xôk§p SûPùTt\ DXL úLôlûT
zz TôoØXô 1 LôoTkRVj§u 11#YÕ Ñt\ô]
¡¬dùLh úTôh¥«u Bߧ AhPj§p
ù_oUu ¡Wôih©¬ TkRVj§p, ùWh×p
Be¡XôkÕ @¦ ¨ë£XôkûR Årj§
@¦«u ùTp´Vm ÅWWô] Uôdv
ØRpØû\VôL DXL úLôlûTûV ûLlTt±VÕ.
ùYovPôlTu 1 U¦ 44 ¨ªPm 31 ®]ô¥
BkR ùRôP¬u AhP SôVL]ôL ¨ë£XôkÕ
L°p BXdûL @ûPkÕ Nôm©Vu ThPjûRd
@¦ úLlPu úLu ®p-VmNu úRoÜ
ûLlTt±]ôo.
ùNnVlThPôo.
zz úTôXkÕ RûXSLo YôoNô®p úTôXkÕ
ALvh JTu UpÙjRm úTôh¥ SûPùTt\Õ. B§p
zz I¥Nô®p 21#YÕ LôUuùYpj úP©s 53 ¡úXô FûPl©¬®p Bk§V ÅWôeLû]
ùPu²v Nôm©Vuµl©p AiLs Utßm ®ú]x úTôLj Bߧl úTôh¥«p úTôXkÕ
ùTiLs ©¬®p ReLm ùYu\Ru êXm ÅWôeLû] úWôdN]ôûY Årj§ ReLlTRdLm
Bk§Vô NôRû] TûPjÕs[Õ. LôUuùYpj ùYu\ôo.
®û[VôhÓ YWXôt±p ØRp Øû\VôL zz ÏúWôµVô®u I£_d¡p TôWô DXLdúLôlûT
ReLjûR ùYu\ Bk§V UL°o @¦ Bߧl ÕlTôd¡ÑÓRp úTôh¥ SûPùTt\Õ. B§p
úTôh¥«p Be¡XôkûR úRôtL¥jRÕ. AiLÞdLô] 10 ÁhPo Go ©vPp úTô¥Vm
zz ùRu ùLô¬Vô®u ÏYôe_þ®p DXL ©¬®p Bk§Vô®u WôÏp _dLôo ReLl
¿fNp Nôm©Vuµl úTôh¥Ls SûPùTt\Õ. TRdLm ùYu\ôo.
B§p APYo 200 ÁhPo ThPo©û[ ¿fNp zz @ùU¬dLô®u YôµePu SL¬p YôµePu
úTôh¥«p aeúL¬ B[m ÅWo ¡±vúPôl JTu ùPu²v ùRôPo SûPùTt\Õ. B§p
ªpd 1 ¨ªPm 50.73 ®]ô¥L°p BXdûL AiLs Itû\Vo ©¬Ü Bߧ AhPj§p
LPkÕ ×§V DXL NôRû] TûPjRôo. Av§úW-V ÅWo ¨d ¡o´úVôv, úP²p
zz N o Y ú R N ¡ ¬ d ù L h L Ü u £ p (ICC) ùUjYúRûY Årj§ Nôm©Vu ThPm
ùY°«hÓs[ ùPvh úThvúU²u ùYu\ôo.
RWY¬ûN«p, Bk§V úLlPu ®Wôh úLô- zz 2020 Utßm 2024 I-m©d úTôh¥LÞdLô]
922 ×s°LÞPu R]Õ ØR-PjûR RdL RVôo ¨ûXûV IÚe¡ûQjÕ Utßm
ûYjÕd ùLôiPôo. ¨ë£Xôk§u úLu Dj§Lû[ YÏjÕf ùNVpTÓjÕYRtLôL 10
®p-VmNu (913) BWiPôYÕ BPj§Ûm, Dßl©]oLû[d ùLôiP DVo UhPd ÏÝ
Bk§Vô®u úNúPxYo ×_ôWô (881) êu\ôYÕ Iuß @ûUdLlThÓs[Õ. BÕ ®û[VôhÓj
BPj§Ûm Ds[]o. Õû\ @ûUfNWô] ¡Wi ¬w_ý®u
zz _lTôu RûXSLo úPôd¡úVô®p _lTôu RûXûU«p ùNVpTÓm.
JTu TôhªiPu ùRôPo SûPùTt\Õ. BRu zz ûRYô²u ûRúT«p SûPùTt\ 2#YÕ DXL
AiLs Itû\Vo ©¬Ü Bߧ AhPj§p LôÕ úL[ôR Bû[Oo élTkÕ Nôm©Vuµl
_lTô²u ùLuúPô ùUôúUôhPô, BkúRôú] 2019 úTôh¥«p RªZL ÅWôeLû]Vô]
µVô®u ¡±v¥ûV Årj§ Nôm©Vu ThPm ú_ ù_o-u @²Lô 4, TRdLeLû[ (1 ReLm,
ùYu\ôo. 2 ùYs°, 1 ùYiLXm) ùTtßs[ôo.
zz BkúRôú]£Vô®u X×Vôu Tú_ô®p 23#YÕ (ùRôPof£ @ÓjR BR¯p...)

 A-Series in
          ©lWY¬ 2020
TNPSC £û\jÕû\ T¦LÞdLô] úRoÜ # 2019
@Np ®]ôjRôs ùTôÕjRªÝm ùTôÕ@±Üm úRoÜ Sôs : 22#12#2019

1. @¥úLô¥hP ùNôtLÞdÏ BXdLQd TÓjÕ


ϱl× RÚL. Ñt\m ºoçdÏ £kRû] NôhûP NeÏ
@kR GûZd ÏZkûR«u LiLs A) ºoçdÏ, NeÏ, £kRû], NôhûP,
ϯkRôrkÕ LôQlTÓ¡\Õ. Ñt\m
A) ùTVùWfNm B) £kRû] ºoçdÏ NeÏ Ñt\m NôhûP
B) ®û]ùVfNm C) NôhûP Ñt\m ºo çdÏ £kRû]
C) Øtù\fNm NeÏ
D) IÚ ùTôÚh TuùUô¯ D) NeÏ NôhûP £kRû] ºo çdÏ
2. ûRj§eLs FuTÕ ............................... ùTVûWd Ñt\m
ϱdÏm 6. I - ú Y ß T ô Ó @ ± k Õ N ¬ V ô ]
A) BPlùTVo ùTôÚû[ @±L.
B) UôRlùTVo SôhÓdÏj úRûY#
C) LôXlùTVo A) @Wu B) @\u
D) ùTôÚhùTVo C) @\i D) @Wi
3. ùNôtLû[ IÝeÏTÓjÕL. 7. Y ô û X , Y ô û Z , Y ô û [ , I -
""At\p ¨Wm©V ùNôtLs L®ûR úYßTôP±kÕ ùTôÚs RÚL.
RôUôLl ùTôe¡ Y¯YÕRôu'' A) ÁuYûL, UWYûL, B[mùTi
A) RôUôLl ùTôe¡ Y¯YÕRôu L®ûR B) B[mùTi, UWYûL, ÁuYûL
At\p ¨Wm©V ùNôtLs C) B[mùTi, ÁuYûL, UWYûL
B) At\p ¨Wm©V L®ûR ùNôtLs D) UWYûL, ÁuYûL, B[mùTi
RôUôLl ùTôe¡ Y¯YÕ Rôu 8. Ae¡X ùNôpÛdÏ BûQVô] Rªr
C) At\p L®ûR ¨Wm©V ùNôtLs ùNôpûX úRoÜ ùNnL?
RôUôLl ùTôe¡ Y¯YÕ Rôu ""BuLm úPdv AKÀv''
D) At\p ¨Wm©V ùNôtLs RôUôLl ""Incometax office''
ùTôe¡ Y¯YÕ Rôu L®ûR
A) YÚUô]j Õû\ @ÛYXLm
4. ùNôtLû[ IÝeÏTÓjÕL. B) YÚUô] Y¬ @ÛYXLm
®V]Lo Ïû\TPôd ùLô[d áÝûP C) YÚUô] @ÛYXLm
ùLô[d D) YÚYôn @ÛYXLm
A) ùLô[d ùLô[d áÝûP ®V]Lo
9. YïDf ùNôpXt\ ùRôPo FÕ?
Ïû\TPôd
I. YXÕ TdLm ÑY±p FÝRôúR
B) á Ý û P ® V ] L o Ï û \ T P ô d
II. YXl TdLf ÑY¬p FÝRôúR
ùLô[dùLô[d
III. YXl TdLf ÑYt±p FÝRôúR
C) ùLô[d áÝûPV ®V]Lo ùLô[d
Ïû\TPd IV. YXl TdLf ÑY±p FÝRôúR
D) ùLô[d ùLô[d Ïû\TPôd áÝûP A) IV B) I
®V]Lo C) II D) III
5. ¸rdLôÔm ùNôtLû[ @LW Y¬ûNl 10. @¦kÕ # F§of ùNôpûXd LiP±L.

©lWY¬ 2020 A-Series in


  
A) ùRôûX®p B) DnjÕ D) DXLj Rªr UôSôÓ Ï¥VWÑj
C) ùLôÓjÕ D) @[jRp RûXYWôp ùRôPe¡ ûYdLl
11. úNojÕ FÝÕL. ùTt\Õ
Tû] + JûX = 16. SôX¥Vôo # Fàm èûXl Tô¥V
YoLs Vôo?
A) Tû]úVôûX
A) NUQ زYo
B) Tû]JûX
B) ûYQYoLs
C) Tû]úYôûX
C) SôVuUôoLs
D) Tû]qúYôûX
D) ùT[jRj Õ\®Ls
12. @uTLjÕ BpXô D«oYôrdûL
YuTôtLi 17. U¦úULûX«u úRô¯
Yt\p UWmR°oj Rtß A) ¾Y§XûL
B§p @uTLjÕ BpXô FuTRû] B) ÑRU§
FqYôß ©¬dLXôm C) UôR¬
A) @u× + TLjÕ + BpXô D) ÅQôT§
B) @u× + @LjÕ + BXô 18. " T h ¥ U i P T m ' F u T Õ N U V d
C) @u× + TLm + BpXô LÚjÕLs ®Yô§dÏm BPm Fuß
D) @u× + @LjÕ + BpXô áßm èp
13. B û V × j ù R ô û P « p @ û U k R A) £XlT§LôWm
ùNôtLû[ @±L. B) U¦úULûX
""¿X Ø¥ R¬jR TX UûXúNo SôÓ'' C) ¿XúL£
D) ÏiPXúL£
¿WØR ùU]lTônkÕ ¨Wm× SôÓ''
A) ¿XØ¥#¿WØR 19. ×\Sôòt±p Ds[ ×\j§û]L°u
Fi¦dûL FjRû]?
B) ¿XØ¥#UûXúNo
A) HkÕ
C) ¨Wm×#SôÓ
B) T§ù]ôuß
D) SôÓ# SôÓ
C) TjÕ
14. RôùVlTl úTÑm ULs Fu\ DXûUj
D) GÝ
ùRôPÚdÏl ùTôÚs RÚL?
A) RkûRûVl úTôuß úTÑRp 20. TôPp Y¬Ls L-j ùRôûL«p
Fl©¬®p BPm ùTtßs[]?
B) ReûLûVl úTôuß úTÑRp
C) RôûVl úTôuß úTÑRp FÝkRÕ ÕLs
D) Rm©ûVl úTôuß úTÑRp Gt\]o Uôo×
15. Ï ¥ V W Ñ j R û X Y o D X L j R ª r L®rkR] UÚh×
UôSôhûPj ùRôPe¡ ûYjRôo. LXe¡]o TXo
ùNn®û]ûV ùNVlTôhÓ ®û]VôL A) ϱg£d L-
UôtßL. B) ØpûXd L-
A) Ï¥VWÑj RûXYo DXLj Rªr C) UÚRd L-
UôSôhûPj ùRôPe¡]ôo D) ùSnRp L-
B) DXLj Rªr UôSôÓ Ï¥VWÑj 21. ""CùV] BWjRp B¯kRuß;
RûXYo ùRôPe¡]ôo @Ru F§o''
C) Ï¥VWÑj RûXYo ùRôPe¡]ôo CúVu Fu\p @R²àm B¯kRuß;''
DXLjRªr UôSôhûP átß 1 : BlTôPX¥Lû[l Tô¥VYo

 A-Series in
          ©lWY¬ 2020
LûZ§u Vôû]Vôo B) Yô¦RôNu
átß 2: BlTôP-p ×LZlTÓm C) LiQRôNu
Ys[p Tô¬ D) YiQRôNu
A) átß BWiÓm N¬ 27. AodLôÓ FuTÕ FkR UWeLs ãrkR
B) átß 2 UhÓm N¬ Tϧ?
C) átß 1 UhÓm N¬ A) AX UWeLs
D) átß BWiÓm RYß B) UôUWeLs
22. "RªûZ Y[odL úYiÓm Fu\ôp ©\ C) @j§ UWeLs
ùUô¯ûV ùYßdL úYiÓm FuTÕ D) Aod UWeLs
ùTôÚ[uß' Fuß á±VYo? 28. @iQp @múTjLûW ""TÏjR±Ü
A) Ø.Y. ùNmUp, UdL°u UôùTÚm
B) §Ú.®.L Y¯Lôh¥'' F] ×LZôWm ãh¥VYo?
C) TôW§RôNu A) _YLoXôp úSÚ
D) L®Oo ÑWRô B) RkûR ùT¬Vôo
23. ""NUWN NuUôodL NeLjûR'' # C) BWôú_k§W ©WNôj
úRôtß®jRYo D) êR±Oo BWô_ô´
A) ÑjRô]kR TôW§Vôo 29. LôUWôNo NhPUu\ Dßl©]WôLj
B) WôU¡ÚxQ TWUamNo úRokùRÓdLlThP AiÓ FÕ?
C) Uû\UûXV¥L[ôo A) 1934 B) 1939
D) BWôU-eL @¥L[ôo C) 1937 D) 1942
24. ÑûY TVu 30. ""@¬Lôp Uô±V @eLi @LpYVp''
a) B²l× 1. B²ûU #BqY¥«p @ûUkÕs[ úUôû]
b) ÕYol× 2. Y[m YûL VôÕ?
c) ×°l× 3. DQoÜ A) úUtLÕYôn úUôû]
d) Lôol× 4. At\p B) ¸rdLÕYôn úUôû]
(a) (b) (c) (d) C) áûZ úUôû]
A) 1 4 3 2 D) ùTô¯l× úUôû]
B) 2 4 1 3 31. AßØL SôYXûW ""YN] SûP ûLYkR
C) 3 2 1 4 YpXô[o'' F]l TôWôh¥VYo Vôo?
D) 4 3 2 1 A) Uû\UûX @¥Ls
B) §Ú.®.L
25. Rª¯]jûRf úNokúRôo Ï¥VWÑj
RûXYoL[ôLj úRokùRÓdLl C) T¬§Uôt LûXOo
ThÓs[ SôÓLs? D) ÏQeÏ¥ UvRôu Nô¡×
A) L]Pô, UúX£Vô 32. ""Rªr ùUô¯ûV YPùUô¯ YpXôi
B) ©¬hPu, WxVô ûU«²ußm ÁhTRtLôLúY Bû\Yu
C) ùRu Al©¬dLô, £eLléo Fuû]l TûPjRôu'' Fuß á±VYo
Vôo?
D) £eLléo, ùUô¬£VÑ
A) úRYúSVlTôYôQo
26. ""Rªrl TpLûXdLZLj§u Rªr
B) T¬§UôtLûXOo
@uû] ®ÚÕ'' ùTt\ ùTÚûUd
ϬVYo Vôo? C) ÏQeÏ¥ UvRôu Nô¡×
A) L®dúLô @lÕp WÏUôu D) ULôL® TôW§Vôo
33. SÓYi @WÑ, D.úY.Nô @YoL°u

©lWY¬ 2020 A-Series in


  
RªrjùRôi¥û]l TôWôh¥ B) @. UÚRLô£
@gNp RûX ùY°«hP AiÓ FÕ? C) DÓUûX SôWôVQ L®
A) 2006 B) 2007 D) ThÓdúLôhûPd LpVôQ ÑkRWm
C) 2008 D) 2010 40. ""BWhûPd ¡[® úTôp BûQkúR YôÝeLs
34. £ t T d L û X Ï ± j Õ R ª r S ô Ó ©¬kRôp ùTôÚ°pûX........''
ùRô¯pÖhT Lp® BVdLLm Fuß DYûU UW©p ×ÕûULû[f
ùY°«hP èûXd LiP±L. úNojRYo Vôo?
A) £tT Uû\ A) Ñl×Wj§] RôNu
B) £tTf £ûX B) TôW§RôNu
C) ùNkèp £tTm C) Yô¦RôNu
D) £tTf ùNkèp D) LiQRôNu
35. SôVdLo LôXj§p £\kÕ ®[e¡V 41. © ù W g Ñ d Ï ¥ V W Ñ j R û X Y W ô p
£tTd LûX FÕ? "ùNYô-Vo' ®Ú§û]l ùTt\Yo?
A) Lt£tTm A) TôW§RôNu
B) ùNl×j §ÚúU² B) Yô¦RôNu
C) UWf £tTm C) Ø¥VWNu
D) RkRf £tTd LûX D) ÑWRô
36. ÑÅPu Sôh¥u úTo XôLod®vh 42. ÑkRWo FqYWNWôp ULuûU ùLôiÓ
Fuàm £ßLûRLû[ Rª¯p Y[odLlThPôo?
ùUô¯ ùTVojRYo? A) SW£eLØû]VûWVo
A) ´.Ù. úTôl B) SW£mUYoUu
B) ´. Ïl×Nôª C) Sk§YoUu
C) ùXhѪ D) SW£eLSôRo
D) T. ù_Vl©WLôÑ 43. Vôo Tô¥V £jRo TôPpLs ""Oô]l
37. BWôU SôPLjûR BVt±VYo Vôo? TôUôûX'' Fuß YZeLlTÓ¡\Õ?
A) ®ÑYSôR Nôv§¬Vôo A) TômTôh¥f £jRo
B) @ÚQôfNXdL®WôVo B) @Lj§Vo
C) BWôUfNk§W L®WôVo C) £YYôd¡Vo
D) Uô¬ØjÕl ©sû[ D) LÓùY°f £jRo
38. "UW×d L®ûR«p úYo TôojRYo: 44. SkR Y]j§úXôo Ai¥ # @Yu
×Õd L®ûR«p UXo TôojRYo' SôXôß UôRUônd ÏVYû] úYi¥
Fuß TôWôhPlTÓTYo Vôo? ùLôiÓ YkRôù]ôÚ úRôi¥
A) @lÕp WÏUôu Fu\ Y¬Lû[l Tô¥VYo#Vôo?
B) S. ©fNêoj§ A) TjW¡¬Vôo
C) Rªr I° B) @Lj§V Oô]m
D) ÑjRô]kR TôW§Vôo C) LÓùY°f £jRo
39. ""Df£ UûX«úX Eßm @Ú®Ls D) £YYôd¡Vo
IúW Y¯«p LXdÏÕ 45. IÚ ùNôpúXô ùRôPúWô BÚ ùTôÚs
ItßûU «pXô U²RÏXm RÚUôß TôÓYÕ?
DVoÜ RôrÜ Y[odÏÕ'' # Fuß A) RtϱlúTt\¦
Tô¥VYo
B) DYûUV¦
A) LiQRôNu

 A-Series in
          ©lWY¬ 2020
C) DÚYL @¦ 52. Suòp FqYû] èp?
D) BWhÓ\ùUô¯Rp @¦ A) ùNn§ Yôd¡Vm
46. F]dÏ ªL ®ÚlTUô] BXd¡Vm B) ®ûZÜ Yôd¡Vm
Iuß DiùPu\ôp @Õ .................................. C) LhPû[ Yôd¡Vm
Fuß úTW±Oo @iQô á±]ôo. D) ®]ô Yôd¡Vm
A) L-eLjÕlTW¦ 53. ®ûPdúLt\ ®]ôûYj úRokùRÓdL
B) §ÚdÏ\s TôW§RôNu ×Wh£dL®Oo F]
C) LmTWôUôVQm @ûZdLlTÓ¡\ôo
D) ÏßkùRôûL A) TôW§RôNu ×Wh£dL®Oo F]
47. "@¬Y¥Ü Uônl©u ]WuY¥Ü Uô¡' @ûZdLlTÓ¡\ôWô?
.......... Fu\ ùRôP¬p @¬ FuTRu B) ×Wh£dL®Oo F] @ûZdLlTÓTYo
ùTôÚs VôÕ? Vôo?
A) U²R Y¥Ym C) TôW§RôNu ×Wh£dL® F] @ûZdLl
B) £eLm TÓYúRu?
C) SW£eLm D) TôW§RôNu FqYôß @ûZdLl
D) úRYoLs TÓ¡\ôo?
48. ""Suß Suù\]l úTôt±úV SPkRÕ 54. TôP-p YÚm §û]ûVd LiÓ©¥
úYeûL'' BqY¥Ls BPm ùTtßs[ ""Yôu DhÏm Y¥¿i U§p,
èp UpXp êço YV úYkúR!''
A) úRYôWm A) TôPôi§ûQ
B) LmT BWôUôVQm B) ùTôÕ®Vp
C) ùT¬V ×WôQm C) YôûLj§ûQ
D) º\ôl ×WôQm D) ÕmûTj §ûQ
49. ùT¬V×WôQjûR @Ú°V úNd¡Zôo 55. BXdLQd ϱl× @±L.
©\kR RtúTôûRV UôYhPm FÕ? A) DYûU
A) ùNuû] B) @uùUô¯jùRôûL
B) LPío C) DÚYLm
C) ®Ýl×Wm D) DmûUjùRôûL
D) Lôg£×Wm 56. @LW Y¬ûN«p FÝÕL.
50. IußùLôXôm Fuàm §ÚlT§Lm ùUô¯ùTVol×, êk¿o, úUÓTs[m,
Tô¥VYo Vôo? U]jÕVo, ÁªûN
A) ÑkRWo A) Øk¿o, U]jÕVo, ÁªûN,
B) §ÚOô] NmTkRo ùUô¯ùTVol×, úUÓTs[m
C) §ÚSôÜdLWNo B) úUÓTs[m, Øk¿o, ÁªûN,
D) Uô¦dLYôNLo ùUô¯ùTVol×, U]jÕVo
51. úStß UûZ ùTnRÕ; @R]ôp, C) ÁªûN, Øk¿o, ùUô¯ùTVol×,
G¬Ï[eLs ¨Wm©]#FqYûL U]jÕVo, úUÓTs[m
Yôd¡Vm F]d LiP±L. D) U]jÕVo, ÁªûN, Øk¿o, úUÓTs[m,
A) ùRôPoYôd¡Vm ùUô¯ùTVol×
B) LXûY Yôd¡Vm 57. "T Ó ® P m ' # B § p T « u ß s [
C) LhPû[ Yôd¡Vm ùRôûL¨ûXj ùRôPo
D) R²¨ûX Yôd¡Vm A) ®û]jùRôûL

©lWY¬ 2020 A-Series in


  
B) Ti×jùRôûL èpLs VôûY?
C) DYûUjùRôûL A) £XlT§LôWm, U¦úULûX
D) DmûUjùRôûL B) ºYL£kRôU¦, U¦úULûX
58. FfùNôp úYof ùNôp @pX? C) ÏiPXúL£, ¿XúL£
A) @¥ B) BÚ D) ÏiPXúL£, ã[ôU¦
C) @± D) B²Õ 66. ùTôÚjRUô] BûQûVj úRoÜ
59. YkRôu # úYofùNôpûXj RÚL. ùNnL.
A) Yôo B) Yô §ûQ £ßùTôÝÕ
C) YÚ D) YkÕ A) ϱg£ # FtTôÓ
60. "" U ô ' ' J ù W Ý j Õ I Ú ù U ô ¯ « u B) ØpûX # SiTLp
ùTôÚû[ @±L. C) UÚRm # ûYLû\
A) Vôû] D) ùSnRp # UôûX
B) ®XeÏ 67. LôXm ©\dÏm Øu ©\kRÕ RªúZ!
C) UôÓ #FkRd
D) LôÓ LôXØm ¨ûXVôn BÚlTÕm RªúZ!
61. YïDf ùNôp BpXôR ùRôPûW # BqY¥L°p T«uß YÚm SVeLs
FÝÕL. A) úUôû], FÕûL, ØWi
A) ùNuû]dÏ @ÚLôûU«p BÚlTÕ B) úUôû], ØWi, @kRô§
UÕûW @pX C) úUôû], FÕûL, BûV×
B) ùNuû]dÏ TdLj§p BÚlTÕ D) BûV×, @[ùTûP, úUôû]
UÕûW BpûX
68. SôX¥VôûWj ùRôÏjRYo
C) ùNuû]dÏ @ÚLôûU«p BÚlTÕ
A) L©Xo
UÕûW @pûX
B) @¦XôÓ Øu±Xôo
D) ùNuû]dÏ @Ú¡p BÚlTÕ UÕûW
C) TÕU]ôo
@uß
D) ØußûWVûWV]ôo
62. ùTôÚkRô BûQûVj úRoÜ ùNnL.
69. §ÚdÏ\°u @¯Vôj RuûUûVl
A) §ÚjRdLjúRYo # Yû[VôT§
Tû\ Nôtßm ùNnÙs èp FÕ?
B) B[eúLôY¥Ls # £XlT§LôWm
A) SôX¥Vôo
C) SôRÏjR]ôo # ÏiPXúL£
D) ºjRûXfNôjR]ôo # U¦úULûX B) TZùUô¯
63. ùTôÚkRôf ùNôpûXj úRoÜ ùNnL. C) §ÚYsÞYUôûX
A) Ds[m D) §¬LÓLm
B) @ZÏ 70. @pXÛtß At\ôÕ @ÝYôû[d
C) ÅÓ LiÓ Ge¡
D) BPm UpXp UÕûWVôo FpXôÚk Rôm
UVe¡
64. @Úm×Rp # F§ofùNôp RÚL.
# BlTôPpLs BPm ùTt\ èp FÕ?
A) UVeÏRp
A) U¦úULûX
B) JÓRp
B) ÏiPXúL£
C) ùU-Rp
C) £XlT§LôWm
D) ®¬Rp
D) Yû[VôT§
65. BWhûPdLôl©Vm # F] YZeÏm
71. átß 1 : ¿XúL£ IÚ NUQNUVd

 A-Series in
          ©lWY¬ 2020
Lôl©Vm D) DúWôUô׬, F¡lÕ, @úW©Vô
átß 2 : ÏiPXúL£ Fuàm 77. Vôû]«u DÚYjûRf ùNÕdÏY§p
èÛdÏ UßlTôL FÝRlThPÕ ûLúRokR £t©Ls Vôo?
¿XúL£ A) úNWo LôXjÕf £t©Ls
A) átß 1 UhÓm N¬ B) TpXYo LôXjÕf £t©Ls
B) átß BWiÓm N¬ C) úNôZo LôXjÕf £t©Ls
C) átß 2 UhÓm N¬ D) SôVdLo LôXjÕf £t©Ls
D) átß BWiÓm RYß 78. §pûXVô¥ Ys°VmûU # B§p
72. átß 1 : "LPÛs UônkR B[mùTÚ §pûXVô¥ FuTÕ
Yݧ' Fu\ ×XYo LPtùNXÜ Iu±p A) Ys°VmûU ©\kR Eo
B\kÕ úTô]Yo B) Ys°VmûU«u RôVôo ©\kR Eo
átß 2: BYo ×\Sôòt±p IÚ C) Ys°VmûU«u RkûR ©\kR Eo
TôPûXÙm, T¬TôP-p IÚ D) Ys°VmûU YôrkR Eo
TôPûXÙm BVt±Ùs[ôo.
79. Lôk§V¥L[ôp ""RjùRÓdLlThP
A) átß 2 UhÓm N¬ ULs'' Fuß @u×Pu @ûZdLl
B) átß 1 UhÓm N¬ ThPYo?
C) átß BWiÓm N¬ A) §pûXVô¥ Ys°VmûU
D) átß BWiÓm RYß B) @m×_jRmUôs
73. RªZ¬u Yôr®Vp LÚîXm FÕ? C) úYÛ Sôf£Vôo
A) ×\Sôòß D) _ôu£Wô¦
B) §ÚdÏ\s 80. U²R êû[«p Ri½¬u @[Ü
C) SôX¥Vôo FjRû] NRÅRm ùLôiPÕ?
D) B²VûY SôtTÕ A) @ßTÕ NRÅRm
74. RªÝdÏd "L§' F]l úTôt\lTÓm B) FÝTÕ NRÅRm
BÚ èpLs C) FiTÕ NRÅRm
A) §ÚdÏ\Þm, SôX¥VôÚm D) ùRôiæß NRÅRm
B) §ÚdÏ\Þm, §ÚYôNLØm 81. ù L ô u v P ô u F u à m B j R ô -
C) §ÚdÏ\Þm, LmTWôUôVQØm ùUô¯f ùNôp-u ùTôÚs VôÕ?
D) £XlT§LôWØm, U¦úULûXÙm A) @gÑTYu
75. ""UûX DÚ® UWm DÚ® B) @PdLØûPVYu
Ui DÚ®tß IÚYô°'' C) @gNôRYu
BqY¬ BPmùTt\ èp D) @±VôRYu
A) ULôTôWRm 82. ´.Ù. úTôl TônUWdLlT-p ùNuû]
B) L-eLjÕlTW¦ YkR FhÓj §eLÞm ..........................., ..........................
C) ùT¬V×WôQm A¡V ùUô¯ èpLû[l T¥jRôo.
D) BWôUôVQm A) Rªr, Ae¡Xm
76. @¬£Ùm, U«púRôûLÙm, NkR] B) YPùUô¯, Lu]Pm
Øm RªZLj§-ÚkÕ @àlTlThP
C) ùRÛeÏ, Rªr
SôÓLs?
A) ¡úWdLm, DúWôUô׬, º]ô D) YPùUô¯, Rªr
B) DúWôUô׬, F¡lÕ, º]ô 83. Il×WÜ FuTRu ùTôÚs
C) ¡úWdLm, DúWôUô׬, F¡lÕ A) @PdLØûPVÕ

©lWY¬ 2020 A-Series in


  
B) Ti×ûPVÕ D) L®U¦ úR£L ®SôVL]ôo
C) EÚdÏ DRÜYÕ 90. A]kRWeLo SôhϱlûTj RWYôLd
D) ùNpYØûPVÕ ùLôiÓ FÝRlThP SôYp FÕ?
84. Ysû[ # Fàm Bu²ûNl TôPp A) NgNôWm
VôWôp TôPlTÓm? B) @gOô¥
A) TôQoLs B) ùTiLs C) Sôû[ IÚ é UXÚm
C) APYoLs D) ÏZkûRLs D) Yô]m YNlTÓm
85. T¬§UôtLûXO¬u R²lTôÑWj 91. A X ô T û ] F u à m L ® û R j
ùRôûL Fuàm è-û] Ae¡Xj§p ùRôÏl©tÏ Nô¡jV @LôùRª ®ÚÕ
ùUô¯ ùTVojRYo Vôo? ùTt\Yo Vôo?
A) ®p-Vm ú_ôuv A) úUjRô
B) úTWô£¬Vo Wôvd B) TôW§RôNu
C) LôpÓùYp C) £t©
D) ´.Ù. úTôl D) @lÕp WÏUôu
86. ""ÅWUôزYo Rªr زYoLÞs 92. úLô. úUôL]WeLu ©\kR Eo FÕ?
IÚYWôL ®[eÏ¡u\ôo'' F]l A) AXl×ûZ
×LrkRYo Vôo?
B) AXkço
A) T¬§UôtLûXOo
C) Aêo
B) Wô.©. úNÕl©sû[
D) A]eáo
C) S. Ø. úYeLPNôª
93. "L p û X l © û N k Õ L ² V ô d Ï m
D) ûYVô׬
ùNkRª¯u ùNôpûX U¦VôLj
87. Rªr YZe¡V FpûX«û] ùRôÓjÕ' Fu\ RôXôhÓl TôPûXl
""úYeLPm ÏU¬ ¾m×]t ùT[YùUu Tô¥VYo Vôo?
\kRôu ùLpûX.....'' A) BWôU-eLl ©sû[
Fuß YûWVßjRYo B) L®U¦ úR£L ®SôVLm ©sû[
A) LôdûLlTô¥²Vôo C) TôpYiQl ©sû[
B) I[ûYVôo D) ©sû[l ùTÚUôs HVeLôo
C) B[e¸W]ôo 94. TôW§RôNu FݧV "©£WôkûRVôo'
D) L©Xo Fuàm èp?
88. NeL BXd¡VeL°u ùUôjR @¥Ls A) L®ûR
FjRû]? B) DûWSûP
A) 26,350 C) £ßLûR
B) 26,375 D) SôPLm
C) 26,400 95. ùTôÚjÕL:
D) 26,411 èp A£¬Vo
89. "SôPLf NôûXùVôjR StLXôNôûX a) Ï«p TôhÓ 1. @lÕp WÏUôu
ùVôuß ¿ÓX¡p DiúPô ¨LrjÕ' b) @Z¡u £¬l× 2. ÑWRô
Fuß á±VYo? c) Õû\ØLm 3. TôW§Vôo
A) TôW§Vôo d) Tôpŧ 4. TôW§RôNu
B) SôUdLp L®Oo
(a) (b) (c) (d)
C) NeLWRôÑ ÑYôªLs
A) 3 4 2 1

 A-Series in
          ©lWY¬ 2020
B) 3 4 1 2 A) ¨ëhPu @Ô Y¥Ym
C) 3 1 2 4 B) úTôo @Ô Y¥Ym
D) 1 2 3 4 C) ìRo úTôoÓ @Ô Y¥Ym
96. ""SWLj§p BPolTúPôm SPûX D) úNôUoÀpÓ @Ô Y¥Ym
BpúXôm'' #Bdát±p "SPûX' 103. Av©¬u UÚk§u úY§lùTVo
Fuàm ùNôp-u ùTôÚs @±L? Fu]?
A) úSôn B) TôÕLôl× A) PôoPô¬d @ªXm
C) ÕuTm D) FUu B) ùTu^ô«d @ªXm
97. TôgNô- NTRj§p BPm ùTt\ C) @£ûPp Nô-£-d @ªXm
TôPpL°u Fi¦dûL VôÕ? D) ùTo Ïú[ô¬d @ªXm
A) 401 B) 405 104. RôYW TpYûLûU ªL @§LUôL
C) 410 D) 412 LôQlTÓYÕ?
98. TXThPûPf ùNôdLSôR ©sû[ A) ùYlT UiPX LôÓLs
FݧV Utù\ôÚ çÕ èp FÕ? B) ªR ùYlT UiPX LôÓLs
A) ùRu\p ®Ó çÕ C) TôûXY]eLs
B) @u]m ®Ó çÕ D) ®YNôV ¨XeLs
C) Rªr ®Ó çÕ 105. ¸úZ ùLôÓdLlThÓs[ Yôd¡Ve
D) Ø¡p ®Ó çÕ Lû[ LY²dL
99. RgûN úYRSôVL Nôj§¬Vôo # @Yo 1. BWjRUô]Õ ÖûWÂW-p BÚkÕ
L°u A£¬Vo Vôo? BPÕ Fh¬Vm Y¯VôL ùNuß
A) ÑYôohv Tô§¬Vôo BPÕ ùYu¼¬dû[ @ûPkÕ
B) LôpÓùYp ©u]o DP-u Ut\ TϧLÞdÏ
ùNp¡\Õ.
C) ÅWUôزYo
2. DP-p BÚkÕ ùT\lThP BWjR
D) Ff.G. ¡Úh¥]l©sû[
Uô]Õ YXÕ Gh¬Vm Y¯VôL
100. DjRWLôiPm Fuàm TϧûV ùNuß YXÕ ùYu¼¬dLû[
BVt±VYo? @ûPkÕ ©u]o ÖûWÂWÛdÏ
A) IhPdájRo ùNp¡\Õ.
B) YôuÁ¡ 3. D P - p B Ú k Õ ù T \ l T h P
C) ×LúZk§Vôo BWjRUô]Õ BPÕ Gh¬Vm
D) LmTo Y¯VôL ùNuß YXÕ ùYu¼¬d
101. @ÔLÚ DûX«p Ds[ LhÓlTôhÓ Lû[ @ûPkÕ ©u]o ÖûWÂW
úLôsLs ÛdÏ ùNp¡\Õ.
A) ¨ÙhWôuLû[ D±gÑm úUp ùLôÓdLlThP Yôd¡VeL°p
B) ¨ÙhWôuL°u úYLjûR @§L¬dÏm FÕ/FûY N¬Vô]ûY?
C) ¨ÙhWôuL°u úYLjûR Ïû\dÏm A) 1 UhÓm
D) DûX«p ùY°VôÏm @§L @[Ü B) 1 Utßm 2
ùYlTjûR D±gÑm C) 2 Utßm 3
102. FkR @Ô Uô§¬ Y¥Ym ¸rLiP D) 1, 2 Utßm 3
átû\ ùLôiPÕ? 106. ©uYÚY]Ytßs, Ï¥¿o Utßm
""FXdhWôuLs @Ô DhLÚûY Tôp UôÑTÓYRôp TWYd á¥V
Ñt± @RtLô] YhPlTôûR«p úSôn FÕ?
UhÓúU Ñt±YÚm'' A) vLo®

©lWY¬ 2020 A-Series in


  
B) UúX¬Vô B) ÀLôo
C) ùPeÏ C) £d¡m
D) ûPTônÓ D) @v^ôm
107. A¬V LPÜsL°p úTôo LPÜs 113. FkR YhPúUû_ UôSôh¥p Lôk§
Utßm LôX¨ûX LPÜ[ôL LXkÕ ùLôiPôo?
BÚ T¦Ls ùNnÙm LPÜ[ôL A) ØRp UôSôÓ
LÚRlTÓTYo Vôo? B) êu\ôm UôSôÓ
A) Bk§Wu C) BWiPôm UôSôÓ
B) YÚQu D) IuߪpûX
C) YôÙ 114. ×® ùYlTUVUôR-u TôoûY«p
D) Új§Wu "ÏZkûR úVÑ' Fu\ ùTVo BRàPu
108. ùNuû] DVo¿§ Uu\j§p ØRu ùRôPo×ûPVÕ?
ØRXôL ¨VªdLlThP Bk§V A) Fp ¨ú]ô
¿§T§ Vôo? B) Xô ¨ú]ô
A) úNjço NeLW SôVo C) TÑûU BpX ®û[Ü
B) úYeLPWôU ¡ÚxQÑYôª HVo D) B[g£Yl× ×Wh£
C) T. ØjÕÑYôª HVo 115. @ V ] U i P X ã \ ô Y ° V ô p
D) ÑlûTVo Ñl©WU¦V HVo ªLÜm @§LUôL Tô§dLlTÓm
109. ¸rLiPûYL°p FûY N¬Vô] Uô¨XeL°u ùTVoLs VôûY?
BûQVôL úNodLlThÓs[Õ? A) RªrSôÓ # úLW[ô
A) ×jRo # LPÜs Gt×m Ußl×m B) J¥Nô # Ak§W©WúRNm
BpXôRYo C) LoSôPLm # J¥Nô
B) ×jRo ©\l× # ϧûW D) úLW[ô # LoSôPLm
C) ×jRo ØRp NUV TWl×ûW # 116. DsSôhÓ ¿oY¯ RPeLû[ ®¬YôdL
RôUûW Lôû[ ùRôPeLlThP §hPm
D) Fi YûL UôodLm # §¬N]ô A) _p LuVô §hPm
110. "LôUuùYpj' YôW Tj§¬dûLûV B) _p Uôod ®Lôv §hPm
ùRôPe¡VYo Vôo? C) TôWj UôRô §hPm
A) TôX LeLôRW §XLo D) DRn §hPm
B) C.R. Rôv 117. ùRô¯pLs S-Ü @ûPYRtLô]
C) @u²ùTNuh @mûUVôo ×\dLôW¦Ls
D) úUô§Xôp úSÚ I. ªu RhÓlTôÓ
111. BWiPôm DXLúTô¬p _lTôu II. ¨§ ©Wf£û]Ls
úSNSôÓL°Pm NWQûPkR ©u× III. @W£u ùLôsûL Uôt\eLs
ÑTôx Nk§WúTôv FeÏ ùNu\ôo? IV. @§L LPu
A) Av§úW-Vô®Ûs[ £h² A) I Utßm IV
B) £eLléo B) II UhÓm
C) KTôoúUôNô®Ûs[ ûRlúT C) I Utßm III
D) ùRuùLô¬Vô®Ûs[ £úVôp D) IV UhÓm
112. F k R B P j § p @ ú L ô m × W h £ 118. úY[ôiûU ®ûX ùLôsûL«u
ùY¥jRÕ? Ød¡V LÚ®VôL BÚlTÕ FÕ?
A) §¬×Wô

 A-Series in
          ©lWY¬ 2020
A) ùTôÚsLû[ úRokùRÓjRp YhPúUû_ UôSôÓ 3. 1927
B) DfN ThN ùLôsØRp ®ûX d) YÏl× Yô¬ Øû\ 4. 1931
C) Ïû\kRThN ARWÜ ®ûX (a) (b) (c) (d)
D) Ïû\kRThN £pXû\ ®ûX A) 1 4 3 2
119. LôouYô-v ©W× ùLôiÓ YkRÕ B) 3 1 4 2
A) WVjYô¬ Øû\ C) 3 1 2 4
B) _ô¡oRô¬ Øû\ D) 2 4 1 3
C) ULpYô¬ Øû\ 124. © u Y Ú m @ h P Y û Q L ° p
D) _ÁuRô¬ Øû\ ùUô¯Ls Tt±V @hPYûQ FÕ?
120. @W£VXûUl× (44#YÕ §ÚjRf) A) FhPôYÕ @hPYûQ
NhPm 1978, ùNôjÕ¬ûUûV B) T§ú]ôWôYÕ @hPYûQ
I. @¥lTûP D¬ûU«-ÚkÕ NhP C) SôuLôYÕ @hPYûQ
D¬ûUVôL Uôt±VÕ D) A\ôYÕ @hPYûQ
II. NhP D¬ûU«-ÚkÕ @¥lTûP 125. Bk§Vj RûXûUd LQdÏ Utßm
D¬ûUVôL Uôt±VÕ R¦dûLj Õû\ RûXYo VôÚdÏ
III. @¥lTûP D¬ûU«-ÚkÕ ùTôßl×ûPVYo?
@¥lTûPd á\ôL Uôt±VÕ A) Ï¥VWÑj RûXYo
úUtLiP átßL°p FÕ N¬Vô]Õ? B) ©WRU @ûUfNo
A) I UhÓm C) DfN ¿§Uu\m
B) II UhÓm D) TôWôÞUu\m
C) III UhÓm 126. IÚ ¡¬dùLh ®û[Vôh¥p ØRp
D) II Utßm III UhÓm 10 JYoL°u JhP ÅRm 3.2 F²p
121. EZp ׬kR @WÑ E¯VûW T¦«p 282 Fu\ BXdûL @ûPV ÁRØs[
BÚkÕ ¿dL @WNûUl× Dßl× 311 40 JYoL°u JhP ÅRm Fu]?
§ÚjRTP úYiÓm F] T¬kÕûWjR A) 6.25 B) 6.5
Lªh¥ C) 6.75 D) 7
A) P.C. úaôhPô ÏÝ 127. IÚ ùNqYL Y¥Y YV-u TWlT[Ü
2
B) ¨oYôL ºo§ÚjR AûQVm 240 Á . @RàûPV ¿[lTdLj
C) @àUkR Wôq ÏÝ §-ÚkÕ 8 ùNÁ. Ïû\jRôp @Õ
D) NkRô]m ÏÝ IÚ NÕWUôÏm. @Ru ¿[m, @LXm
122. ""Bk§V DfN¿§Uu\m DX¡u FkR Øû\úV
Sôh¥Ûs[ DfN ¿§Uu\jûR A) 12 ùN.Á, 20 ùN.Á
®PÜm @§L @§LôWm DûPVÕ B) 12 ùN.Á, 8 ùN.Á
AÏm'' BqYôß á±VYo Vôo? C) 20 ùN.Á, 12 ùN.Á
A) @pXô¥ ¡ÚxQÑYôª @nVo D) 20 ùN.Á, 8 ùN.Á
B) H.C. Uôjço 128. ùRôPo RsÞT¥Ls Øû\úV 15%
C) M. TdRYjNXm Utßm 25% Fu\Yôß RWlThÓ
D) Dr. BWôú_k§W ©WNôj IÚ NXûY BVk§Wm ì.15,000dÏ
123. ùTôÚjÕL: ®tLlThPÕ F²p @Ru ϱjR
®ûX Fu]?
a) ûNUu ÏÝ 1. 1928
b) úSÚ @±dûL 2. 1932
A) ì. 25,500
c) BWiPôYÕ B) ì. 23,000

©lWY¬ 2020 A-Series in


  
C) ì. 20,000 11352
@[Ü L.ùNÁ. Utßm @Ru
D) ì. 25,000 7
ùY° AWm 8 ùN.Á F²p @dúLô[j
129. 8 AiLs Utßm 12 £ßYoLs §u Ds AWjûRd LôiL.
úNokÕ IÚ úYûXûV 10 A) 6 ùN.Á B) 5 ùN.Á
SôhL°p ùNnÕ Ø¥lTo. @úR
C) 7 ùN.Á D) 8 ùN.Á
úYûXûV 6 AiLs Utßm 8
£ßYoLs úNokÕ 14 SôhL°p 133. TôÛm ¿Úm LXkR BÚ LXûYLs
ùNnÕ Ø¥lTo. IÚ Ai R²VôL Ds[]. ØRp LXûY«p Tôp, ¿¬u
@qúYûXûV FjRû] SôhL°p ®¡Rm 5:3 F]Üm BWiPôYÕ
ùNnÕ Ø¥lTôo? IÚ £ßYu LXûY«p Tôp, ¿¬u ®¡Rm 5:4
R²VôL @qúYûXûV FjRû] F] Ds[Õ BÚ LXûYûVÙm FkR
SôhL°p ùNnÕ Ø¥lTôu? ®¡Rj§p LXkRôp TôÛm ¿Úm 4:3
A) Ai 140 SôhLs, £ßYu 280 Fu\ ®¡Rj§p BÚdÏm?
SôhLs A) 4 : 3 B) 3 : 4
B) Ai 280 SôhLs, £ßYu 140 C) 3 : 1 D) 1 : 3
SôhLs 134. IÚ Y¬ûN«p £X úTo ¨uß
C) Ai 150 SôhLs, £ßYu 300 ùLôi¥Úd¡u\]o. IÚ ÏZkûR
SôhLs @kR Y¬ûN«p BPªÚkÕ 14#YÕ
D) Ai 300 SôhLs, £ßYu 150 BPj§Ûm IÚ ùTi YXªÚkÕ
SôhLs 8#YÕ BPj§Ûm ¨t¡u\]o.
130. ¸rLiPYtßs, ùPeÏ ûYWv ÏZkûRdÏm, ùTi¦tÏm BûP«p
FkR ÑZt£ Øû\«p TWÜ¡\Õ? 4 STo BÚdÏm @kR Y¬ûN«p
Ds[ ùUôjR SToL°u Fi¦dûL
A) U²Ru#U²Ru
Fu]?
B) U²Ru#T\ûY#U²Ru
A) 25 B) 26
C) U²Ru#ùLôÑ#U²Ru
C) 29 D) 32
D) U²Ru#Tu±#U²Ru
135. ØRp 100 BVp FiL°u NWôN¬
131. A, B BÚYÚm IÚ úYûXûV 12 Vô]Õ?
SôhL°p ùNnÕ Ø¥lTo. B, C
A) 55 B) 50.5
@úR úYûXûV 15 SôhL°p ùNnÕ
Ø¥lTo. C, A @úR úYûXûV 20 C) 51.5 D) 50
SôhL°p ùNnÕ Ø¥lTo. êYÚm 136. 2 , 6 , 18 .... Fu\ ùTÚdÏ ùRôPo
úNokÕ Utßm R²jR²VôLÜm 5 25 125
@qúYûXûV FjRû] SôhL°p Y¬ûN«u (G.P) ùTôÕ ®¡Rm LôiL.
ùNnÕ Ø¥lTo? 3 4
A) B)
A) úNokÕ = 10 SôhLs R²jR²VôL 5 5
A = 30, B = 20, C = 60 1 2
C) D)
B) úNokÕ = 15 SôhLs R²jR²VôL 5 5
A = 35, B = 25, C = 65
137. 8 Ap YÏTÓm êu±XdL BVp
C) úNokÕ = 20 SôhLs R²jR²VôL FiL°u Fi¦dûLûVd LôiL.
A = 40, B = 30, C = 70 A) 112 B) 110
D) úNokÕ = 25 SôhLs R²jR²VôL C) 111 D) 108
A = 45, B = 35, C = 75
138. ¥.Fu.G. Y¬d ϱ«ÓR-u RkûR
132. IÚ DsÇPt\ úLô[j§u L] F]d LÚRlTÓTYo Vôo?

 A-Series in
          ©lWY¬ 2020
A) vªj B) d¬vÀpv C) @ÚQôfNX ©WúRNm Utßm
C) Tôp ùaToh D) Lôoh]o @v^ôm
139. ù Y s û [ V ô û ] ú R N m F u ß D) TgNôl Utßm BUôfNX ©WúRNm
@ûZdLlTÓm SôÓ FÕ? 146. "§ KTôp AKl F vTôoúWô' Fu\
A) ªVôuUo B) ®VhSôm èûX FݧVYo?
C) RônXôkÕ D) ©-lûTuv A) YkR]ô µYô
140. ¨¥ AúVôd FlúTôÕ SûPØû\dÏ B) @ÚkR§ Wôn
ùLôiÓ YWlThPÕ? C) ´m LôoùTh
A) 2013 B) 2014 D) NÄm @-
C) 2015 D) 2016 147. 2011#Am AiÓ UdLsùRôûL
141. R ª r S ô h ¥ p F u ± - Ú k Õ LQdùLÓl©uT¥, Ïû\Yô]
©[ôv¥d ùTôÚhLs TVuTÓj UdLs @Poj§ T§Ü ùNnVlThP
ÕYÕ RûP ùNnVlThÓs[Õ? Uô¨Xm?
A) ALvh 15, 2018 A) ªú_ôWm
B) _]Y¬ 1, 2019 B) £d¡m
C) _]Y¬ 26, 2019 C) @ÚQôfNX ©WúRNm
D) ALvh 10, 2018 D) SôLôXôkÕ
142. .................................... BXôdLôYôp ÑYf 148. B . N d § F u \ § h P m F k R
(çnûUT¬VRôu) l¬VRôu @ûUl©u ¸r ùRôPeLlThPÕ?
@ûXúT£ ùNV- ùNVpTÓjRl A) ùN©
TÓ¡\Õ B) STôoÓ
A) Y]jÕû\ C) Bk§V úY[ôiûU AWônf£
B) ÑtßXô Y[of£ D) ¡WôUl×\ ªuUVUôdÏm LZLm
C) ÑLôRôWm 149. ¸úZ ùLôÓdLlThÓs[ átß
D) U²R Y[m FRû] ϱd¡\Õ?
143. Ñ l W U ¦ V m T ô X ô ´ U t ß m ""IÚ ùTôÚ°u ùYlT¨ûXûV IÚ
RªrSôÓ @WÑ Fu\ YZd¡tÏ ¥¡¬ DVojÕYRtÏ úRûYlTÓm
ùRôPo×ûPV ®YLôWm ùYlTUôÏm''.
A) úRoRp @±dûL«u Ds[PdLm A) ùYlT LPjRp
B) úRoRp SPjûR ®§Ls B) ùYlT L§oÅfÑ
C) BûPjúRoRp C) ùYlT NX]m
D) úYhTô[¬u Rϧ D) ùYlT Gtק\u
144. Bk§Vô®p FkR AiÓ ØRu 150. IÚ ùTôÚs SLÚm çWm @Õ
Øû\VôL @Ô AÙR úNôRû] SLoYRtÏ FÓjÕd ùLôsÞm
SPjRlThPÕ? úSWj§u BÚU¥VôL (YodLm)
A) 1998 B) 1999 BÚdÏmúTôÕ @Ru BVdLm FkR
C) 1974 D) 1975 YûLûVf NôokRÕ?
A) ºWô] Ïû\ ØÓdL BVdLm
145. Bk§Vô®p ªL ¿[Uô] NôûX
Utßm W«p TôXm ©uYÚm FkR B) ºWt\ ØÓdL BVdLm
BÚ Uô¨XeLû[ BûQd¡\Õ? C) ºWô] ØÓdL BVdLm
A) Ak§W ©WúRNm Utßm ùRÛeLô]ô D) úUp á±VûY FÕÜm BpûX
B) Wô_vRôu Utßm Ï_Wôj 151. ªLÜm @±ØLUô] "LôUôd³u'

©lWY¬ 2020 A-Series in


  
Fu\ éf£dùLôp- UÚk§u D) úYßThP B]eLû[ úNokR
úY§lùTVo Fu]? ®XeÏLs ùYqúYß BPj§p
A) DDT YôÝm ùTôÝÕ
B) ùTuÊu ùad^ô Ïú[ôûWÓ 157. U²R StTi× úRoÜ FuTÕ
C) EDTA A) D«¬Lû[ Tô§dÏm LôWQjûR
D) RôúXh @±YÕ
152. Tôd¥¬VôdLs êXdáß ûShW_û] B) úSô«û] ÏQlTÓjÕYÕ
¸rdLiP FkR úNoUUôL Uôtß¡\Õ? C) U²R B]jûR úUmTÓjÕYÕ
A) HNO3 D) UWTÔ £¡fûN Øû\
B) @ªú]ô @ªXeLs 158. Bk§V UdLsùRôûL«p FjRû]
C) NO2 Ød¡V B]eLû[ ùLôiPÕ?
D) NH3 A) Aß Ød¡V B]eLû[ Ds[Pd¡
VYoLs
153. ´lNm FuTÕ ¸ZdLiPYt±p FÕ?
B) HkÕ Ød¡V B]eLû[ Ds[Pd
A) CaCO3.2H2O
¡VYoLs
B) Ca(OH)2 C) S ô u Ï Ø d ¡ V B ] e L û [
C) CaSO4 .2H2O Ds[Pd¡ VYoLs
D) MgSO4 .7H2O D) GÝ Ød¡V B]eLû[ Ds[Pd
154. ©uYÚY]Ytßs RôYWeL°u ¡VYoLs
LôoúTô#ûahúWhÓL°u 159. FkR Bk§V Uu]o YôvúLôPLôUô
úNªl×l ùTôÚ[ôLÜm, DQîhPj úLô¯dúLôÓ Yk§\e¡V ùTôÝÕ
§p At\-u ØRuûU ARôWUôLÜm @YûW Nk§jRôo?
®[eÏYÕ FÕ? A) TÏYuXôp
A) ÏÞúLôv B) ùNpÛúXôv B) NôúUô¬u
C) RWNm D) lWdúPôv C) Wôú_k§W SôVo
155. IjR RuûU DûPV ÏúWôúUôúNôm D) £Wôw#Dj#ùR[Xô
LÞdÏ BûPúV Ëu Uôt\m 160. `¬Vj, FkR Y¬ @àU§dLlTP
SûPùTßm ùRôϧ ®pûX?
A) û^dúLô¼u A) ®YNôV Y¬
B) úTdûL¼u B) Øv-m @pXôúRôo Y¬
C) ¥lú[ô¼u C) YojRL Y¬
D) PVôûLù]³v D) §ÚUQ Y¬
156. ®XeÏL°ûPúV DQÜ, ReϪPj 161. ÑRk§Wô Lh£ûV úRôtß®jRYo
§tLô] úTôh¥ @§LUôL BÚlTÕ, Vôo?
FlùTôÝÕ? A) O.P. WôUNôª B) P.S. ÏUôWNôª
A) IúW B]jûR úNokR ®XeÏLs C) Wô_ô´ D) LôUWôw
IúW BPj§p YôÝm ùTôÝÕ
162. _ Y a o X ô p ú S Ú © \ k R E o
B) úYßThP B]eLû[ úNokR
............................?
®XeÏLs IúW BPj§p YôÝm
A) @ÜWeLôTôj
ùTôÝÕ
B) AdWô
C) IúW B]jûR úNokR ®XeÏLs
ùYqúYß BPj§p YôÝm C) cSLo
ùTôÝÕ D) @XLôTôj

 A-Series in
          ©lWY¬ 2020
163. ù N [ ¬ # ù N [ W ô ¡ W ô U m F e Ï C) Y\iP T¦ úUmTôÓ
@ûUkÕs[Õ? D) úUtLiP @û]jÕm
A) Uj§V ©WúRNm 170. C. NfNôo ...................... u ùRôPo×ûPVÕ?
B) Dj§Wl ©WúRNm A) ùToWô NhPm
C) Ï_Wôj B) H.Ao.¥.G. NhPm
D) ULôWôx¥Wô C) Fm.Ao.¥.©. NhPm
164. 1928 úSÚ @±dûLVô]Õ FRû] D) ¨ßY]f NhPm
£Tô¬Ñ ùNnVlThPÕ? 171. ùLôÓdLlThP át±p FÕ ©WRU
A) éWQ ÑRk§Wm Uk§¬«u T«o TôÕLôl× §hPm
B) Ru]ôh£ @kRvÕ 2016#u N¬Vô]Õ?
C) áhPôh£ (@) ÑúRN @WÑLs I. úY[ôiûUûV BVtûL úTW¯®p
D) ©¬h¥x ùTôÕ NûT«p BÚkÕ LôlTÕ
Dßl©]o AÏRp II. @ÓjR YÚPj§tÏ T«o ùNnV
LPu ùT\ RϧVôRp
165. Bk§Vô®p ØRu ØR-p ùUhúWô
W«p §hPm @ûUdLlThP SLWm? úUúX ùLôÓdLlThÓs[ át±p FÕ
N¬Vô]Õ?
A) ùNuû]
A) I UhÓm
B) ùLôpLjRô
B) I Utßm II
C) ùPp-
C) II UhÓm
D) ØmûT
D) I Utßm II BpûX
166. Bk§Vô®p @§L RWm YônkR
LÚûUVô] ûUdLô FkR Uô¨Xj§p 172. ¸ r L i P Y ô d ¡ V j û R F Ý R Ü m
BÚkÕ ¡ûPd¡\Õ? Utßm RY\ô] Yôd¡VjûR úRok
ùRÓdLÜm?
A) úUtÏ YeLô[m
I. U j § V , U ô ¨ X N h P e L û [
B) Dj§W©WúRNm
SûPØû\lTÓjÕYRtÏ ¿§ Õû\
C) ÀLôo @§LôWUô]Õ T¡WlThÓs[Õ
D) úLW[ô II. Itû\Vôh£ Øû\ BÚlTRôp
167. ©uYÚY]Ytßs ¿ûW ®P Ïû\ @û]jÕ @§LôWeLÞm Uj§«p
Yô] @Poj§ ùLôiP úLôs FÕ? UhÓm Ds[].
A) N² B) ùYs° III. Uj§V Uô¨X @WÑ D\ÜL°p
C) Å]v D) ®VôZu IÚe¡ûQlúT Ød¡V @mNm
168. úY[ôiûU Uô²Vm Fu\ôp AÏm
Fu]? IV. ùTôÕlTh¥Vp ùRôPoTô]
A) ùRô¯p Y[of£dÏ @°lTÕ NhPj§u ÁÕ ØWiTôÓ
B) YÚUô]j§tÏ ARWYôL @°lTÕ úRôußUô]ôp, Uj§V @WÑ
C) ®YNô««u DtTj§dÏm @Y²u Uô¨XeLs NhPjûR Uôt\
YÚUô]j§tÏ ARWYôL @°lTÕ Ø¥VôÕ
D) ¨§ Y[of£dLôL @°lTÕ A) III UhÓm
B) I Utßm II
169. U ² R Y [ ú U m T ô h ¥ p , T ¦
úUmTôh¥u ùTôÚs Fu]? C) II Utßm III
A) T¦Vô[oL°u ¨ûX UôßRp D) I, II Utßm IV BpûX
B) T¦Vô[o IÚ T¦¨ûX«-ÚkÕ 173. Bk§V áhPôh£«u £\lTmNe
DVokR T¦dÏ UôßRp L[ô]ûY

©lWY¬ 2020 A-Series in


  
1) @§LôWl ©¬Ü A) 1:4 B) 1:8
2) @§LôWl T¡oÜ C) 1:1.6 D) 1:2
o
3) ÑRk§WUô] ¿§Uu\m 180. 88 u ªûL ¨Wl×d úLôQm ................... .
4) ©WRU¬u RûXûU A) 2o B) 92o
5) FÝRlThP @W£VXûUl× C) 80o D) 98o
A) 1, 2 Utßm 5 181. I Ú Y o 1 0 A i Ó L ° p ú N o j R
B) 1, 3 Utßm 5 ùUôjRj ùRôûL ì. 16,500/# ØRp
C) 1, 4 Utßm 5 YÚPj§tÏl ©\Ï IqùYôÚ
AiÓm ØkûRV AiûP ®P
D) 2, 3 Utßm 5
ì.100 @§LUôL úNªjRôp ØRp
174. Uj§V ×X]ônÜ AûQVjûR FkR YÚP úNªl× ùRôûLûVd LôiL.
ÏÝ T¬kÕûW ùNnRÕ?
A) ì. 1,100 B) ì. 1,200
A) TôWôÞUu\ ÏÝ
C) ì. 1,000 D) ì. 1,250
B) ¨§ ÏÝ
182. ÑÚdÏL.
C) NkRô]m ÏÝ
3 7
D) úUjRô ÏÝ 7 +  −10 
10  21 
175. ¸ r L i P Y t ß s F Õ U ô ¨ X l
Th¥V-p úNWôRÕ? 7 7
A) − 3 B) 3
A) Áu©¥ ùRô¯p 210 210
B) Ds[ôh£ @WNôeLm 3 3
C) L²UeLs ÁRô] Y¬ D¬ûULs C) 7 D) − 7
210 210
D) Ød¡V Õû\ØLeLs
183. IÚ LûPdLôWo Ru Yô¥dûLVô[o
176. Uj§V ¨oYôLj ¾olTôVm BRàPu LÞdÏ 10% RsÞT¥ RkÕm
ùRôPo×ûPVÕ 20% BXôTm @ûP¡u\ôo. IÚ
A) T¦«p As úNojRp ùTôÚ°u DiûU ®ûX ì.450
B) TR® DVoÜ F²p, @lùTôÚ°u ϱjR
C) IÝeÏ SPY¥dûL ®ûXûVd LôiL.
D) T¦«p AsúNolTÕ Utßm A) ì. 500 B) ì. 550
@û]jÕ úNûYLs C) ì. 600 D) ì. 650
177. ©uYÚm ÏÝ®p NØRôV Y[of£ 184. 15 SôtLô-L°u ®ûX ì.7,500 F²p
§hPj§p UdL°u TeL°lûT ì. 12,000dÏ FjRû] SôtLô-Ls
T¬kÕûWjRÕ FkR ÏÝ? YôeLXôm?
A) @úNôd úUjRô ÏÝ A) 24 B) 26
B) TpYkRWôn úUjRô ÏÝ C) 28 D) 30
C) ¡WôUl×\#SLol×\ D\Üd ÏÝ 185. (?-968)÷79×4=512.
D) Fp.Fm. £eL® ÏÝ A) 10185
178. 32:13::29:? B) 10190
A) 10 B) 85 C) 11080
C) 42 D) 12 D) 10180
186. «p Utßm
179. R e L f ù N q Y L j § u T d L e L s
F²p u U§l×
úRôWôVUôL FkR ®¡Rj§p @ûUk
VôÕ?
§ÚdÏm?
A) 45o B) 70o

 A-Series in
          ©lWY¬ 2020
C) 55o D) 65o §hPj§u úSôdLm Fu]?
187. Aß ÏÓmTeL°u UôR YÚYôn A) ¡WôUeLû[ UôSLWeLú[ôÓ
Øû\úV ì. 3,500. ì. 2,700, ì. 3,000, BûQjRÕ
ì. 2,800, ì. 3,900 Utßm ì. 2,100 B) UôYhPeLû[ UôSLWeLú[ôÓ
F²p YÚYô«u NWôN¬ûVd LôiL. BûQjRÕ
A) ì. 2,700 B) ì. 3,900 C) IqùYôÚ ¡WôUjûRÙm Ød¡V
C) ì. 3,000 D) ì. 3,500 SLWeLú[ôÓ BûQjRÕ
188. AiÓ Yh¥ ÅRm 2% ÅRm 3 D) IqùYôÚ ¡WôUeLû[Ùm Iuú\ôÓ
AiÓLÞdÏ R² Yh¥ ì. 300 Iuß BûQjRÕ
¡ûPdÏm F²p @NûXd LôiL. 194. ©uYÚm LÚjÕÚûY GtßdùLôs:
A) ì. 5,000 B) ì. 3,000 1. YVSôÓ Y]®XeÏ NWQôXVm
C) ì. 2,000 D) ì. 1,000 úLW[ô®p Ds[Õ.
189. BÚ TLûPLs IÚ úNW DÚhPl 2. £úWô úR£V éeLô úULôXVô®p
TÓmúTôÕ, ØRp TLûP«p 4u Ds[Õ.
LôW¦Ls ¡ûPdLl ùTßYRtLô] 3. TLôUWô úULôXVô®p Ds[Õ.
¨LrRLÜ Fu]? úUúX á\lThP FkR LÚjÕ N¬?
1 1 A) 1, 2, 3 B) 1, 3
A) B)
18 36 C) 1, 2 D) 2, 3
1 1 195. @u]éWQ U¡Zô UiPp §hPm
C) D)
2 3 áßYÕ?
190. © u Y Ú m Ø ¥ ® - j ù R ô P ¬ u A) UL°ÚdÏ @§LôWU°jRp
áÓRp LôiL. B) ùRô¯Xô[oLÞdÏ @§LôWU°jRp
108, 36, 12, 4......... C) B û [ O o L ° u § \ û U L û [
A) 216 B) 162 úUmTÓjÕYÕ
C) 160 D) 256 D) ÏZkûR ùRô¯Xô[o I¯l×
191. N o Y £ d ` ô @ © V ô u § h P m 196. TôWôÞUu\j§u 124#YÕ NhPj
(@û]YÚdÏm Lp® §hPm) §ÚjRj§u êXm 10% BP IÕd¸Ó
2001#Am AiÓ ØRp @ØpTÓjRl YZeLlThPÕ?
ThPRu Ød¡V úSôdLm A) Th¥Vp B]jRYo
1. קV Ts°Ls B) TZeÏ¥«]o
2. Ts°«p Re¡ T¥dÏm YN§ C) ªLÜm ©tTÓjRThPYo
GtTÓjÕRp
D) ùTôÚ[ôRôWj§p ©u Re¡V
3. TôÕLôl×Pu á¥V úTôdÏYWjÕ ©¬®]o
4. ºÚûP Utßm U§V DQÜ
197. ×®«u ùYt± ÅWo Fu\ ®ÚûR
¸ ú Z D s [ á t ± p F Õ / F û Y YZeÏYÕ
N¬Vô]ûY @pX A) H.Sô. NûT B) ©¬dv
A) 1 Utßm 2 B) 2 Utßm 3 C) Nôod D) @W× Äd
C) 3 Utßm 1 D) 1 Utßm 4 198. Bk§Vô®p FlúTôÕ "úR£V ùYs[
192. ""úR£V TÑûU ¾olTôVm'' FkR AnÜd ÏÝ' ¨ßYlThPÕ?
AiÓ @ûUdLlThPÕ? A) 1982 B) 1996
A) 2009 B) 2010 C) 1972 D) 1976
C) 2011 D) 2014 199. ©uYÚm ùUô¯L°p, FkR ùUô¯
193. ©WRôu Uk§¬ ¡Wôªu NRd úVô_]ô Hd¡V SôÓLs NûT«u @ÛYp

©lWY¬ 2020 A-Series in


  
ùUô¯VôL BpûX? FÕ?
A) º] ùUô¯ ×jRLm A£¬Vo
B) vTô²V ùUô¯ A) Tôi¥Vu T¬Ñ # TôW§RôNu
C) ùLô¬V ùUô¯ B) AXôTû] # @lÕp WyUôu
D) @úW©V ùUô¯ C) @Lp ®[dÏ # YWRWôN]ôo
200. ¸rdLiPYtßs RY\ô] BûQ D) _ô]¡WôUu # @lTô®u £ú]¡Ro

1. D 2. C 3. D 4. D 5. D 6. D 7. B 8. B 9. C 10. A 11. A 12. D

13. D 14. C 15. D 16. A 17. B 18. B 19. B 20. B 21. C 22. B 23. D 24. B

25. D 26. A 27. C 28. B 29. C 30. A 31. C 32. A 33. A 34. D 35. D 36. B

37. B 38. A 39. D 40. A 41. B 42. A 43. B 44. C 45. D 46. A 47. B 48. D

49. D 50. C 51. A 52. D 53. D 54. B 55. C 56. D 57. A 58. D 59. B 60. B

61. D 62. A 63. B 64. C 65. A 66. C 67. C 68. C 69. C 70. C 71. B 72. C

73. A 74. C 75. D 76. C 77. B 78. B 79. B 80. C 81. C 82. D 83. C 84. B

85. D 86. B 87. A 88. A 89. D 90. D 91. D 92. B 93. B 94. D 95. A 96. C

97. D 98. A 99. A 100. A 101. A 102. B 103. C 104. A 105. B 106. D 107. A 108. C

109. A 110. C 111. C 112. D 113. C 114. A 115. B 116. B 117. C 118. C 119. D 120. A

121. A 122. A 123. B 124. A 125. D 126. A 127. C 128. D 129. A 130. C 131. A 132. B

133. D 134. B 135. B 136. A 137. A 138. C 139. C 140. C 141. B 142. B 143. A 144. C

145. C 146. D 147. C 148. B 149. D 150. C 151. B 152. D 153. C 154. C 155. C 156. A

157. C 158. A 159. B 160. D 161. C 162. D 163. B 164. A 165. B 166. C 167. A 168. C

169 .B 170. C 171. B 172. D 173. B 174. C 175. D 176. D 177. B 178. B 179. C 180. B

181. B 182. A 183. C 184. A 185. C 186. D 187. C 188. A 189. C 190. B 191. D 192. B

193. C 194. B 195. A 196. D 197. A 198. D 199. C 200. D

 A-Series in
          ©lWY¬ 2020

You might also like